Vous êtes sur la page 1sur 140

Solutions

of
Electricity & Magnetism

Lesson 20th to 25th

By DC Pandey
20 Current Electricity
Introductory Exercise 20.1
q 2 pr 2.0
1. i = , here q = e, t = =
t v 8.43 ¥ 1028 ¥ 1.6 ¥ 10-19 ¥ 3.14
ev
\ i= ¥ (0.5 ¥ 10-3 )2
2pr
1.6 ¥ 10-19 ¥ 2.2 ¥ 106 = 1.88 ¥ 10-6 ms -1
=
2 ¥ 3.14 ¥ 5 ¥ 10-11 3. Yes.
= 1.12 ¥ 10 -3
A As current always flows in the direction of
electric field.
= 1.12 mA
4. False.
2. No. of atoms in 63.45 g of Cu = 6.023 ¥ 1023
In the absence of potential difference,
\No. of atoms in 1 cm 3 (8.89 g) of Cu
electrons passes random motion.
6.023 ¥ 1023
= ¥ 8.89 5. Current due to both positive and negative
63.54
ions is from left to right, hence, there is a net
= 8.43 ¥ 1022 current from left to right.
As one conduction electron is present per dq
6. i = 10 + 4 t fi = 10 + 4 t
atoms, dt
n = 8.43 ¥ 1022 cm -3 or 8.43 ¥ 1028 m -3 q 10
fi Ú0 dq = Ú0 (10 + 4t ) dt
As i = neAvd
i fi q = [10t + 2t 2 ]10
0 = 300 C
fi vd =
neA

Introductory Exercise 20.2


rL 0.49 ¥ 3.14 ¥ (0.42 ¥ 10-3 )2
1. R = =
A 2.75 ¥ 10-8
35
= 1.72 ¥ 10-8 ¥ 2 = 9.87 A
Ê 2.05 ˆ
3.14 ¥ Á ¥ 10-3 ˜ (b) V = EL = 0.49 ¥ 12 = 5.88 V
Ë 2 ¯
V 5.88
= 0.57 W (c) R = = = 0.6 W
i 9.87
E
2. (a) J= 3. Let us consider the conductor to be made up
r
of a number of elementary discs. The
EA conductor is supposed to be extended to form
fi i = JA =
r a complete cone and the vertex O of the cone
3
is taken as origin with the conductor placed rl
R=
along x-axis with its two ends at x = r and pr ( l + r ) tan 2 q
x = l + r. Let q be the semi-vertical angle of But, r tan q = a
the cone.
( r + l) tan q = b
Consider an elementary disc of thickness dx rl
at a distance x from origin. R=
p ab
Resistance of this disc,
dx 1
dR = r 4. True. r=
A s
1
If y be the radius of this disc, then \ r¥ s= ¥ s =1
s
A = py2 5. RCu = RFe
But y = x tan q
4.1 (1 + a Cu DT ) = 3.9 (1 + a Fe DT )
dx
dR = r 2 4.1 [1 + 4.0 ¥ 10-3( T - 20)]
px tan 2 q
= 3.9 [1 + 5.0 ¥ 10-3( T - 20)]
\Resistance of conductor
l +r rdx 4.1 + 16.4 ¥ 10-3( T - 20)
R = Ú dR = Ú
r px tan 2 q
2 = 3.9 + 19.5 ¥ 10-3( T - 20)
l +r
r È 1˘ 3.1 ¥ 10-3( T - 20) = 0.2
R= - 0.2
p tan 2 q ÍÎ x ˙˚ r T - 20 =
3.1 ¥ 10-3
r È1 1 ˘
= 2 Í - ˙ = 64.5∞ C
p tan q Î r l + r ˚
T = 84.5∞ C

Introductory Exercise 20.3


1. Potential difference across both the resistors VB = VA + 2 = 2 V
is 10 V. VC = VA + 5 = 5 V
i1 i2 VD = VC + 10 = 15 V
V - VB
\ i1 = C =3 A
1
2W 10 V 4W V - VA 15
and i2 = D = = 7.5 A
2 2
3. Current in the given loop is
E + 15
10 i=
Hence, i1 ==5 A 8
2
10 Ê E + 15 ˆ
VAB = E - 2i = E - 2 ◊ Á ˜ =0
and i2 = = 2.5 A Ë 8 ¯
4
2. As A is grounded, VA = 0 fi E =5 V
1W i1 B
4. Effective emf,
C
E = 8 ¥1 - 2 ¥1 = 6 V
5V Effective resistance of circuit
10 V 2V R1 i1
i2 i3

A 10 V R2 R3 10 V
D i2 2W
4
R = Rexternal + 10r = 2 + 10 ¥ 1 = 12 W E
6. i =
E 6 R+ r
\ i= = = 0.5 A
R 12 Also, V = E - ir
5. As R2 = R3 and V1 = V2 E-V
i=
Potential difference across R1 is zero. r
Hence, current through R1 fi i1 = 0 I I
(a) (b)
and current through R2
E E
V
fi i2 = 1 r r
R2
10
= =1 A
10 O R O E V

Introductory Exercise 20.4


1. 6 =1 - E
12 V r E = -5V
P i
Q
And from
E 1W
VST = VQP
1A
U R 6 = - ir + 12
12 - 6 6
r= = =2W
3W i 3
T S
2A 2. Power delivered by the 12 V power supply,
Applying KCL at junction R P1 = Vi = 12 ¥ 3 = 36 W
i=1 + 2 =3 A and power dissipated in 3 W resister,
VST = VRU = VQP P3 = i32 R3 = 22 ¥ 3 = 12 W
Taking VST = VRU

Introductory Exercise 20.5


E1 E2 E3 10 4 6
+ + + + E r
r r2 r3
1. E = 1 = 1 2 2
1 1 1 1 1 1
+ + + +
r1 r2 r3 1 2 2
10 + 2 + 3
=
2
R
= 7.5 V
1 1 1 1 Rate of dissipation of energy
and = + +
r r1 r2 r3 E2 R
P = i2 R =
1 1 1 ( R + r )2
= + + =2
1 2 2 For maximum or minimum power
1 dP
fi r= =0
2 dR
= 0.5 W È ( R + r )2 - 2 R ( R + r ) ˘
E fi E2 Í ˙ =0
2. i = Î ( R + r )4 ˚
R+ r
5
( R + r )( r - R) 5. G = 100 W, ig = 50 mA, i = 5 mA
fi E2 =0
( R + r )4 igG 50 ¥ 10-6 ¥ 100
\ S= =
E 2 ( r - R) i - ig 5 ¥ 10-3 - 50 ¥ 10-6
fi =0
( R + r )3 1 1
= =
fi R=r 1 - 0.01 0.99
d 2P È ( R + r ) 3
( - 1 ) - 3( r - R)( R + r )2 ˘ 100
= E2 Í = W
2 ˙ 99
dR Î ( R + r )6 ˚ 100
- E2(4r - 2 R) By connecting a shunt resistance of W.
= 99
( R + r )4 V
6. ig =
d 2P G
Clearly is negative at R = r. nV
dR2 and R = - G = ( n - 1) G
ig
Hence, P is maximum at R = r
E2r E2 15
and Pmax = = 7. VAB = E
( r + r) 2
4r 16
Potential gradient
3. When the batteries are connected in series V 15E
Eeff = 2E = 4V, reff = 2r = 2 W k = AB =
L 16 ¥ 600
For maximum power E
= V/cm
R = reff = 2W 640
2
Eeff (4)2 E
(a) = kL fi L =
E
= 320 cm
and Pmax = = =2 W
4reff 4 ¥ 2 2 2k
E 7E
4. I g = 5 mA, G = 1 W, V = 5 V (b) V = kl = ¥ 560 =
640 8
V 5
R= -G = -1 Also, V = E - ir
Ig 5 ¥ 10-3
7E
\ E - ir =
= 999 W 8
A 999 W resistance must be connected in E
i=
series with the galvanometer. 8r

AIEEE Corner
Subjective Questions (Level 1)
q ne 337.5
1. i = = = = 2.11 ¥ 1021
t t 1.6 ¥ 10-19
Given, 2 pr 1 v
4. T = fif = =
i = 0.7 , t = 1 s, e = 1.6 ¥ 10-19 C v T 2 pr
it 0.7 ¥ 1
\ n= = 2.2 ¥ 106
e 1.6 ¥ 10-19 =
2 ¥ 3.14 ¥ 5.3 ¥ 10-11
= 4.375 ¥ 108
= 6.6 ¥ 1019 s -1
2. q = it = 3.6 ¥ 3 ¥ 3600 q
I= = ef
= 38880 C T
3. (a) q = it = 7.5 ¥ 45 = 337.5 C = 1.6 ¥ 10-19 ¥ 6.6 ¥ 1019
q = 10.56 A
(b) q = ne fi n =
e
6
2 -8
5. (a) I = 55 - 0.65 t = 1.7 ¥ 10 W - m
dq l = 24.0 m
I=
dt Ê dˆ
2
Ê 2.05 ˆ
2
A = p Á ˜ = 3.14 ¥ Á ¥ 10-3 ˜
fi dq = Idt Ë2¯ Ë 2 ¯
fi q = Ú I dt = 3.29 ¥ 10-6 m 2
8 8 2 24.0
\ q = Ú Idt = Ú (55 - 0.65 t ) dt R = 1.7 ¥ 10-8 ¥
0 0 3.29 ¥ 10-6
8
È t2 ˘
= 55 [ t ]80 - 0.65 Í ˙ = 0.12 W
Î 2 ˚0 L
10. R=r
= 440 - 20.8 = 419.2 C A
rL
(b) If current is constant A=
q 419.2 R
I= = = 52.4 A
t 8 If D is density, then
6. i µ vd D r L2
m = DV = DA L =
vd 2 i2 R
\ = 8.9 ¥ 103 ¥ 1.72 ¥ 10-8 ¥ (3.5)2
vd1 i1 =
i2 6 . 00 0.125
fi vd 2 = vd1 = ¥ 1.20 ¥ 10-4 -2
= 1.5 ¥ 10 kg = 15 g
i1 1 . 20
11. At 20∞C,
= 6.00 ¥ 10-4 ms -1
i R1 = 600 W, R2 = 300 W
7. vd =
neA At 50∞C,
1 R1¢ = R1(1 + a 1Dt )
=
8.5 ¥ 1028 ¥ 1.6 ¥ 10-19 ¥ 1 ¥ 10-4 = 600 (1 + 0.001 ¥ 30) = 600 ¥ 1.03
= 0.735 ¥ 10-6 ms -1 = 618 W
= 0.735 mm/s R2¢ = R2(1 + a 2Dt )
l 103 = 300 (1 + 0.004 ¥ 30) = 336
t= =
vd 0.735 ¥ 10-6 \ R¢ = R1 ¢ + R2 ¢ = 618 + 336
= 1.36 ¥ 109 s = 43 yr = 954 W
R¢ - R 954 - 900
8. Distance covered by one electron in 1 s a= =
R ¥ Dt 900 ¥ 30
= 1 ¥ 0.05 = 0.05 cm
Number of electrons in 1 cm of wire R = 600 + 300 = 900 W
= 2 ¥ 1021 = 0.002 ∞ C -1
\ Number of electrons crossing a given area 12. As both the wires are connected in parallel,
per second VAl = VCu
= Number of electrons in 0.05 cm of wire iAl RAl = iCu RCu
= 0.05 ¥ 2 ¥ 1021 = 1020 L L
q ne iAl r Al Al2 = iCur Cu Cu 2
i= = p dAl p dCu
t t
iCu r Cu LCu
1020 ¥ 1.6 ¥ 10-19 fi dCu = dAl
= = 1.6 ¥ 10 = 16 A iAl r Al LAl
1
L 2 ¥ 0.017 ¥ 6
9. R = r = 1 ¥ 10-3
A 3 ¥ 0.028 ¥ 7.5
Given, = 0.569 ¥ 10-3 m
r = 0.017 mW - m = 0.569 mm.
7
V 0.938 20
13. (a) E = = = 1.25 V/m fi R2 = W = 1.82 W
L 75 ¥ 10-2 11
E 1.25 and R1 = 20 - R2 = 20 - 1.82 W
(b) J = fi r =
r 4.4 ¥ 107 = 18.2 W
r = 2.84 ¥ 10-8 W - m 17. The circuit can be redrawn as
E V
14. (a) J = =
r rL
Current density is maximum when L is 24 V 12W 8W
minimum, ie, L = d, potential difference
should be applied to faces with dimensions
2d ¥ 3 d.
V 8 ¥ 12
J min. = . Reff = = 4.8 W
rd 12 + 8
V VA V 24
(b) i = = I= = =5A
R rL Reff 4.8
Current is maximum when L is minimum 18. Here, A and C are at same potential and B
and A is maximum. and D are at same potential,
Hence, in this case also, V should be applied A
8W
B
to faces with dimensions 2d ¥ 3 d
V (2d ¥ 3 d ) 6Vd
and imax = = .
r ( d) r 24V 4W 6W
L
15. (a) R = r C
A D 12W
RA
r=
L Hence, the circuit can be redrawn as
d
[r = = 1.25 mm = 1.25 ¥ 10-3 m] A,C
2
0.104 ¥ 3.14 ¥ (1.25 ¥ 10-3 )2
=
12W
14.0
8W
24V 4W 6W

= 3.64 ¥ 10-7 W - m
V EL 1.28 ¥ 14
(b) i = = = = 172.3 A
R R 0.104 B,D
(c) i = neAvd 1 1 1 1 1
i \ = + + +
vd = R 4 8 12 6
neA 6+3 +2+4
172.3 =
= 24
8.5 ¥ 1028 ¥ 1.6 ¥ 10-19 ¥ 3.14 ¥ (1.25 ¥ 10-3 )2 15 5
= =
= 2.58 ¥ 10-3 ms -1 24 8
16. For zero thermal coefficient of resistance, 8
R= W
DR = 0 5
RC a CDT + RFe a Fe DT = 0 = 1.6 W
R1 - a Fe - 5.0 ¥ 10-3 V 24
= = = 10 i= =
R2 aC - 0.5 ¥ 10-3 R 1.6
= 15 A
R1 = 10 R2
19. Given circuit is similar to that in previous
Also, R1 + R2 = 20
question but 4 W resistor is removed. So the
fi 10 R2 + R2 = 20 effective circuit is given by
8
A,C V 12
i= =
R 36 / 13
13
= A
3

12W
24 V 8W 6W
12 + 6
21. (a) i = =3 A
1+2+3
B,D i = 3A
A
1 1 1 1 12 V 1W
= + +
R 8 12 6
B
1 3+2+4 9 3
= = = G 2W
R 24 24 8
8 C
R = W = 2.67 W
3 6V 3W
V 24
i= = =9 A
R 2.67 D
20. VG = 0
6W
A B VA = VG + 12 = 12 V
4W VA - VB = 3 V
12V 12W 6W VB = 12 - 3 = 9 V
3W VB - VC = 6 V
D C
2W VC = 9 - 6 = 3 V
flA VG - VD = 6 V, VD = - 6 V
6W 4W (b) If 6 V battery is reversed
4W 12 - 6
12 B C 12W i= =1A
1+2+3
3W 2W
D i = 1A
A

Wheatstone bridge is balanced, hence 4 W 12 V


resistance connected between B and C be
B
removed and the effective circuit becomes
G
A
6W 4W C
12V B C 12W 6V
3W 2W
D
D

A VG = 0,
VA - vG = 12 V, VA = 12 V
12V 9W 6W 12W
VA - VB = 1 V
fi VB = 11 V
fl VB - VC = 2 V
fi VC = 9 V
36
12V W
13 VD - VG = 6 V
fi VD = 6 V
9
200 = 42.26 W
22. i = =5 A
5 + 10 + 25 E
i= = 0.102 W
1 Reff
5W Reading of voltmeter
200 V
2 V = E - ir
10W = 4.3 - 0.102 ¥ 1
ª 4.2 W
0
Reading of Ammeter,
25W V 4.2
i1 = = = 0.08 A
3 R + Ra 42

(i) V3 - V0 = 5 ¥ 25 24. Consider the directions of current as shown


fi V3 = 125 V in figure.
42V 5W 4W
(ii) V0 - V2 = 5 ¥ 10 A
V2 = - 50 V
(iii) V2 - V1 = 5 ¥ 5 I2 10V
I1 6W
V1 = - 75 V
8W
(iv) V3 - 2 = 5 ¥ 35 = 175 V B
(v) V1 - 2 = - 5 ¥ 5 = - 25 V 1W C
(vi) V1 - 3 = - 200 V I3
23. (a) 6W
i E r D
4V

i1 R Applying KVL in loop 1, 2 and 3, we


A respectively get,
i2 S I1 + 6 ( I1 - I2 ) + 5I1 = 42
V fi 12I1 - 6I2 = 42
Reff = R|| Rv + Ra + r fi 2I1 - I2 = 7 …(i)
50 ¥ 200 4I2 + 6 ( I2 - I1 ) + 8 ( I2 + I3 ) = 10
= +2+1
50 + 200 fi 9I2 - 3 I1 + 4I3 = 5 …(ii)
= 43 W 8 ( I2 + I3 ) + 16I3 = 4
E 4.3 2 I2 + 6 I3 = 1 …(iii)
i= = = 0.1 A
Reff 43 On solving, we get,
\Reading of ammeter, i = 0.1 A I1 = 4.7 A, I2 = 2.4 A, I3 = 0.5 A
and reading of voltmeter = i ( R|| Rv ) Resistor 5W 1 W 4W 6W 8W 16 W
= 0.1 ¥ 40 = 4 V Current 4.7 A 4.7 A 2.4 A 2.3 A 2.9 A 0.5 A
(b) i E r
25. R = 400W
V 200W
400W
i1 R
A
100W 100W 200W
i2 fi A 100W 100W D
C
i2 i2 B 200W
V 100W 100W
i1 i1
Reff = ( Ra + R)|| Rv + r i
52 ¥ 200 i 10 V i 10 V
= +1
52 + 200
10
As Wheatstone bridge is balanced, 100 W E
resistance between B and D can be removed,
ie,
A 100W B 200W C R1' R2'

i2
100W 200W R2 RV2 6000
i1 D R2¢ = = = 1200 W
R2 + RV2 5

10 V As R1 ¢ = R2 ¢
10 1 Hence,
\ i1 = i2 =
= A
300 30 reading of V1 = reading of V2
Hence, reading of voltmeter 1200
= ¥ 200 = 100 V
1200 + 1200
= Potential difference between B and C
20 (b) Current distribution is shown in figure
= 200 ¥ i2 = V
3 E
= 6.67 V
26. (a) (i) When S is open. i i2
i1
E V1 V2
i2 S i1
R1 R2
V1 V2
E
R1 S R2 i=
R1 ¢ + R2 ¢
200 1
R1 i= = A
3000 2400 12
V1 = E= ¥ 200
RV1 + RV2 5000 RV1 3000 1
i1 = i= ¥
= 120 V R1 + RV1 5000 12
RV2 2000 1
V2 = E= ¥ 200 = A
RV1 + RV2 5000 20
R1 2000 1
= 80 V i2 = i= ¥
R1 + RV1 5000 12
(ii) When S is closed, 1
E A =
30
\Current flowing through
1 1
V1 V2 S = i1 - i2 = -
20 30
R1 S R2 1
= A
60
Now, R1 and V1 are in parallel and their 27. Effective emf of 2 V and 6 V batteries
effective resistance connected in parallel
R1 RV1 E r + E2r1 2 ¥ 1 - 6 ¥ 1
6000 E¢ = 1 2 =
R1 ¢ = = = 1200 W r1 + r2 1+1
R1 + RV1 5
= -2V
Similarly, rr 1
R2 and V2 are in parallel with their effective and r¢ = 1 2 = W
r1 + r2 2
resistance,
= 0.5 W
11
2V 1W 32. V1 = E - i1r fi E - 1.5 r = 8.4 …(i)
4V 0.5W V2 = E + i2r fi E + 3.5 r = 9.4 …(ii)
On solving, we get
6V 1W r = 0.2 W
fl E = 8.7 V
0.5W 0.5W
33. In case of charging
4V 2V V = E + i r = 2 + 5 ¥ 0.1 = 2.5 V
Net emf, E = 4 - 2 = 2 V 34. Clearly current through each branch is zero.
2W
28. (a)
A B
2W 4W 8W 8W fi 4W 4W 4W
– – 2V
E1 E2 2V 2V 2V
+ +
2V 2V
E
35. i1 =
As E1 > E2 R+G
Current will flow from B to A. R
G
(b) E1 is doing positive work
i1
(c) As current flows from B to A through
resistor, B is at higher potential.
E
29. i2 R = 2 W < 5 W
Clearly X is doing negative work. On shunting the galvanometer with
resistance S,
A i
S
R=2.0W E B R R'
G
P 0.5
(a) P = Vi fi V = = = 5.0 V i2
i 1.0
(b) E = V - iR = 5 - 2 = 3 V E

(c) It is clear from figure that positive E


i2 =
terminal of X is towards left. GS
R + R¢ +
150 - 50 S+G
30. i = = 20 A
3+2 As i1 = i2
3.0W E E
Q =
R + G R + R¢ + GS
G+S
– 150 V – GS
+ 50 V + fi R + R¢ + =R+G
i
G+S
2.0W
G2
P R¢ =
G+S
VP - VQ = 50 + 3.0 i
36.
VQ = 100 - (50 + 60)
I A I1 r
= - 10 V A B
I2
31. (a) As voltmeter is ideal, it has infinite V
resistance, therefore current is zero.
r V
(b) V = E - ir fi E = 5.0 V I2 = I=
R+ r R
(c) Reading of voltmeter fi V = 5.0 V
12
r V ig
fi = 39. S = (G + R)
R + r IR i - ig
R IR - V 5 ¥ 2500 - 100 S
fi = =
r V 100
100
r= ¥ 2500 = 20.16 W R
12400 A G B
37. i - ig
60 V R= S -G
ig
20 - 10-3
300W 400W = ¥ 0.005 - 20
10-3
V = 79.995 W
L1 - L2 0.52 - 0.4
Let R be the resistance of voltmeter 40. r = R= ¥ 5 = 1.5 W
L2 0.4
As reading of voltmeter is 30 V,
1 1 1 41. Let R be the resistance of voltmeter
+ = fi R = 1200 W V
R 400 300
If voltmeter is connected across 300 W A
100W
resistor, i
3W
60 V
3.4 V

300W 100 R
400W Re = 3 + 2 +
100 + R
V 100 R
=5 +
100 + R
Effective resistance of 300 W resistor and
voltmeter 3.4
i= = 0.04
300 ¥ 1200 100 R
R¢ = = 240 W 5+
300 + 1200 100 + R
60 4R
i= fi 0.2 + = 3.4
400 + 240 100 + R
60 fi R = 400 W
= A
640 Reading of voltmeter,
3 100 R 100 ¥ 400
= A V =i¥ = 0.04 ¥
32 100 + R 100 + 400
\Reading of voltmeter,
3 = 3.2 V
V = iR ¢ = ¥ 240 If the voltmeter had been ideal,
32
= 22.5 V Reading of voltmeter
R¢ 100
38. V2 = V, = ¥ 3.4 = 3.24 V
R1 + R2 ¢ 105
L R
rR2 120 42. 1 = 1
R2 ¢ = = L2 R2
r + R2 3
L1 8
= 40 W fi = (L1 + L2 = 40 cm)
40 40 - L1 12
V2 = 120
60 + 40 fi L1 = 16 cm from A.
= 48 V
13
ig Maximum power dissipated by the circuit
43. S = (G + R)
i - ig 2
P ¢max = Imax Re
i - ig 3
fi R= S -G = 15 ¥ ¥ 2.4 = 54 W
ig 2
20 - 0.0224 47. Total power of the circuit, P = P1 + P2 + P3
= ¥ 0.0250 - 9.36
0.0244 = 40 + 60 + 75
= 12.94 W = 175 W
E V2 V2
44. (a) i = As P = fi R=
RV + r R P
Rv (120)2
= = 82.3 W
V 175
i 48. Thermal power generated in the battery
r
R
E
E i
V = iRV = RV r
RV + r
r 1 E
(b) = 1% =
RV + r 100 P1 = i2r = i ( E - V )
RV = 99r = 99 ¥ 0.45 = 0.6 W
= 44.55 W Power development in the battery by electric
V RV forces
(c) =
E RV + r P2 = IE = 2.6 W
As RV decreases, V decreases, decreasing 49. The given circuit can be considered as the
accuracy of voltmeter. sum of the circuit as shown.
45. (a) When ammeter is connected 3
2W 2W A
E 21/6A 2W 16
IA = 35 5/16A
A 14/6A 2/16A
RA + R + r 16
3W + 3W
When ammeter is removed 7V 1V
E R + R+ r
I= = A IA fl
R+ r R+ r 2W 2W
I 2A
(b) A = 99% 1A
I 3W 1V
R+ r 99 7V
=
RA + R + r 100
1 1 \ P1 = 7 ¥ 2 = 14 W,
fi RA = ( R + r) = (3.8 + 0.45)
99 99 P2 = - 1 ¥ 1 = 1 W
RA = 0.043 W E1 - E2 12 - 6
50. (a) i = = = 0.5 A
IA R+ r R1 + R2 4 + 8
(c) As = , as RA increases, I A
I RA + R + r (b) Power dissipated in R1 = I 2 R1 = 1 W
decreases, decreasing the accuracy of and power dissipated in R = I 2 R2 = 2 W
ammeter. (c) Power of battery E1 = E1I
r max 36
46. Imax = = = 15 A = 12 ¥ 0.5 = 6 W (supplied)
R 2.4
Power of battery E2 = E2I
For the given circuit
= - 6 ¥ 0.6 = - 3 W (absorbed)
1 3
Re = R + R = R
2 2
14
E 12 1 1 1 1 1
51. I = = =2A 54. (a) = + + +
R+ r 5+1 R 4 6 14 4
4W 4W
(a) P = EI = 12 ¥ 2 = 24 W
4W
(b) P1 = I 2 R = 22 ¥ 5 = 20 W 8W 4W
2W

6W fiA 8W 6W B
(c) P2 = I 2r = 22 ¥ 1 = 4 W
2W
A B
52. (a) 4W 4W
1.60W fl
I1 4W
I2 2.40W
6W

I3 4.80W A 14W B
I

4W
1 31
28.0V =
R 42
1 1 1 1 42
= + + R=
R R1 R2 R3 31
1 1 1 1 1 1 1 1
= + + (b) = + +
R 1.60 2.40 4.80 Re 2 R 2 R R
R = 0.80 W
V 28.0 R R
(b) I1 = = = 17.5 A R
R

R
R1 1.60
R R
V 28.0 A B A B
I2 = = = 11.67 A R R
R2 2.40
R R
V 28.0
I3 = = = 5.83 A fl
R3 4.80 2R
(c) I = I1 + I2 + I3 = 35.0 A 2R
(d) As all the resistance connected in A B
parallel, voltage across each resistor is
28.0 V. R
V 2 (28)2
(e) P1 = = = 490 W Wheatstone bridge is balanced
R1 1.6 R
V 2 (28)2 fi Re =
P2 = = = 326.7 W 2
R2 2.4 i2+ i3 4W
V 2 (28)2
P3 = = = 163.3 W (c) 2W i3 i3 2W
R3 4.8
i2 3W
V2 i1+ i2 3W i1 i + i
1 2
(f) As, P =
R A 1W i1 B
i1 1W
Resistor with least resistance will dissipate fl
maximum power. 4W
V2
53. (a) P = fi V = PR 2W
R 3W 3W 2W
= 5 ¥ 15 ¥ 103 = 2.74 ¥ 102
A B
= 274 V 1W 1W
2
V (120)2
(b) P = = = 1.6 W
R 9 ¥ 103
15
fl 2W 8W C
2.4W 4W 2W 8W
(e) A 2W B fi A
2W 6W 2W fi 2W 6W 2W 2W B
A B A B
4W 10W 4W 10W
2W 2W D

6.4 8.5R C
1
1.52W W 8W
fi A B 1W 2
11W fiA B
A B A B O
2W 1W 10W
fl D
i3– i4 10W 5.8W
(d) i4 A B
5W i3
i3 5W
i4 By Star-Delta Method
10W 5W 10W R2 R3
RA =
5W

i1 + i2 + i3
i2 i2
i1 + i2 + i3 R1 + R2 + R3
A B R1 R3
i1 10W RB =
R1 + R2 + R3
10W fl 10W R2 R3
5W 5W RC =
R1 + R2 + R3
10W
10W 10W fi 10W 10W R R R R
5W 10W
5W
R R R R
R
A B A B R R R R
10W 10W fi
(f)
fl R R
R R R R R
5W R R
A B
25W A B
fl fl
10W 10W R 2R
A B
5W
fl R R 2R
A B fi
5W R R
4.17W
A B A B
2R
A B
fl 2R
3R 6
W
fi 5
A B
A B
2R

As circuit is symmetrical about


perpendicular bisector of AB, lying on it are
at same potential.
16
A B D F B R D F
2i1+i2
R R
fi R
R
(g) i1 1W i1
A C E A C R E
B
i1 i1 2W fl
2W 2W R2 = 3R
2W

1W 1W R R R
E 2W F
C D
2W
R2 3 R
fl \ = = 0.6
R1 5 R
A
A fi R2 = 0.6 R1
B A D
1W 2W 15W 15W
8W
56. F
2W fi 5W 1W 5W
B 20W 6W
2W
2W 30W 40W
2W E


1W 1W B 15W 15W

0.71W A 2W 6W 40W 8W
A B B
Clearly C and D, E and F are at same
potential. C,E 20W 30W D,F
B D fl
F
30W
55. 2W 46W 8W
A B
A C E
50W
Let R be the resistance of each conductor,
and R1 be the effective resistance between A Here, C and E , D and F are at same
and F in first case then, potential.
\ R1 = 5 R Re = 23.3 W
If R2 be effective resistance between A and F r
2
in second case then, 2r r
r 3
57. (a) r r
r r
r
a b b
r a b a
r r

5 5
r r
8 3
A B
r
17
b fl
r
r
(b) r r 2r r r
fi r
o a b a
r r a fi
b a b
fl r
4 2r r
r r/ 3
3 r fl
fi B r
b a
r r
(c) r r As Wheatstone bridge is balanced.
r
r
58. Re =
b a fi b a 2
r i1–i3
r r
r r (b)
i3
i3 i1
fl 2i1+i2 i1 i2
2r a i b
i1 2 i3 i1
r i3
b a fib a
i1–i3
2r

As Wheatstone bridge is balanced r
b r r
r r
r r r r r
r r
r r 4 r 2r
(d) fi r fi fi
a r b a 2r b
r r r r r
a r r r

r 2r/3
8r/3
r r r r
r
(e) a fi a 2r b
r r fi r
2r b r r
a b
a b
r 2r/ 3
r r 8r/ 3
r fl 4/5r
a b

Objective Questions (Level-1)


1. When ammeter is connected in series mvd
fi E= = not constant
et
Re = R + RA
V
Hence, net current decreases. So RA should 3. R =
I
be very low.
[V] [ML2T -3I-1 ]
2. Amount of charge entering per second from fi [R] = =
[I] [I]
one face is equal to the amount of charge
leaving per second at the other, hence I is = [ ML2T -3I-2 ]
constant. 1
4. s =
Again, r
I
vd = = not constant. As unit of resistivity is ohm-m and
neA
eF unit of s is ohm -1- m -1.
As vd = t
m 5. Fact.
18
6. E = I ( R + r ) I1 + I2 + I3 = 0
Case I 6 - V0 3 - V0 2 - V0
+ + =0
E = 0.5 (3.75 + r ) 6 3 2
Case II fi 6 - V0 + 2 (3 - V0 ) + 3(2 - V0 ) = 0
E = 0.4 (4.75 + r )
On solving V0 = 3 V
I I
r = 0.25 W, E = 2 V 13. vd = =
I 50 5 neA ne pr 2
7. = fi I = Ig
I g 20 2 2I v v
vd ¢ = = d =
Ig I - Ig nep (2r )2 2 2
S= GfiG = S
I - Ig Ig 14. Voltmeter has higher resistance than
3 ammeter.
= ¥ 12
2 Again higher the range of voltmeter, higher
will be its resistance.
= 18 W
1
1
8. I g = 2% I = I R2
50 15. I2 = I
Ig 1 1 1
G + +
S= G= R1 R2 R3
I - Ig 49
R1
V2
9. P = I1
R
2 R2=20W
V I
P + DP = A
R + DR I2
I3
R3=15W
As R µ l
DR = - 10% R
V2 V2 Ê 1 ˆ 1 I Ê 1 1 ˆ
DP = - =Á - 1˜ P fi = - ÁÁ + ˜
0.9 R R Ë 0.9 ¯ R1 I2 R2 Ë R2 R3 ˜¯
10 0.8 Ê1 1ˆ
= P ª 11% P = -Á + ˜
9 0.3 ¥ 20 Ë 20 15 ¯
10. Potential difference between any two points 1
=
is zero. 60
l -l R1 = 60 W
11. r= 1 2 R
l2 E
75 - 60 16. (d) I1 = , V1 = I1 RV
= ¥ 10 RA + RV
60
A V
= 2.5 W
12. (b) By applying KCL at O I1
A I1 B
I2
E
6W 3W
O V0 = E - I1 RA
If resistance is connected in parallel with
2W
voltmeter,
I3

C
19
R V2
Pe = = n 2P
Re
A V
19. As bulb A is in series with entire circuit.
I2
E1 + E2 18
E 20. I = =
R + r1 + r2 R + 3
E
I2 = > I1 Vab = E2 - Ir2 = 0
RRV
RA + 18
R + RV 3- ¥1 = 0
R+3
and V2 = E - I2 RA < V1
fi R=3 W
17. Before connectivity resistance is parallel R
with ammeter 21. I2 = I
R + RV
A V
I I1 R
I1 A
I2
E V

E V R
I1 = , V1 = I1 RV = I
RA + RV RV R + RV
100 R
= E - I1 RA = 5
2500 R + 2500
After connecting resistance in parallel to the
ammeter. R + 2500 = 125 R
RA 2500
R= W ª 100 W
24

A V
22.
R/10 R/10
I2
R/10 R/10

E R/10 R/10
E R/10 R/10
I2 = ,
RA
+ RV R/10 R/10
2
1
Reading of ammeter = I2
2 fl
E 1 R/5
= > I1
RA + 2 RV 2
R/5
2E
V = I2 RV = < 2V1 R/ 25
RA + 2 RV R/5

R R/5
18. Re =
n2
R/n R/5

R/n
R1 20 1
23. = = …(i)
R/n R2 80 4
R1 + 15 40 2
2
= =
V R2 60 3
P=
R
20
R1 15 2 VAB = - I (25 + 15)
+ =
R2 R2 3 1
= - ¥ 40 ª - 4 V
15 2 1 5 9
= - =
R2 3 4 12 30. (a) VAB = kL = 0.2 ¥ 100 = 20 mV
fi R2 = 36 W, RAB
VAB = E
R RAB + R
R1 = 2 = 9W
4 RAB
fi 0.02 = ¥2
V RAB + 490
24. (b) As V1 = , R1 = R2
2
fi RAB + 490 = 100 RAB
RV ¥ 100
= 50 490
100 + RV RAB = ª 4.9 W
99
RV = 100 W 31. (c) When key is open,
E 2 2E
25. (d) = = 0.4 W I1 =
RPB + r 4+1 3R
VAB = IRAB = 1.6 W R 2R 3R
VPB 1.6
K = = = 0.016 V/cm
L 100
E 1.2 2R R fi 3R
L= 1 = = 75 cm I1 I1
K 0.016
A A
26. (d) VAB = 3 ¥ 2 + 3 + 1 ¥ 4 - 2 + 6 ¥ 1
E E
= 17 V 3/2R fl
1A 2A
3R
2W 2W I1
3A 2W 2W 1W 1W
A
B
A E
3V 2V
E1r2 + E2r1 When key is closed
27. (c) Ee = =2V R 2R
r1 + r2 2R/3 2R/3
rr
re = 1 2 = 0.5 W
r1 + r2
2R R fi I2
For maximum power R = re I2
Ee2 (2)2 A A
and Pmax = = =2W E E
4re 4 ¥ 0.5 fl
R 4R/3
28. (a) V = E
R+ r
ÊE ˆ Ê 2.2 ˆ
r = Á - 1˜ R = Á - 1˜5 I2
ËV ¯ Ë 1.8 ¯
A
10
= W
9 3E
I2 =
E1 - E2 4R
29. (d) I =
R1 + R2 + r1 + r2 I1 8
\ =
10 - 5 1 I2 9
= = A
25 + 15 + 2.5 + 2.5 9
21
1
I Ig a
32. (b) S = G = 34 ¥ 3663
I - Ig 33 R2 60° R1
I b
34
= 111 W
r Hence R1 and R2 are in parallel
2r R1 R2
r r r r Re =
r R1 + R2
r r
33. (b) A B fi A
B
r = 2.5W
r r
r r 36. (c) Let RAD = RBC = x
r 2r
1W 1W 1W 1W 1W

5/3r fl
2r/3 1W 1W 1W 1W
r r
A B fiA r
B
1W 1W
A B 1W 1W
r
5/3r 2r/3 Clearly x < 1 as 1 W resistor is in parallel
fl with some combination.
5/11r
Now RAB = x + 1 + x
= 2x + 1
5
Re = r As x < 1
11
11 1 < RAB < 3
r= ¥ 1.5
5 R ( R + R0 )
37. (d) RAB = R + = R0
= 3.3 W 2 R + R0
r r r r fi 2 R2 + RR0 + R2 + RR0 = 2 RR0 + R02
r fi 3 R2 = R02
34. (b) r r r r R
A r r B fiA r r B fi R= 0
3
r
38.
r r r r
P P
fl R
R R R/2 R
2r R
2r fi
r/2
A B fiA 2r B R R/3
R
Q Q
2r
P fl
As the circuit is symmetrical about
perpendicular bisector of AB, all points 5 5
lying on it are at same potential. 6 R = 25W ‹ 11 R R
L1
35. (c) R1 = R
L1 + L2 Q
R
fi R1 = =3 W
6
l2
R2 = fi R2 = 15 W
l1 + l2
22
39. Wheatstone bridge is balanced. 4
R1 = ¥ pr = 2 = R2
A P 2pr
R 4 4
R R R R R3 = ¥2 r =
R 2pr p
R R
‹ R
R R 1 1 1 1
2R/3 2R/3 R \ = + +
R R Re R1 R2 R3
B
R R 1 1 p 4+ p
2R/3 2R/3 = + + =
B 2 2 4 4
flA 4
Re = W
R R R R 4+ p
7R/3 7R/3
fi fi 42. (d) Points C and D are shorted hence the
4R/3 4R/3 4R/3 4R/3 portion above line CD can be removed.
B
7
fi Re =R
6
C D
L1 1
40. (d) R1 = R= R
L1 + L2 12
=3 W A B

A C D

30°
B
O A B

R/2 R/2
R/2 R
L2 11 A B ‹ ‹
R2 = = R = 33 W A B
L1 + L2 12 R A B
R
R1 and R2 are in parallel,
R1 R2 3 ¥ 33 43. (b) As AB is line of symmetry,
Re = = we can fold the network about AB.
R1 + R2 3 + 33
A R R
= 2.75 W
41. (a) Resistance per unit length of wire R R
R
4
= R R
2 pr R
A R R B

R/2 R/2 R
A R/2 R/2
R/2 R/2 A
fi B
R1 R3 R2 R/2
R/2 R

B 3R/2
A B

B
23

JEE Corner
Assertion and Reason
1. (d) V = IR, If V = 0 either I = 0 or R = 0 E2 R
P= is maximum at R = r.
2. (b) As all the resistors are in parallel ( R + r )2
potential difference is same, hence V V2
V2 8. (c) I = ,P= both I and P are inversly
P= is maximum if R is minimum. R R
R proportional to R hence both decrease with
I 2t r increase in R which increases with
3. (b) dH = I 2dRt = dH temperature.
A
I is same everywhere, hence portion having According to Ohm’s law V µ I not V = IR.
less area is more heated. As R can be variable also.
I 9. (d) Drift velocity is average velocity of all the
Again J =
A electrons but velocity of all electrons is not
\ J A > J B. constant.
Reason is also correct but does not explain L
10. (a) R = r
assertion. A
m
4. (b) Both assertion and reason are correct but r= 2
reason does not explain the cause of decrease ne t
in voltmeter reading. with increase in temperature, electron
5. (b) As RA < RV , more current passes through collide more frequently, i.e., t decreases,
increasing r and hence R.
ammeter when positions of ammeter and
E r + E2r1
voltmeter are interchanged and potential 11. (d) E = 1 2 \ E1 < E < E2
difference across voltmeter becomes less that r1 + r2
emf of cell. If E1 < E2
6. (c) During charging current inside the rr
r = 1 2 , r < r1, r < r2
battery flows from positive terminal to r1 + r2
negative terminal. Reason is false while R1 L1
assertion is true. 12. (d) =
R2 L2
E
7. (d) I = is maximum when R is zero Hence there is no effect of one while
R+ r
measuring using meter bridge.
hence reason is false.

Objective Questions (Level-2)


E - E1 1.5 - 1.3 2. (c) Let R = Resistance of voltmeter,
1. (b) I = 2 =
r1 + r2 r1 + r2 ER
V1 = = 198 V …(i)
0.2 R1 + R
= …(i)
r1 + r2 ER ER
V2 = = = 180 V …(ii)
V = E1 + Ir1 R2 + R 2 R1 + R
0.2 2 R1 + R 198 11
fi 1.45 = 1.3 + r1
r1 + r2 = =
R1 + R 180 10
r1 0.15
= fi 0.2r1 = 0.15r1 + 0.15r2 20 R1 + 10 R = 11 R1 + 11 R
r1 + r2 0.2
9 R1 = R
0.05r1 = 0.15 r2 r1 = 3 r2
24
From Eq. (i), 35
\Required ratio = =1
ER 35
= 198 L - L2 x-y
R1 + R 9. (a) r = 1 R= R
10 R1 L2 y
E = 198 ¥ = 220 V
9 R1 R - 20 t - 10
10. (d) =
40 - 20 30 - 20
3. (b) P = I 2 R
As R is same for all bulbs and maximum fi R = t + 10
current passes through bulb A, it will glow E 10
I= =
most brightly. R t + 10
V dq 10
4. (c) R + RA = =5W =
I dt t + 10
R = 5 - RA < 5 W 30 10
dt = 10[log e( t + 10)]
30
L1 - L2 10 q=Ú
10 t + 10 10
5. (a) r = R= ¥ 132.40
L2 60
= 10 log e 2
ª 22.1 W
11. (b) Let l1 length is kept fixed and l2 is
6. (b) Current through R when S is open. stretched,
E1 + E2 l l
I1 = R1 = r 1 , R2 = r 2
R + r1 + r2 A A
Current throughR when S is closed Initial resistance,
E1 R = R1 + R2 …(i)
I2 =
R + r1 3
Now full is stretched times, ie,
2
DI = I2 - I1
3
E1 E1 + E2 l2 ¢ = ( l1 + l2 ) - l1
= - 2
R + r1 R + r1 + r2
1
E1r2 - E2( R + r1 ) = ( l1 + 3 l2 )
= 2
( R + r1 )( R + r1 + r2 ) A ¢l ¢ 2 Al2
A2 ¢ = 2 2 =
DI = + ve if E1r2 > E2( R + r1 ) l2 l1 + 3 l2
7. (a) VA = IR 1
( l1 + 3 l2 )2
2 R2 ¢ = r 2
I
3 B 2 Al2
I
x A y ( l1 + 3 l2 )2
R2 ¢ = r
1 C 4 Al2
I
3
Now, R¢ = 4 R
2
VB = I ¥ 1.5 R = IR R1 + R2 ¢ = 4 ( R1 + R)
3 ( l + 3 l2 )2
1 l1 + 1 = 4( l1 + l2 )
VC = I ¥ 3 R = IR 4l2
3
l2 1
\ VA = VB = VC fi =
l1 7
8. (d) Current through 15 W resistor
l2 1
30 fi =
= =2A l1 + L2 8
15
X l1
VBC = (2 + 5) ¥ 5 = 35 V 12. (b) = fi l1 = 40 cm
R 100 - l1
Voltage drop across R = 100 - (30 + 55)
= 35 V If R¢ = 8 W
25
X l1 ¢ when, k1 is open and k2 is closed, net
=
R¢ 100 - l1 ¢ resistance is
100 ¥ 100
fi l1 ¢ = 60 cm Rnet = r + R1 + = ( r + R1 + 50)
l1 ¢ - l1 = 20 cm 100 + 100
0.1 I0 E
13. (c) I1 = I \ = …(ii)
0.1 + 9.9 2 r + R1 + 50
9W 10 mA The above two equations are satisfied if r = 0
A and R1 = 50 W.
I1
I2 16. (b) 20 W, 100 W and 25 W resistors are in
parallel.
0.1W 0.9W 4W
I
+
10W
A B C 4W 20W 100W 25W
V fi
But I1 = 10 mA
10 6W 6W
I= ¥ 10 mA = 1000 mA –
0.1
4A
= 1 kA
R = 20 W
14. (d) Effective emf of two cells
E r + E2r1 2 ¥ 6 + 4 ¥ 2 V = IR = 80 V
E= 12 =
r1 + r2 2+6 17. (a) Hence, points A and C, B and D are at
20 same potential.
= = 2.5 V
8 V
RAB = 4 ¥ 4 = 16 W
R B R R
RAB 16 A D
VAB = E0 = ¥ 12 C
R + RAB 4 + 16 E, r
= 9.6 V A
V
k = AB = 2.4 V/m The equivalent circuit is given by
L
Now, E = kl V
E 2.5 25
fi L= = = A, C R B, D
k 2.4 24 R
15. When k1 and k2 both are closed, the R
resistance R1 is short circuited. Therefore
net resistance is
k1 A
I
E, r

V
R/3
A

100 ¥ 100 A
Rnet = r + = r + 50
100 + 100 E, r
E
\ I0 = …(i)
r + 50
26
E 21. (a) Voltage sensitivity of voltmeter
I= =1A
R 1
+r µ
3 Resistance of voltmeter
R
V = I ¥ =3 V Vs1 R +G
3 \ = 2
Ig Vs2 R1 + G
r
18. (c) S = G , G = r, S =
I - Ig 4 30 R2 + 50
=
1 1 20 2950 + 50
Ig = (I - Ig ) fi Ig = I
4 5 30 ¥ 3000
R2 + G = = 4500
= 0.006 A 20
10 5 fi R2 = 4450 W
19. (d) I1 = = A
14 7 22. (b) For x = 0
8W B 6W
I1 VAB = E
P E
4W 3W k1 =
L
I I2 A EL1
E0 = k1L1 = …(i)
L
10 V
For x = x (say)
10 RAB
I2 = A VAB = E
7 RAB + x
VP - VA = I 2 ¥ 4 RABE
40 k2 =
= A ( RAB + x ) L
7
40 RABEL2
VP - VB = I1 ¥ 8 = A E0 = k2L2 = …(ii)
7 ( RAB + x ) L
VA - A B = 0 From Eqs. (i) and (ii),
Another method R + L2
R R L1 = AB
As, 1 = 3 , VB = VA ( RAB + x )
R2 R4
10 ¥ 30
fi 20 =
20. (b) For series connection 10 + x
V1 R
= 1 fi x =5W
V2 R2
R1 3 23. (d) To obtain null point similar terminal of
\ = both the batteries should be connected.
R2 2
L rL 24. (c) Wheatstone bridge is balanced.
Now, R1 = r 1 = 12 , I1 20W 4W
A1 pr1 20W 4W
L rL 1.4A 1.4A
R2 = d 2 = 22 15W fi
A2 pr2 I2
2
R1 L1 Ê r2 ˆ 50W 10W 50W 10W
\ = ¥Á ˜
R2 L2 ÁË r1 ˜¯ R2
\ I1 = ¥I
r1 R2L1 2 ¥6 R1 + R2
= = =2 60
r2 R1L2 3 ¥1 = ¥ 1.4
r2 1 84
= =1A
r1 2
27
25. (b) L1 - L2
29. (c) r= R
E,B,H L2
R R L2r
fi R=
A B R L1 - L2
R 490 ¥ 10
D
C E fi R = = 490 W
10V A,C,F 10
F H R More than One Correct Options
50W G 10W D G
10V 30. H = P1t1 = P2t2
fl H H
fi t1 = , t2 =
P1 P2
I1 R R
I2
If connected in series
1 1 1
R = +
I3 R R P P1 P2
I ‹
fi t = t1 + t2
If connected in parallel
10V 10V
P = P1 + P2
10 tt
I1 = = 2.5 A fi t= 12
2 ¥2 t1 + t2
26. (b) Effective resistance of voltmeter and E1r2 + E2r1 6 ¥ 3 + 5 ¥ 2
31. E = =
3 kW resistor, r1 + r2 2+3
3 ¥6
R1 = = 2 kW = 5.6 V
2+6
As there is no load.
R1 2
V1 = E = ¥ 10 = 5 V V = E = 5.6 V
R1 + R2 4
If E1 = E2, I = 0
27. (d) P1 = P2 = P3, Clearly R2 = R3 E - E2 6 - 5
R2 I= 1 = = 0.2 A
r1 + r2 2+3
R1
32. Let V = Potential difference between T1 and
i
T2.
R3
I1 A B
i T1
\ i2 = i3 = T2
2
2 I2
i 1
P1 = i2 R1, P2 = ÊÁ ˆ˜ ¥ R2 = i2 R2
C
Ë2¯ 4 V
1 2 I1 =
P3 = i R3 RA + RB
4
V
R2 = 4 R1, R3 = 4 R1 I2 =
RC
\ R1 : R2 : R3 = 1 : 4 : 4
E 2 Now,. I A = I B = I1
28. As E = kL1 fi k = = = 250 IC = I2
L1 500
1 Also, V = IC RC = I1( RA + RB )
= V/cm
250 = I A RA + ID RB
1 I B I1 RC
V = kL2 = ¥ 490 cm = =
250 IC I2 RA + RB
= 1.96 V
28
33. As R1 π R2 E
Ie = >I
R
V1 V2 +r
2
A B If S2 is closed
E
V3 Ie = >I
r
R+
2
V1 π V2
38. Vb - Va = - 10 + 2I = 2 V
Also, V3 = V1 + V2
2W I 10 V
34. As R1 = R2 a C b
I R1 R2
I =6A
V1 = V2 From b to a.
L Vc - Va = 2 ¥ 6 = 12 V
R=r
A 39.
a a a
But
r r r r r r
L2 = 2L1
and R1 = R2 b d b d b d

\ A2 = 2 A1 r r r r r r
1 c c c
Also, vd µ (For constant current)
A fl fl fl
a
1 a
vd 2 = vd1 fi vd1 = 2 vd 2 2r
2 r r r r
b r d
Again, vd µ E b d b d
\ E1 = 2 E2 r r 2r
2r
35. If E > 18 V current will flow from B to A and fl
c
vice-versa. fl fl r/ 2
36. V = kl a b d
a
If Jockey is shifted towards right, I and r r
hence k will decreases as k µ I.
2r 2r
Hence L will increase. b c
2r
If E1 is increased, k will increase, hence L 3
will decrease. fl c
r fl
If E2 is increased L will increase as V will
increase.
b a
If r is closed V will decrease hence L will 5r r
decrease. 3
E E fl
37. Ie = , Initially, I = 5r
Re + re R+ r
8
b a
If S1 is closed
29

Match the Columns


1. By applying KCL at e
S R3
i1 + i2 + i3 + i4 = 0
R1 R2
b
V1 V2
2W
1W e
i3
E
a c
i1 i4 1W
[V] [ML2T -3A -1 ]
4. [R] = =
2W [I] [A]

d = [ML2T -3A -2 ]
[W ] [ML2T -2 ]
2 - Ve 4 - Ve 6 - Ve 4 - Ve [V] = =
+ + + =0 [ q] [AT]
1 2 1 2
Ve = 4 V, I1 = - 2 A, i2 = 0 , i3 = 2 A, i4 = 0 = [ML2T -3A -1 ]
2 -3 -2 2
[ R][ A ] [ML T A ][L ]
2. Current is same at every point and A1 < A2 [ s] = =
i [L] [L]
J = fi J1 > J 2
A = [ML3T -3A -2 ]
i 1
vd = fi vd1 > vd 2 [ s ]= = [M-1L-3T 3A 2 ]
neA [r ]
R r EA - EB
r= = fi r1 > r2 5. I = =1A
L A R + rA + rA
V
k = fi k1 > k2
L
4V 1W 1W
3. When switch S is closed 1V
V1 decreases, V2 increases,
\ Current through R1 decreases and through
R2 increases.
1W

VA = EA - IrA = 3 V
VB = EB + IrB = 2 V
PA = IVA = 3 W
PB = IVB = 2 W
21 Electrostatics
Introductory Exercise 21.1
1. No, because charged body can attract an 4. No. of electrons in 3 g mole of hydrogen atom
uncharged by inducing charge on it. = 3 ¥ 6.022 ¥ 1023
2. Yes. \ q = ne = 3 ¥ 6.022 ¥ 1023 ¥ 1.6 ¥ 10-19
3. On clearing, a phonograph record becomes = 2.9 ¥ 105 C
charged by friction.

Introductory Exercise 21.2


1 q1q2 1 e2 3. Let us find net force on charge at A.
1. Fe = ◊ 2 = ◊
4pe0 r 4pe0 r 2 A
q
Gm1m2
Fg =
r2
Fe e2
=
Fg 4pe0 ◊ Gm1m2
q
9 ¥ 109 ¥ (1.6 ¥ 10-19 )2 B C
= q
6.67 ¥ 10-11 ¥ 9.11 ¥ 10-31 ¥ 1.67 ¥ 10-27
1 q2 1 q2
FAB = ◊ 2 FAC = ◊
= 2.27 ¥ 1039 4pe0 a 4pe0 a 2
1 q1q2 Net force on charge at A
2. F= ◊
4pe0 r 2 FA = FAB cos 30∞ + FAC cos 30∞
qq 3 q2
e0 = 1 2 2 =
4p Fr 4pe0 ◊ a 2
[ qe ]2
[ e0 ] = FAB cos 30°
[ F ][ r ]2
[IT ]2 FAC FAC cos 30°
= -2 2
FAB
[MLT ][L]
= [ M-1 L-3 T 4 I 2 ]
60°
SI units of e0 = C 2 N-1m -2.
FAC sin 30° FAB sin 30°
A
31
Æ Æ
4. F OA = F OC The cause of attraction is more attractive
Æ Æ
force due to small distance. But if
and F OB = - F OD electrostatic force becomes independent of
distance, attractive force will become equal
A B
q
to repulsive force, hence net force becomes
q
zero.
6. When the charged glass rod is brought near
–q the metal sphere, negative induces on the
O portion of sphere near the charge, hence it
get attracted. But when the sphere touches
q q the rod it becomes positively charged due to
D C conduction and gets repelled by the rod.
Hence, net force on charge at centre is zero. 7. Yes as qmin = e
1 e2
5. No. In case of induction while charge comes Fmin = ◊
closer and like charge moves further from 4pe0 r 2
the source. 8. No. Electrostatic force is independent of
+ + + + +
presence or absence of other charges.
– + Æ Æ ^ ^
– + 9. F21 = - F12 = ( - 4 i + 3 j) N.
– +
– +

Introductory Exercise 21.3


1 q
1. False. E = ◊ A
4pe0 r 2
2. VA > VB as electric lines of force move from
higher potential to lower potential. E
B
q q
3. False. Positively charged particle moves in O
the direction of electric field while negatively
charged particle moves opposite to the q q
direction of electric field. D C

4. False. Direction of motion can be different Hence net field at O is same as produced by
from direction of force. A done but in opposite direction,02 i.e.,
s 1 q
5. E = fi s = Ee0 = 3.0 ¥ 8.85 ¥ 10-12 E= ◊
e0 4pe a 2
= 2.655 ¥ 10-11 C/m 2 8. Net field at the centre (O) of wire is zero. If a
small length of the wire is cut-off, net field
6. q1 and q3 are positively charged as lines of
will be equal to the field
force are directed away from q1 and q3. q2 is
due to cut-off portion, i.e.,
negatively charged because electric field 1 dq
lines are towards q2. dE = ◊ O
4pe0 R2
7. If a charge q is placed at A also net field at q
centre will be zero. dl R
1 2p R
= ◊
4pe0 R2
q dl
=
8p 2e0 R3
32
Æ 1 q Æ
9. E= ◊ 3 r
4pe0 r
9 ¥ 109 ¥ 2 ¥ 10-6 ^ ^ ^ ^
=- (3 i + 4 j) = - 144 (3 i + 4 j) N/C
(3 2 + 42 )3/ 2

Introductory Exercise 21.4


1. Gain in KE = loss of PE 3. Whenever work is done by electric force,
1 1 Ê1 1ˆ potential energy is decreased.
mv2 = ◊ q1q2 Á - ˜
2 4pe0 Ár ˜ W = - DU
Ë 1 r2 ¯
1 U2 = U1 - W = - 8.6 ¥ 10-8 J
¥ 10-4v2 qq
2 4. No. As U = 1 2
1 1 ˆ 4pe0r
= - 1 ¥ 10-6 ¥ 2 ¥ 10-6 ¥ 9 ¥ 109 ÊÁ - ˜
Ë 1 0.5 ¯ If there are three particles
2 1 È q1q2 q2q3 q3q1 ˘
v = 360
U= ◊Í + + ˙
v = 6 10 ms -1 4pe0 ÍÎ r12 r23 r31 ˙˚
2. W = q ( VA - VB ) Here U may be zero.
Ê 1 - 1 ¥ 10-6 -6
1 - 1 ¥ 10 ˆ In case of more than two particles PE of
= 2 ¥ 10-6 Á ◊ - ◊ ˜
systems may same as if they were separated
Á 4pe0 1 4pe0 2 ˜
Ë ¯ by infinite distance but not in case of two
-3
= - 9 ¥ 10 J particles.
= -9 m J

Introductory Exercise 21.5


Wa Æ b a L x dx

4pe0 Ú0 x + d
1. Vba = = 12 ¥ 102 = 1200 V fi ◊
q
a È L L dx ˘

4pe0 ÍÎ Ú0
2. l = a x = ◊ Í dx - d Ú ˙
0x + d
(a) SI Units of l = C/m ˙˚
l a
a= = ◊ [[ x ]L0 - d [ln ( x + d )]L0 ]
x 4pe0
C/ m a È L + d˘
Hence SI unit of a = = C/m 2. = ◊ Í L - d ln ˙
m 4pe0 Î d ˚
3. Consider an elementary portion of length dx
L
at a distance x fro my centre O of the rod.
P P
x
d
d
(b) Consider an elementary portion of rod at
a distance x from origin having length d
dx. Electric potential at P due to this O x d–x
element. 2l
1 l dx Electric potential at P due to this element,
dV = ◊
4pe0 x + d 1 l dx
dV = ◊
4pe0 d 2 + x 2
Net electric potential at P
L 1 l dx l l dx
V =Ú ◊ V= ◊Ú
0 4 pe
0 x + d 4pe0 l-
d2 + x 2
33
l È -1 x ˘ l Charge on the elementary ring;
= ◊ sin
4pe0 ÍÎ d ˙˚ - l dQ = s dA =
Q
◊ 2px dl
q x p RL
= ¥ 2 sin -1
4pe0 ◊ 2l d 2Ql sin q
or dQ = dl
q x RL
V= sin -1
4pe0 l d Potential at O due to this ring
1 dQ
4. Consider the cone to be made up of large dV = ◊
number of elementary rings. 4pe0 l
O Q sin q
= dl
2p e0 RL
q
l Total potential at O
Q sin q L QL sin q
2pe0 RL Ú0
L V= dl =
dl 2pe0 RL
x
Q
= [L sin q = R]
2pe0 L
U = qV
R
Qq
=
Consider one such ring of radius x and 2pe0 L
thickness dl. Let q be the semi-vertical angle
of cone and R be the radius of cone.

Introductory Exercise 21.6


1. (a) V = a ( x 2 - y2 ) V is constant hence E is zero.
Ê ∂v ^ ∂v ^ˆ ^ ^ For x >4
E = - ÁÁ i= j˜˜ = - 2ax i + 2 y j
Ë ∂ x ∂ y ¯ V is decreasing at constant rate, hence E is
(b) V = axy positive.
dV (50 - 100)
Ê ∂v ^ ∂v ^ˆ ^ ^ 3. E = - =- = 10 V/m
E = - ÁÁ i= j˜˜ = - a ( y i + x j) dr 5 -0
Ë ∂ x ∂ y ¯
True.
2. From x = - 2 to x = 0 & x = 2 to x = 4 Æ Æ
V is increasing uniformly. 4. (a) VP - VD = E ◊ l = 0
Æ Æ
(b) VP - VC = E ◊ l = 20 ¥ 1 ¥ cos 0∞
= 20 V

A B
Æ
x 1m E = 20V/m
–2 O 2 4 1m
D C

(c) VB - VD = - 20 ¥ 1 = - 20 V
Hence, E is uniform and negative (d) VC - VD = - 20 ¥ 1 = - 20 V
From x = 0 to x = 2
34

Introductory Exercise 21.7


Æ ^
B+q Æ E = - ( E3 - E1 cos q - E2 cos q) j
E
F1 È 2
-q cos q cos q ˘ ^
= Í 2 - 2 2
- 2 ˙j
x 4pe0
ÍÎ y y +a y + a 2 ˙˚
2l 2l 2q È 1 y ˘^
Q =- Í - ˙j
Q 4pe0 ÍÎ y2 ( y2 + a 2 )3/ 2 ˙˚
x 2 2 3/ 2
A 2q È ( y + a ) - y3 ˘ ^
q =- Í 2 2 ˙j
F29 4pe0 ÍÎ y ( y + a 2 )3/ 2 ˙˚
È Ê 2 3/ 2 ˘
ˆ
1. F1 = qE towards right Í y3 Á1 + q ˜ - y3 ˙
2q Í Ë Á 2˜
y ¯ ˙^
F2 = qE towards left =- Í ˙j
Net torque about q, 4pe0 Í y2 ( y2 + a 2 )3/ 2 ˙
Í ˙
t = qE (2l - x ) sin q + qEx sin q ÍÎ ˙˚
= q (2l) E sin q = pE sin q As y >> a
Æ Æ Æ
t = p¥ E È 3Ê 3 q2 ˆ ˘
Í y ÁÁ1 + ˜ - y3 ˙

1 q 2q Í Ë 2y ¯ ˙^
E1 = ◊ E=- ◊ ˙j
2.
4pe0 ( y2 + a 2 )2 4pe0 Í y5
Í ˙
1 q ÍÎ ˙˚
E2 = ◊
4pe0 ( y2 + a 2 )2 2
3 qa ^
E=- j
1 2q 4pe0 y4
E3 = ◊
4pe0 y2
Net field at P

Introductory Exercise 21.8


1. (a) Charge q is completely the hemisphere 1 q
fEFGH = =
hence flux through hemisphere is zero. f 2pe0
(b) Charge inside the sphere is q hence flux and fAEHD = 0
through hemisphere
q 3. True. As electric field is uniform, flux
f= entering the cube will be equal to flux
e0
leaving it.
(c) As charge q is at the surface, net flux q
\ fnet = 0 fi fnet =
through hemisphere e0
q
f= fi q =0
2 e0
4. (a) As net charge inside hemisphere is zero,
2. When charge is at any of the vertex, net flux E
through the cube, 1
q
f=
8e0
If charge q is at D, 2
flux through three faces containing D is zero
and the flux f is divided equal among other f1 + f2 = 0
three faces, hence But E is parallel to surface 2.
35
\ f2 = 0
Hence, f1 = 0 –POQ = 2q = 120∞
(b) Again, f1 + f2 = 0 2p
\Length of arc PQ = R
E 3
1 2 Charge inside sphere,
q 2p q
q= 0 ¥ Rfi 0
2pR 3 3
\Flux through the sphere
q q
f= = 0
f2 = E ¥ pR2 = p R2E e0 3 e0
\ f1 = - f2 = - p R2E
6. Net charge inside the cube = 0.
R/2 1
5. cos q = = , q = 60∞ \Net flux through the cube = 0.
R 2

P Q
R
q R —
2
O

Introductory Exercise 21.9


1 q + q + qB
1. VB = ◊ =0
4pe0 2R ) q' +
q+Q
+Q
Q

B q
+

–(q +Q)
'+
3q

–2q C
–(q

q+Q
B
A +2q
A
–q R

q R 2R 3R

2R

1 Ê q¢ + q + Q 2q - Q ˆ
VB = ◊Á + ˜ =0
q B = - 2q 4pe0 Ë 2R 3R ¯
Total charge inside a conducting sphere fi 3 q¢ + q + f = 0 …(i)
appears on its outer surface, Again, VP = VC
\Charge on outer surface of A = 2q 1 Èq + Q q 2q - Q ˘ 1 3 q + q¢
◊Í + + ˙= ◊
and charge on outer surface of B 4pe0 Î R 2R 3 R ˚ 4pe0 3R
= 2q - 2q = 0 6( q + Q ) + 3 q¢ + 2(2q - Q ) = 2(3 q + q¢ )
2. Let q¢ = charge on sphere B and charge f 4q + 4q + q¢ = 0
flows from sphere C to A.
On solving
36
5 24 3.
Q=- q, q¢ = - q 2q + q'
11 11
–(q') C
A B C B
q'
Charge on 0 - (q + Q ) - (q¢ + q + Q ) –2q A + 2q
–q
inner surface 6 18
=- q = q q A
11 11
Charge on q + Q = 6 q¢ + q + Q 3q + q = 9 q
outer surface 11 18 11
=- q
11

A B C
Charge on - q - 2q 4
+ q
inner surface 3
Charge on + 2q 4q 2
- + q
outer surface 3 3

AIEEE Corner
Subjective Questions (Level-1)
1 q (Q - q) 1 q1q2
1. F = ◊ 4. F1 = ◊ …(i)
4pe0 r2 4pe0 r 2
For maximum force 1 q2
F2 = ◊ …(ii)
dF 1 Q - 2q 4pe0 r 2
= ◊ =0
dq 4pe0 r2 [As both the spheres are identical, find
Q charge on both the spheres will be equal]
q= q - q2
2 q= 1
d 2F 1 -2 2
= ◊ <0
dq2 4pe0 r 2 fi q1 - q2 = 2q
Q From Eq. (ii),
Hence F is maximum at q = .
2 q2 = 4pe0 r 2F2
2. Minimum possible charge on a particle = e. (50 ¥ 10-2 )2 ¥ 0.036
9
1 e 2 9 ¥ 10 ¥ (1.6 ¥ 10 )
-19 2 = = 10-12
\Fmin = ◊ 2= 9 ¥ 109
-2 2
4pe0 r (1 ¥ 10 )
q = 10-6 C = 1 mC
= 2.3 ¥ 10-24 N From Eq. (i),
1 q1q2
3. Fe = ◊ …(i) (50 ¥ 10-2 )2 ¥ 0.108
4pe0 r 2 q1q2 = 4pe0r 2F1 =
9 ¥ 109
Gm1m2
Fg = …(ii) = 3 ¥ 10-12
r2
Fe q1q2 Also, q1 + q2 = 2q = 2 ¥ 10-6
=
Fg 4pe0 Gm1m2 On solving
(3.2 ¥ 10-19 )2 ¥ 9 ¥ 109 q1 = ± 3 mC
=
6.67 ¥ 10-11 ¥ (6.64 ¥ 10-27 )2 and q2 = + 1 mC
= 3.1 ¥ 1035
37
1 q1 Q 1 4q2
5. (a) F1 = ◊ FAB = ◊
4pe0 (3 a / 2)2 4pe0 L2
–a +a/2 +a 1 qQ
–a1 FAO = ◊
O Æ Q Æ q2 4pe0 x 2
F2 F1
1 qq Q
1 4q1 Q = ◊
F1 = ◊ …(i) 4pe0 L2
4pe0 9a 2
1 qQ For net force on Q to be zero.
F2 = ◊ 2
4pe0 ( a / 2)2 FAB = FAO
1 4q2 1 qq Q
1 4q2 Q ◊ = ◊
= ◊ …(ii) 4pe0 L2 4pe0 L2
4pe0 a 2
9
For net force on Q to be zero fi q= Q
4
F1 = F2 4
fi Q= q
or q1 = 9 q2 9
1 4q1 Q -4
(b) F1 = ◊ As Q is negative fi q = q
4pe0 25a 2 9
+a (b) PE of the system
–a +Q 2
+ 1 È 4q2 qQ 4qQ ˘
q1 O Æ Q Æ U= ◊Í + + ˙
F2 F1 q2 4pe0 ÍÎ L x L - x ˙˚
1 4q2 Q 1 È 4q2 4qQ 8qQ ˘
F1 = ◊ = ◊Í - - ˙ =0
4pe0 9a 2 4pe0 ÍÎ L 3L 3 L ˙˚
For net force on Q to be zero. Hence, equilibrium is unstable.
F1 + F2 = 0 7. FBD of af placed at left can be given by
q1 25 N sin 60° O
fi =
q2 9 N

6. (a) In order to make net force on charge at A R R


60°
and B zero, Q must have negative sign. Fe N cos 60°
O –x
x
A B A B
+q Q +4q R
mg
Let the charge Q is planed at a distance x
from A (+ Q charge) DABD is equilateral
1 qQ As beads are in equilibrium
FOA = ◊
4pe0 x 2 mg = N sin 60∞
1 4q Q Fe = N cos 60∞
FOB = ◊
4pe0 ( x - x )2 Fe
= cot 60∞
mg
For net force on Q to be zero.
FOA = FOB q2 = 4pe0 R2 mg cot 60∞
1 qQ 1 4q Q 4pe0 R2 mg
◊ 2 = ◊ q=
4pe0 x 4pe0 ( L - x )2 3
( L - x )2 = (2x )2 6pe0mg
= 2R
L 3
x=
3
Force on A,
38
8. As ball are in equilibrium Here, dE sin f components of field will cancel
O each other.
Hence, Net field at O
a 1 l p/ 2
E = Ú dE cos f = ◊ Ú cos f df
T T 4pe0 R - p/ 2
1 2l
= ◊
4pe0 R
T cosa
13. Consider elementary portion of the rod of
a
length dl at a distance l from the centre O of
Fe Fe the rod.
T sina
r dE sin f
mg mg dl
df dE

Fe = T sin a l
mg = T cos a f
O dE cos f
Fe = mg tan a Q dE cos f
P
q2 = 4pe0r 2 tan a
Here, r = 2 l sin a dE
q = 16pe0 l2 sin 2 a tan a
2

dl dE sin f
q = 3.3 ¥ 10-8 C.
9. Same as Q.7. Introductory Exercise 21.3. Charge on this portion
Q
10. See Q.7. Introductory Exercise 21.3. dq = l dl = dl
1 q Æ L
11. E= ◊1 r 1 dq
4pe0 r 3 \ dE = ◊
9 -9 4pe0 ( a sec f)2
9 ¥ 10 ¥ ( - 8.0 ¥ 10 ) ^ ^
= (1.2 i - 1.6 j ) 1 Q dl
((1.2)2 + (1.6)2 )3/ 2 = ◊
4pe0 La 2 sec 2 f
^ ^
= - 18 2 (1.2 i - 1.6 j ) N/C. Now,
12. Consider an elementary portion on the ring l = a tan f
of length dl subtending angle df at centre ‘O’ fi dl = a sec 2 f df
of the ring. 1 Q df
\ dE = ◊
Charge on this portion, 4pe0 La
dE cos f Net Electric field at P.
dE cos f
dE
dE E = Ú dE cos f
[ dE sin f components will cancel each other
df O as rod in symmetrical about P.]
dE sin f dE sin f
1 Q q
4pe0 La Ú- q
f = ◊ cos f df

R 1 2Q sin q
= ◊
4pe0 La
dl dl L L
But sin q = =
2
L 4a + L2
2
2 a 2 + ÊÁ ˆ˜
dq = l dl = l Rdf Ë2¯
1 dq 1 l df 1 2Q
\ dE = ◊ 2 = \ E= ◊
4pe0 R 4pe0 R 4pe0 a 4a 2 + L2
39
14. (a) As shown in figure, direction of electric Clearly resultant field is along angle bisector
field at P will be along + ve y-axis. of field towards 9 and 10.
y 6E1
6E1
–Q 6E1
E
E1 E2 E
6E1
x
P
6E1

Q 6E1

Hence time shown by clock in the direction


of electric field is 9 : 30.
(b) Positive x-axis. F - eE
16. (a) a = =
y m m
- 1.6 ¥ 10-19 ¥ 1 ¥ 103
=
E2 9.1 ¥ 10-31
= - 1.76 ¥ 1014 ms -2
+Q
P
x u = 5.00 ¥ 108 cm/s = 5 ¥ 106 ms -1
E
v=0
+Q E1 v2 - u 2 = 2as
(5 ¥ 106 )2
s= = 1.4 ¥ 10-2 = 1.4 cm
(c) Positive y-axis. 2 ¥ 1.7 ¥ 1014

E
(b) v = u + at
5 ¥ 106
E1 E2 t= = 2.8 ¥ 10-8 = 28 ns.
1.76 ¥ 1014
–Q
(c) Dk = work done by electric field.
P = F ◊ x = - eEx
= - 1.6 ¥ 10-19 ¥ 1 ¥ 103 ¥ 8 ¥ 10-3
+Q = - 1.28 ¥ 10-18 J
Loss of KE = 1.28 ¥ 10-18 J
25
1 q 17. Here, ux = u cos 45∞ = ms -1
15. Let E1 = ◊ 2
4pe0 R2 u E
12
11 1
E12 q
E11 E1
10 2
E10 E2 25
u y = u sin 45∞ = ms -1
9 E9 E3 3 2
E8 E4 a x = qE = 2 ¥ 10-6 ¥ 2 ¥ 107
8 4 = 40 ms -1
E7 E E5
6
7 5 a y = - 10 ms -1
6 1 2
y = u yt + t
Resultant fields of two opposite charges can ay
be shown as given in figure. 25
y= t - 5t 2
2
40
at the end of motion, Case I.
t = T and y = 0 In between two charges : let potential is zero
5 at a distance x from q1 towards q2.
\ T= s
2 x 100–x
q1 q2
Also at the end of motion,
x=R 1 q1 1 q2
V= ◊ + ◊ =0
1 4pe0 x 4pe0 100 - x
\ x = ux t + a x t 2 -6 -6
2 1 2 ¥ 10 1 3 ¥ 10
2 = ◊ - ◊ =0
25 5 Ê 5 ˆ 4pe0 x 4pe0 100 - x
R= ¥ + 20 ¥ Á ˜
2 2 Ë 2¯ fi 200 - 2x = 3 x
= 312.5 m x = 20 cm
m 2 sin 2q Case II.
18. (a) R=
qE Consider the potential is zero at a distance x
qER from charge q, on its left.
sin 2q =
mu 2 x 100 cm
1.6 ¥ 10-19 ¥ 720 ¥ 1.27 ¥ 10-3 q1 q2
=
1.67 ¥ 10-27 ¥ (9.55 ¥ 103 )2 1 q1 1 q2
\ V= ◊ + ◊ =0
= 0.96 4pe0 x 4pe0 100 + x
-6 -6
2q = 88∞ or 92∞ 1 2 ¥ 10 1 - 3 ¥ 10
= ◊ + ◊ =0
q = 44∞ or 46∞ 4pe0 x 4pe0 100 + x
2mh sin q 200 + 2x = 3 x
T=
2E
x = 200 cm
1
2 ¥ 9.55 ¥ 103 ¥ ¥ 1.67 ¥ 10-31 21. Let us first find the potential at a point on
= 2
the perpendicular bisector of a line charge.
1.6 ¥ 10-19 ¥ 720
Consider a line of carrying a line charge
= 1.95 ¥ 10-11 s density l having length L.
Æ -19
Æ eE 1.6 ¥ 10 ¥ 120 ^
19. (a) a = - =- j
m 9.1 ¥ 10-31 dl

^
= - 2.1 ¥ 1013 i m/s
l
-2
Dx 2 ¥ 10 4
(b) t = = 5
= ¥ 10-7 s f
ux 1.5 ¥ 10 3 L
q
vy = u y + a y t
4
= 3.0 ¥ 106 ¥ 2.1 ¥ 1013 ¥ ¥ 10-7
3
= 0.2 ¥ 106 m/s
Æ ^ ^ r
v = (1.5 ¥ 105 ) i + (0.2 ¥ 106 ) j
Consider an elementary portion of length dl
20. Absolute potential can be zero at two points on the rod.
on the x-axis. One in between the charges
and other on the left of charge a1 (smaller in Charge on this portion
magnitude). dq = l dl
1 l dl
\ dV = ◊
4pe0 r sec f

100cm Now, l = r tan f


O
q2
X dl = r sec 2 f df
q1
41
l sec f df Hence, particle at B is faster than that at A.
\ dV =
4pe0 24. Centre of circle is equidistant from every
l q
point on its periphery,
\ V = Ú dV = ◊ Ú sec 2 f df
4pe0 q - 1 q
Hence, V0 = ◊ ,
l 4pe0 R
= [ln|sec q + tan q|]q- q
4pe0 where q = Q1 + Q2 = - SQ
l È Ω sec q + tan q Ω˘ 1 5Q
= Í ln Ω Ω˙ \ V0 = - ◊
4pe0 ÍÎ Ω sec q - tan q Ω˙˚ 4pe0 R
2l 1 q
= ln|sec q + tan q| Similarly, Vp = ◊
4pe0 4pe0 R + Z2
2

In the given condition 1 SQ


=- ◊
q = 60∞ 4pe0 R + Z2
2

Potential due to one side 25. Initial PE


2l 1 q1q2
V1 = V2 = V3 = ◊ ln|sec 60∞ + tan 60∞| Ui = ◊
4pe0 4pe0 r1
2l 1 q1q2
= ◊ ln|2 + 3 | Uf = ◊
4pe0 4pe0 r2
Work done by electric force
W = - DU = - (Uf - Ui )
O 1 Ê1 1ˆ
=- ◊ q1q2 Á - ˜
4pe0 Ár ˜
Ë 2 r1 ¯
fi W = - 9 ¥ 109 ¥ 2.4 ¥ 10-6 ¥ ( - 4.3 ¥ 10-6 )
Total potential at O Ê 1 1 ˆ
6l Á - ˜
V = 3 V1 = ◊ ln|2 + 3 | Ë 0.25 2 0.15 ¯
4pe0
W = - 0.356 mJ
Q
= ◊ ln|2 + 3 | 1 È q1q2 q2q3 q3q1 ˘
2pe0 a 26. (a) U = ◊Í + + ˙
4pe0 ÍÎ r12 r23 r31 ˙˚
Æ Æ
22. (a) V2 - V1 = - E◊ d = - 250 ¥ 20 ¥ 10-2 È 4 ¥ 10-9 ¥ ( - 3 ¥ 10-9 )
= 9 ¥ 109 Í
= - 50 V ÍÎ 0.2
W = DV = q( V2 - V1 ) ( - 3 ¥ 10-9 ) ¥ (2 ¥ 10-9 )
= 12 ¥ 10-6 ¥ - 50 = - 0.6 mJ +
0.1
(b) V2 - V1 = - 50 V
4 ¥ 10-9 ¥ 2 ¥ 10-9 ˘
23. By work energy theorem + ˙
0.1 ˙˚
W =DK
1 1 U = 9 ¥ 10-8 [ - 6 - 6 + 8] = - 360 nJ
q( V1 - V2 ) = mv22 - mv12
2 2 (b) Let the distance of q3 from q1 is x cm. Then
- 5 ¥ 10-6 (20 - 800) 1 È q1q2 qq qq ˘
1 U= ◊Í + 2 3 + 3 1˙ =0
= ¥ 2 ¥ 10-4( V22 - (5)2 ) 4pe0 ÍÎ 0.2 0.2 - x x ˙˚
2
È 4 ¥ 10-9 ¥ ( - 3 ¥ 10-9 )
v22 = 55 fi 9 ¥ 109 Í
v2 = 55 = 7.42 ms -1 ÍÎ 20 ¥ 100- 2
When a particle is released in electric field it ( - 3 ¥ 10-9 ) ¥ 2 ¥ 10-9
+
moves in such a way that, it decreases its PE (20 - x ) ¥ 10-2
and increases KE
42
-9 -9
2 ¥ 10 ¥ 4 ¥ 10 ˘ ^ ^
E = - 10 ( 2 i + 2 j + 2 k )
^
+ ˙ =0
x ¥ 10-2 ˙˚ ^ ^ ^
= - 20 ( i + j + k )
6 6 8
fi - - + =0 Æ Æ
10 20 - x x 32. VB - V0 = - E◊ r
fi x = 6.43 cm fi V - 0 = - (40 + 60)
27. Let Q be the third charge fi V = - 100
1 È q2 qQ qQ ˘ ∂v
U= ◊Í + + 33. (a) Ex = - = - ( Ay - 2 Bx )
˙ =0 ∂x
4pe0 ÍÎ d d d ˙˚
∂V
q Ey - = - ( Ax + C )
Q=- ∂y
2
∂V
Æ Æ Ez = - =0
28. V = - E◊ r ∂Z
Æ
(a) r = 5 k
^ (b) For E = 0
Ex = 0 and Ey = 0
^ ^ ^
V = - (5 i - 3 j) - (5 k ) = 0 Hence, Ey = 0
Æ ^
(b) r = 4 i + 3 k
^ Ax + C = 0
C
^ ^ ^ ^ x=-
V = - (5 i - 3 j) - (4 i + 3 j) A
= - 20 kV Ex = 0
C
Æ
29. E = 400 j V/m
^ Ay - 2 B ÊÁ - ˆ˜ = 0
Ë A¯
Æ ^
(a) r = 20 j cm = (0.2 j) m
^ 2 BC
y=-
Æ Æ A2
V = - E◊ r = - 80 V Ê C 2 BC ˆ
Hence, E is zero at Á - , - ˜.
Æ
(b) r = ( - 0.3 j) m
^ Ë A A2 ¯
q
Æ Æ 34. f =
V = - E◊ r = 120 V e0
Æ
(c) r = (0.15 k )
^
q = e0f = 8.8 ¥ 10-12 ¥ 360
= 3.18 ¥ 10-9 C
V =0
Æ ^
= 3.186 nC
30. E = 20 i N/C 3.60 ¥ 10-6
q
Æ 36. (a) f = =-
^
(a) r = (4 i + 2 j ) m
^ e0 8.85 ¥ 10-12
Æ Æ
V = - E◊ r = - 80 V = 4.07 ¥ 105 V-m.
q
Æ ^ ^
(b) f = fi q = e0f
(b) r = ( 2 i + 3 j ) m e0
Æ Æ
V = - E◊ r = - 40 V = 8.85 ¥ 10-12 ¥ 780 = 6.903 ¥ 10-9
q = 6.903 nC
[V ] [ ML2 T -3 I-1 ]
31. (a) [ A ] = = (c) No.
[ xy + yz + zx ] [L2 ] Net flux through a closed surface does
= [ ML0 T -3 I-1 ] not depend on position of charge.
Æ Ê ∂v ^ ∂v ^ ∂v ^ ˆ Æ Ê3 ^ 4 ^ˆ
(b) E = - — V = - ÁÁ i+ j+ k˜ 36. E = Á E0 i + E0 j˜
Ë ∂x ∂y ∂z ˜¯ Ë 5 5 ¯
Æ ^ 1^ 2
^ ^
= - A [( y + z ) i + ( z + x ) j + ( x + y) k ]
^ S = 0.2 j m 2 = jm
5
(c) at (1m, 1m, 1m) Æ Æ 4
\ f = E◊ S = Nm 2/C
25
43
4 1 p
= ¥ 2.0 ¥ 103 N-m 2/C cos a = fia=
25 2 3
= 320 N-m 2/C R = b tan a = 3 b
Æ E0 x ^ Hence proved
37. E = i x1 = 0
l Æ ^ ^ ^ Æ ^ Æ ^
Æ 39. E = - B i + C j - D k, S1 = - L2 i, S2 = - L2 j,
E1 = 0
Æ ^ Æ ^ Æ ^
x2 = a S3 = - L2 i, S4 = - L2 j, S4 = - L2 k,
Æ E a^ Æ ^
E2 = 0 i S6 = - L2 k
l
ÆÆ ÆÆ
Flux entering the surface \ f1 = E ◊ S1 = BL2, f2 = E ◊ S2 = CL2,
f1 = 0 ÆÆ ÆÆ
f3 = E ◊ S3 = - BL2, f4 = E ◊ S4 = - CL2,
Flux leaving the surface
E a3 ÆÆ ÆÆ
f2 = E2a 2 = 0 f5 = E ◊ S5 = - DL2, f6 = E ◊ S6 = DL2
l
(b) f = f1 + f2 + f3 + f4 + f5 + f6 = 0
5 ¥ 103 ¥ (1 ¥ 10-2 )3
= 40. W = 2p (1 - cos f)
2 ¥ 10-2
= 0.25 N-m 2/C
q
Net flux, f =
e0
q a
f2 - f1 = q –q
e0
q = e0 ( f2 - f1 )
= 8.85 ¥ 10-12 ¥ 0.25
= 2.21 ¥ 10-12 C = 2.21 pC
38. Consider the charge is placed at vertex of the W 2p (1 - cos a ) q
cone of height b and radius R. f1 = f2 = cot ftotal = ◊
4p 4p e0
1
= (1 - cos a )
R 2e0
Total flux through the ring
a f = f1 + f2
q q ÊÁ l ˆ
˜
Q = (1 - cos a ) = 1-
e0 e0 Á 2 2˜
R +l ¯
Let a be the semi-vertical angle of the cone, Ë
then solid angle subtended by the cone. 41. From the given equation,
y
W = 2p (1 - cos a )
Flux passing through cone
W a
f= ◊ ftotal
4p q 45°
x
1 O
But f = ftotal (Given)
4
W=p
2p (1 - cos a ) = p a
1
1 - cos a = radius of hemisphere = a
2
and its centre is at ( a, 0, 0)
44
Ê 1 ˆ
VB =
1 Ê q A + q B + qC ˆ
W = 2p (1 - cos a ) = 2p Á1 - ˜ Á ˜
Ë 2¯ 4pe0 Ë b c ¯
Ê 1 ˆ s Ê a2 ˆ
2p Á1 - ˜ = Á - b + c˜
W 2¯ q Á ˜
f= ftotal = Ë ◊ e0 Ë b ¯
4p 4p e0 1 Ê q A + q B + qC ˆ
q Ê 1 ˆ VC = Á ˜
f= Á1 - ˜ 4pe0 Ë c ¯
2 e0 Ë 2¯
s Ê a 2 - b2 + c 2 ˆ
2 2 = Á ˜
42. q1 = s (4pr ), q2 = s (4p R ) e0 Á c ˜
q2 Ë ¯
44. (a) As charge Q is placed at the centre of the
q1
sphere, charge - Q will appear on the
inner surface and Q on its outer surface.
+Q
r A B
–Q

R
Q

Q
But, q1 + q2 = Q fi s =
4 p ( r + R2 ) 2
-Q
Hence, sin =
1 Ê q1 q2 ˆ 4pa 2
VA = ◊Á + ˜
4pe0 Ë r R¯ Q
and sout =
2 2
1 Ê s (4pr ) s (4p R ) ˆ 4pa 2
= ◊Á + ˜
4pe0 ÁË r R ˜ (b) Entire charge inside the sphere appears
¯
on its outer surface, hence
1 Q ( r + R) Q+q
= ◊ Q
4pe0 r 2 + R2 sin = - and sout =
4pa 2 4pa 2
1 Ê q1 + q2 ˆ 1 Q (c) In case (a)
VB = ◊Á ˜= ◊
4pe0 Ë R ¯ 4pe0 R 1 Q
E= ◊
2
43. q A = s (4pa ), q B = - s (4pb ) 2 4pe0 x 2
s +Q
–Q
–s

s
Q
a
x
A b
B
In case (b)
C c
E = E1 + E2
E1 = Field due to charge Q.
qC = s (4pc 2 )
E2 = Field due to charge on shell.
1 Ê q A q B qC ˆ
VA = Á + + ˜ 1 Q
4pe0 Ë a b c ¯ E= ◊
4pe0 x 2
s
= ( a + b + c) for x < a
e0
As field due to shell is zero for x < a.
45
1 Q+q (d) Let Q A be the charge on inner sphere.
and E= ◊ , for x > a
4pe0 x 2 –Q
45. Let Q be the charge on the shell B,
b QA
—q –q
c
b –b R
—q —q
c c
C –q
q C 3R
A
B
a
A
B
b 1 È QA Q ˘
VA = ◊Í - =0
4pe0 Î R 3 R ˙˚
Q
QA =
3
1 È q + Q - q˘
VB = Í + ˙ =0 47. (a) At r = R
4pe0 Î b c ˚ 1 È Q - 2Q 3Q ˘
V= ◊Í + + ˙
Ê b - cˆ 4pe0 Î R 2R 3 R˚
Q = qÁ ˜
Ë c ¯ 1 Q
= ◊
Charge distribution on different surfaces is 4pe0 R
shown in figure.
At r = 3 R
46. (a) Let E1 and E2 be the electric field at P due 1 È Q - 2Q + 3Q ˘
to inner shell and outer shell respectively. V= ◊Í ˙
4pe0 Î 3R ˚
1 2Q
+2Q = ◊
P r 4pe9 3 R

R B (b) Let E1, E 2 and E 3 be the electric fields at


5
A r = R due to shells A, B and C
3R 2
–Q respectively.
C
1 2Q 3R B
Now, E1 = ◊ and E2 = 0
4pe0 r
A
1 2Q
\ E = E1 + E2 = E1 = ◊ R
4pe0 r –2Q 3Q
1 È 2Q Q ˘ Q
(b) VA = ◊ - 2R
4pe0 ÍÎ R 3 R ˙˚
1 È 2Q - Q ˘
VB = ◊Í ˙
4pe0 Î 3 R ˚
1 È 2Q 2Q ˘ 1 Q
VA - VB = ◊ - \ E1 = ◊
4pe0 ÍÎ R 3 R ˙˚ 4pe0 Ê 5 ˆ 2
Á R˜
1 4Q Ë2 ¯
= ◊
4pe0 3 R 1 4Q
= ◊ (outwards)
4pe0 25 R
(c) Whenever two concentric conducting
spheres are joined by a conducting wire 1 2Q
E2 = ◊
entire charge flows to the outer sphere. 4pe0 Ê 5 ˆ 2
Á R˜
\ Q A = 0, Q B = 0 Ë2 ¯
46
1 8Q 1 È Q - q - 2Q + 3Q + q ˘
= ◊ (inward) VB = ◊Í ˙
4pe0 25 R 4pe0 Î 3R ˚
E3 = 0 1 2Q
= ◊
5 4pe0 3 R
Net field at r = R
2 But VA = VB
1 4Q
E = E2 + E1 = ◊ (inward) 1 3Q - 2q 1 2Q
4pe0 25 R ◊ = ◊
4pe0 3R 4pe0 3 R
(c) Total electrostatic energy of system is the Q
fi q=
sum of self-energy of three shell and the 2
energy of all possible pairs i.e., Q
2 \Charge on innermost shell = Q - q =
1 È Q 2 ( - 2Q ) (3Q )2 2
U= ◊Í + +
4pe0 ÍÎ 2 R 2 ¥ 2 R 2 ¥ 3 R 7Q
and charge on outermost shell= 3Q + q =
Q ( - 2Q ) ( - 2Q ) ¥ 3Q Q ¥ 3Q ˘ 2
+ + + ˙ 1 3Q - 2q
2R 3R 3R ˚ and VA = ◊
4pe0 3R
1 Q
U= ◊ 1 2Q
4pe0 R = ◊
4pe0 3 R
(d) Let q charge flows from innermost shell
(c) In this case
to outermost shell on connecting them 1 Q
with a conducting wire. E1 = ◊ 2
4pe0 5
3Q +q 2 R ÊÁ Rˆ˜
Ë2 ¯
–2Q 1 2Q
= ◊ (outward)
2R 4pe0 25 R
1 2Q
E2 = ◊
Q2 R 4pe0 Ê 5 ˆ 2
Á R˜
Ë2 ¯
1 8Q
3R = ◊ (inward)
4pe0 25 R
5
Net electric field at r = R
1 È Q - q - 2Q 3Q + q ˘ 2
VA = ◊Í + + ˙ 1 6Q
4pe0 Î R 2R 3R ˚ E = E2 - E1 = ◊ (inward)
4pe0 25 R
1 3Q - 2q
= ◊
4pe0 3R

Objective Questions (Level-1)


1. f = E ◊ A 3. Electric lines of force terminate at negative
Units of f = N/C ¥ m 2 = N - m 2/C charge.
or V/m ¥ m 2 = V - m 1 q2
4. F = ◊
4pe0 l2
2. Net force
F ¢ = mg - qE Initial PE
qE 1 Ê q2 q2 q2 ˆ
g¢ = g - Ui = ◊Á + + ˜ = 3 Fl
m 4pe0 ÁË l l l ˜¯
l
T ¢ = 2p >T Find PE
g1
47
1 Ê q2 q2 q2 ˆ 3 2 q2 q2
Uf = ◊Á + + ˜ = Fl W = Uf - Ui = - + [4 - 2 ]
4pe0 ÁË 2l 2l 2l ˜¯ 2 4pe0 a 4pe0 a
3
W = Uf - Ui = - Fl q2
= [4 - 2 2 ] J
2 4pe0a
5. KE = qV 8. Potential at point P
1
mv2 = qV
2
2qV 2R
v=
m
P
q1V1 q2V2 q3V3 R
fi V1 : V2 : V3 = : :
m1 m2 m3
e ¥1 e ¥2 2e ¥ 4 3R
fi V1 : V2 : V3 = : :
m 2m 4m
1 q1 + q2
V1 : V : V3 = 1 : 1 : 2 V= ◊
4pe0 3R
1 q 9 ¥ 109 ¥ 3 ¥ 10-6
6. V = ◊ fi = 9000
4pe0 r 3R
R =1m
9. As distance of every point of ring from axis is
4r same.
kq
V= , But x = 2 R
R2 + x 2
kq
=
1 2q V 3R
V¢ = ◊ = 10. For equilibrium,
4pe0 4r 2
w
1 È q2 q2 ˘
7. Ui = Í- ¥4 + ¥ 2˙ Fe
4pe0 ÍÎ a 2a ˙˚ B
q
+q –q +q –q r g sin
m

30° mg mg cos q
A

mg sin 30∞ = Fe
–q +q +q –q 1 q2
mg sin 30∞ = ◊
- q2 4pe0 r 2
= [4 - 2 ]
4pe0 a 1
r=q
1 È q 2
q q2
˘ 2 4pe0 mg sin 30∞
Uf = ◊ Í- ¥2 + ¥2 - ¥ 2˙
4pe0 ÍÎ a a 2 ˙˚ 91 ¥ 109
= 2.0 ¥ 10-6
2 q2 1
=- 0.1 ¥ 10 ¥
4pe0 a 2
ª 20 cm
48
11. Net force on C = 0 13. Data is not sufficient.
A B
14. If the charges have opposite sign, electric
field is zero on the left of smaller charge.
15. Net field is only due to charge on C.
Aq qB
O

C FCD
D F qC
FCO
FCA
FCB
q q
E D
1 (2 2 - 1) Q 2
2
FCB = ◊ 1 q q
4pe0 a2 E= ◊ 2
=
2 2 4pe0 (2a ) 16pe0 a 2
1 (2 2 - 1) Q
FCD = ◊ 16. On touching two spheres, equal charge will
4pe0 a2
2 2 appear on both the spheres and for a given
1 (2 2 - 1) Q total charge, force between two spheres is
FCA = ◊
4pe0 2a 2 maximum if charges on them are equal.
2
1 2 (2 2 - 1) Q 17. Charge distribution is shown in figure.
FCO = ◊
4pe0 a2 +8Q
–2Q
Net force on C +2Q
F = FCA + FCO + FCB cos 45∞ + FCO cos 45∞ –4Q
+4Q
(2 2 - 1)Q È (2 2 - 1)Q (2 2 - 1)Q
= Í +
a2 ÍÎ 2 2
(2 2 - 1)Q 2q ˘
+ + ˙ =0
2 1 ˙˚
7Q
q=-
4
s 1 q
12. E = 18. V = ◊
e0 4pe0 r
es If drops coalesce, total volume remains
F = eF =
e0 conserved,
4 4
Acceleration of proton pR3 = 1000 ¥ pr 3
F se 3 3
a= =
m me0 R = 10r
1 1 1000q
s = ut + t 2 V¢ = ◊ = 10V
a 4pe0 10q
u =0 1 Èq Q˘
19. VA = ◊ + ˙
25 me0 4pe0 ÍÎ r R˚
25 B q
t= =
a se
r
2 ¥ 0.1 ¥ 1.67 ¥ 10-27 ¥ 8.8 ¥ 10-12 A
=
2.21 ¥ 10-9 ¥ 1.6 ¥ 10-19
R
= 2 2 ms
49
1 È q + Q˘
VB = ◊Í ˙
4pe0 Î R ˚
q È1 1 ˘
VA - VB = ◊ - O
4pe0 ÍÎ r R ˙˚ P
R r
\ VA - VB µ q
If q is doubled, VA - VB will become double.
20. Charge distribution is shown in figure. 1 q 1 1 3q
2Q ◊ = ◊ ◊
4pe0 R + r 2 4pe0 2 R
–3Q
3Q 1 3∞
=
–Q R + r 4R
Q
4R = 3 R + 3r
R
r=
3
25. Net charge on any dipole is zero.
26. For net force to be zero.

ÆÆ ^ ^ ^ T T cos q
21. f = E ◊ S = (5 i + 2 j) ◊ ( i ) = 5 V-m. q
1 Qq q
22. FDA = FDC = ◊
4pe0 a 2 qE
q 7 sin q
A Q B

mg
Q
mg
T cos q = mg fi T =
cos q
q Q qE
FDC or T sin q = qE fi T =
D C sin q
1 q V1
FDB 27. E1 = ◊ =
4pe0 a a
FDA 1 q V2
E2 = =
1 q2 4pe0 b2 b
FDB = ◊ 2
4pe0 2a But E1 = E2
Net force on charge at D V1 V2
=
Æ a b
F0 = FDB + FDA cos 45∞ + FDB cos 45∞ = 0 V1 a
fi =
1 q Èq Q Q˘ V2 b
fi 4pe0 ◊ 2 Í2 + + ˙ =0
4 a Î 2 2˚ 28. Electric field on equatorial lines of dipole is
q = -2 2Q opposite to dipole moment.
23. As VB = 0, Total charge inside B must be zero 29. Potential difference between two concentric
and hence charge on its outer surface is zero spheres is independent of charge on outer
and on its inner surface is - q. sphere.
1 1 q
24. Vp = 30. E = ◊ 2
V0 4pe0 r
50
1 q 1 q VR
V= ◊ = Er E= ◊ = 2
4pe0 r 4pe0 r 2 r
V 3000
r =Ω Ω = =6m 36. When outer sphere is earthed field between
Ω E Ω 500 the region of two spheres in non-zero and is
6 ¥ ( - 3000) zero in all other regions.
q = 4pe0rV = = - 2 mC
9 ¥ 109 ÆÆ
37. W = F ◊ s = qEs cos q
31. F1 = F2 W 4
1 q1q2 1 qq E= = = 20 N/C
◊ = ◊ 1 2 qs cos q 0.2 ¥ 2 ¥ cos 60∞
4pe0 r12 4p Ke0 r22
r 50 1 ÈÍ Q Q ˘
˙
r2 = 1 = = 10 5 m 38. V1 = ◊ -
K 5 4pe0 Í R d 2 + R2 ˙˚
Î
ª 22.3 m Q –Q
32. Electric field at a distance r from infinite line
charge R R
l
E=
2pe0r
dV = - E dr
V2 b r d
ÚV1
dV = - Ú E dr
a
1 2

fi V2 - V1 =
l
◊ ln
1 1 ÈÍ Q Q ˘
˙
V2 = - +
2pe0 2 . Í R d 2
+ R 2 ˙
4pe0 Î ˚
ql 1
W = q( V2 - V1 ) = ln È ˘
2pe0 2 1 Í 2Q 2Q ˙
V1 - V2 = - -
. Í R d 2
+ R 2˙
33. As negative charge is at less distance from 4pe0 Î ˚
the line charge, it is attracted towards the È ˘
line charge. Q Í 1 1 ˙
V1 - V2 = - +
. Í R d + R2 ˙˚
2
34. r = (4 - 1)2 + (2 - 2)2 + (0 - 4)2 = 5 m 4pe Î
0
9 -8
1 q 9 ¥ 10 ¥ 2 ¥ 10 39. Electric field inside a hollow sphere is
V= ◊ = = 36 V
4pe0 r 5 always zero.
ÆÆ ÆÆ
(b) and (c) are wrong. 40. W = F ◊ r = q E ◊ r
1 q
35. V = ◊ ^ ^ ^ ^
= q ( E1 i + E2 j) - ( a i + b j)
4pe0 R
At a distance r from the centre, = q ( aE1 + bE2 )

JEE Corner
Assertion and Reason
1. Negative charge always moved towards If q1 and q2 have opposite sign, U decreases
increasing potential. with decrease in r.
On moving from A to B potential energy of dU
F=- fi work done by conservative force
negative charge decreases hence its KE dr
increases. always decreases PE.
1 q1q2 dV
2. U = ◊ 3. E = - = - (10) = 10 V/m along x-axis.
4pe0 r dr
1 q
4. V = ◊
4pe0 R
51
Inside the solid sphere. ^ ^ ^
VA = - (4 i + 4 j) ◊ (4 i ) = - 16 V
1 qr
E= ◊ ^ ^ ^
4pe0 R3 VB = - (4 i + 4 j) ◊ (4 i ) = - 16 V
R VA = VB
at r =
2 Hence, Assertion is false.
1 q V
E= ◊ = 7. In the line going A and B, the energy of third
4pe0 2 R2 2 R
charge is minimum at centre.
Assertion is correct. 8. Dipole has both negative and positive
Reason is false as electric field inside the charges hence work done is not positive.
sphere is directly proportional to distance
9. Charge outside a closed surface can produce
from centre but not outside it.
electric field but cannot produce flux.
5. Gauss theorem is valid only for closed 1 qx a
surface but electric flux can be obtained for 10. E = ◊ 2 2 3/ 2
is maximum at x =
4pe0 ( x + a ) 2
any surface.
1 q
6. Let V0 = Potential at origin, But V = ◊ is maximum at x = 0.
4pe0 a 2 + x 2

Objective Questions (Level-2)


1. Electrostatic force always acts along the line Æ ^ Æ ^ ^
2. v1 = v j, v 2 = 2v cos 30∞ i + 2v sin 30∞ j
joining the two charges, hence net torque on
^ ^
charge + 2q is always zero. = 3 i+vj
U
As velocity along y-axis is unchanged,
electric field along x-axis is zero.
30° For motion along x-axis,
+Q +2q vx2 - ux2 = 2a x ( x - x0 )
R
( 3v)2 - 0 3v2
ax = =
As net torque is zero angular momentum of 2a 2a
charge remains conserved. 3 mv2
Fx = ma x =
Initial angular momentum 2a
Li = m ( V sin 30∞ ) R Æ 3 mv2 ^
fi F = i
When the separation between the charges 2a
become minimum, direction of motion of Æ Æ
charge + 2q become perpendicular to the line Also, F =-eE
joining the charges. Æ 3 mv2 ^
V' E =- i
2ea
Rate of work done by electric field at B
Æ Æ Ê 3 mv ^ˆ ^ ^
P = F◊ v = Á i ˜ ◊ ( 3v i + v j)
r +2q Ë 2 a ¯
3 3 mv3
+1q =
2a
\find angular momentum 3. Electric field is always possible, hence a
mvr
Lf = mv¢ r = must be positive and b must be negative.
2 +q –Q
By conservation of angular momentum a b
3
Li = Lf fi r = R
2
52
2 2
4. The system can be assumed as a 1 q q
Uf = ◊ =
combination of three identical dipoles as 4pe0 2 ¥ 3 a 24pe0 a
shown in figure. q2
Here, P1 = P2 = P3 = Q (2a ) Heat produced = Ui = Uf =
8pe0a
Æ
P2 kq2
=
2a
7. Let Q charge flows to C
D –q
Æ 60°
P1 4a
60° Q
q

Æ
P3 3a q

Net dipole moment of the system C 2a


A
P = P1 + P2 cos 60∞ + P3 cos 60∞ B
= 2 p = 4 Qa
Electric field on equatorial lines of short
dipole is given by
1 P
E= ◊ 1 È q + Q ( - q) ˘
4pe0 x 3 VC = ◊Í + ˙ =0
4pe0 Î 3 a 4a ˚
1 4 Qa Qa q
= ◊ = Q=-
4pe0 x 3 pe0x 3 4
5. Potential at centre will be same as potential 1 È q Q - q˘
\ VA = ◊ + +
at the surface of inner shell i.e., 10 V. 4pe0 ÍÎ 2a 3 a 4a ˙˚
6. Initial charge distribution is shown in figure, 1 È q ˘
= Í ˙
Initial energy of system 4pe0 ÍÎ s a ˙˚
+q 1 q
VA - VC = ◊
4pe0 s a
–q kq
=
+q sa
a 1 q
8. VS = ◊
S 4pe0 R
2q 1 3q
and VC = ◊
3a 4pe0 2 R
1 q
\ VC - VS = ◊
4pe0 2 R
2
1 È q2 ( - q) q2 4
Ui = ◊Í + + p dR 3 d R 2
4pe0 ÍÎ 2a 2 ¥ 2a 2 ¥ 3 a 1 3
= ◊ =
q ( - q) q ( q) q ( - q) ˘ 4 pe0 2R s E0
+ + + ˙
2a 3a 3a ˚ 9. As particle comes to rest, force must be
5a 2 repulsive, hence it is positively charged.
=
48pe0 a Again on moving down its KE first increases
than decreases, PE will first decrease than
When switch S is closed, entire charge flows increase.
to the outer surface of outer shell,
53
10. (1) is correct as the points having zero qE l
v=
potential are close to Q2, |Q2| < |Q1|. m
Again as potential near Q1 is positive, Q1 is At point B
positive, hence (2) is correct. mv2
T = qE + = 2 qE
At point A and B potential is zero not field, r
hence they may or may not be equilibrium
17. Velocity of particle at any instant
point.
Hence (3) is wrong.
V E
At point C potential is minimum, Q positive
charge placed at this point will have O (x, 0)
unstable equilibrium but a negative charge qE
will be in stable equilibrium at this position. V = at = t
m
Hence, (4) is wrong.
\ L = mvr = qE x0t
11. V1 is always negative and V2 is always Hence, angular momentum of the particle
positive. increases with time.
12. Electric field between the two points is 18. By work energy theorem
positive near q1 and negative near q2, hence
W = DK
q1 is positive and q2 is negative.
1
Again neutral point is closer to q1, hence fi mv2
q ( VS - VC ) = 0 -
2
q1 < q2.
Ê 1 Q 1 3Q ˆ 1
13. Electric field due to a conductor does not fi qÁ ◊ - ◊ ˜ = - mv2
Á 4pe R 4pe 2 R ˜ 2
depend on position of charge inside it. Ë 0 0 ¯
Æ Qq
^ ^
14. E = 400 cos 45∞ i + 4000 sin 45∞ j fi u=
4pe0mR
^ ^
= 200 2 ( i + j) 19. Potential at the centre of negatively charged
Æ Æ Æ Æ ring
V A - V B = - E ◊ rAB –Q +Q
^ ^ ^ ^ -2
= - 200 2 ( i + j) ◊ (2 j - 3 i ) ¥ 10 2R
= 2 2 V ª 2.8 V
15. Potential difference between two concentric
spherical shells does not depend on charge of R R
outer sphere. Hence,
÷3R
V A ¢ - VB ¢ = VA - VB 1 2
But VB ¢ = 0 1 Ê -Q Q ˆ
V1 = ◊Á + ˜
\ VA ¢ = VA - VB. 4pe0 Ë R 2R¯
16. By work energy theorem, Q
=-
Æ A 8pe0 R
E
Potential at the centre of positively charged
l ring
1 ÊQ Q ˆ
V2 = ◊Á - ˜
q
B
4pe0 Ë R 2 R ¯
l(1–cosq) Q
=
Work done by electric field 8pe0 R
= charge is KE Kinetic energy required = Work done required
1 Q
qE l (1 - cos q) = mv2 - 0 = q ( V2 - V1 ) =
2 4pe0 R
54
Vx 2 - Vx1 16 - 4 mv 2
20. Ex = - =- = 3 V/m 24. T - mg cos q - qE sin q =
x2 - x1 -2 -2 l
Vy 3 - Vy1 12 - 4
Ey = - =- = - 4 V/m
y3 - y1 4 -2 E
^ ^ ^ ^
\E = Ex i + Ey j = (3 i - 4 j) V/m.
q T
21. Consider a point P ( x, y)
q
qE
where potential is zero. mg
Now, VP
Tension will be minimum when velocity is
minimum.
P(x,y)
Minimum possible in the string is zero.
2Q mv2
Q ie, = - ( mg cos q + qE sin q)
(–3a,0) O (+3a,0) l
Diff. both sides w.r.t. q
2mv dv
= mg sin q - qE sin q …(i)
l dq
For minima or maxima
1 ÊÁ Q - 2Q ˆ
˜ dv qE
VP = + = 0 fi q = tan -1
4pe0 Á (3 a - x ) + y
2 2
( x + 3 a ) 2
+ y 2 ˜ dq mg
Ë ¯
=0 -1 qE
or p + tan
mg
fi ( x + 3 a )2 + y2 = 4 [(3 a - x )2 + y2 ]
fi 3 x 2 + 3 y2 - 30ax + 27a 2 = 0 Differentiating Eq. (i) again,
2
fi x 2 + y2 - 10ax + 9a 2 = 0 2mv d 2v 2m Ê dv ˆ
2
+ ◊ Á ˜ = mg sin q + qE sin q
The equation represents a circle with radius l dq l Ë dq ¯
2 d 2v qE
10a ˆ \ 2 = + ve for q = tan -1
= ÊÁ 2
˜ - 9a = 4a dq mg
Ë 2 ¯
-1 qE
10 and –ve for q = p + tan
and centre at ÊÁ a, 0ˆ˜ = (5a, 0) mg
Ë2 ¯
25. q A = s (4pa 2 ), q B = - s (4pb2 )
Clearly points x = a and x = 9a lie on this
circle. and qC = - s (4pc 2 )
1 Ê q A + q B qC ˆ
22. Work done = qEy = Charge in KE VB = ◊Á + ˜
4pe0 Ë b c ¯
1
Hence, K f = mv2 + qEy s È a2 ˘
2 = Í 2 - b + c˙
All other statements are correct. e0 ÍÎ b ˙˚
2 2 2
23. Electrostatic force of attraction provides 1 Èq -q q q2 q2 ˘
26. Ui = ◊Í - + - - ¥ 2˙
necessary centripetal force. 4pe0 ÍÎ a a a a 2a ˚˙
mv2 lq
ie, = - 2 q2
r 2pe0r =
4pe0a
lq
fi V= 1 q2
2pe0m Uf = ◊
4pe0 a
2pr 2pe0m m
T= = 2pr = 2pr 1 q2
V lq 2 K lq W = Uf - Ui = ◊ ( 2 + 1)
4pe0 a
55
27. q A = s (4pa 2 ), q B = - s (4pb2 ), qC = s (4pc 2 ) On dividing Eq. (ii) by Eq. (i),
Given, VA = VC Fe ¢ mg - FB
=
1 Ê q A q B qC ˆ 1 Ê q A + q B + qC ˆ Fe mg
◊Á + + ˜= ◊Á ˜
4pe0 Ë a b c ¯ 4pe0 Ë c ¯ 1 FB 0.8 1
=1 - =1 - =
a 2 - b2 K mg 1.6 2
fi a-b+ c= +c
c K =2
fi a+ b=c 1 Íq È q ˘
32. VP = ◊ ¥2 - ¥ 2˙
28. Potential at minimum at mid-point in the 4pe0 Í a a 2 + b2 ˙
region between two charges, and is always Î ˚
2q Í a + b - a ˘˙
È 2 2
positive.
= ◊
1 q2 4pe0 Í a a 2 + b2 ˙
29. Ui = ◊ =U Î ˚
4pe0 r
È Ê 2 1/ 2 ˘
ˆ
1 q2 Í a Á1 + b ˜ - a˙
Uf = ◊ ¥3 = 3 U 2q Í ÁË a 2 ˜¯ ˙ 2q b2
4pe0 r = = ◊ 3
Í ˙
4pe0 Í a a 2 + b2 ˙ 4pe0 a
\W = Uf - Ui = 2 U
Í ˙
30. Loss of KE = Gain in PE ÍÎ ˙˚
1 1 qQ [As b << a]
mv2 = ◊
2 4pe0 r 1 5q
33. In any case electric field at origin is ◊
1 4pe0 r 2
rµ 2
v 1 5q
along x-axis and ◊ along y-axis.
31. When the spheres are in air 4pe0 r 2
O 2
1 1 Ê 1 q ˆ
34. < u > = e0 E2 = e0 Á ◊ ˜
q 2 2 ÁË 4pe0 R2 ˜¯
T È 9 ¥ 109 ¥ 1 ¥ 10-9 ˘
Tcosq 1 Í 9 ˙
= e0 Í ˙
q
Tsinq 2 Í 12 ˙
Fe Fe ÍÎ ˙˚
e0 3
mg
mg
= J/m
2
T cos q = mg 35. If Q is initial charge on B
1 Q
T sin q = Fe then, VA - VB = ◊ =V
4pe0 b
\ Fe = mg tan q …(i)
When the spheres are immersed in liquid Now, if A is earthed, let charge q moves on A
from ground, then
O
Q
q q
T'
T'cosq T'
FB
q
F'e F'e
Tsinq
mg
mg
1 Ê q Qˆ
F1 ¢ = T ¢sin q VA = ◊Á + ˜ =0
4pe0 Ë a b¯
g - FB = T ¢cos q
a
\ Fe ¢ = ( mg - FB ) tan q …(ii) q=- Q
b
56
1 q+Q 42. Let charge q¢ flows through the switch to the
VB = ◊
4pe0 b ground, then
1 QÊ a a 1 È Q - q¢ Q ˘
= ◊ Á1 - ˆ˜ = V ÊÁ1 - ˆ˜ ◊Í - ˙ =0
4pe0 b Ë b¯ Ë b¯ 4pe0 ÍÎ r 2 r ˙˚
Æ Ê ∂v ^ ∂v ^ ∂v ^ ˆ 1
36. E = - ÁÁ i+ j+ k˜ q¢ = Q
Ë ∂x ∂y ∂z ˜¯ 2
Ê-2 ^ -2 ^ -2 ^ˆ 43. After n steps
=-Á i+ j+ k˜ 1 -1
Ë 1 1 1 ¯ q¢ = n Q and q = n - 1 Q
^ ^ ^
2 2
= 2 ( i + j + k ) N/C
1 Ê q¢ qˆ
\ VA = ◊Á + ˜ =0
If VP is potential at P, then 4pe0 ÁË r 2 r ˜¯
ÆÆ 1 q¢ + q
VP - V0 = - E ◊ r VB = ◊
4pe0 2 r
^ ^ ^ ^ ^ ^
VP - 10 = - 2( i + j + k ) ◊ ( i + j + k ) = - 6
1 È Q ˘
= Í ˙
VP = 4 V n+1
2 ÍÎ 4pe0r ˙˚
37. On touching two spheres, charge is equally
44. Consider a spherical Gaussian surface of
divided among them, then due to induction a
q radius r ( < R) and concentric with the sphere,
charge ÊÁ - ˆ˜ appears on the earthed sphere.
Ë 2 ¯
38. Negative charge will induce on the conductor
near P. O R
Ï r
Ô 0 for r < rA
Ô
Ô kQ P
39. Ì for rP < r < rB
Ô r
Ô k(Q A - Q B ) for r > r Charge on a small sphere of radius r
Ô r
B
Ó dq = d dV = 4pr 2d dr
As|Q B| > |Q A| Ê r3 ˆ
E is –ve for r > rB. = r pd0 Á r 2 - ˜ dr
Á R ˜¯
Ë
Æ Ê ∂v ^ ∂v ^ˆ
40. E = - ÁÁ i+ j˜ Total charge inside the Gaussian surface,
Ë ∂x ∂y ˜¯
r Ê r3 ˆ
q = 4p d0 Ú Á r 2 - ˜ dr
^
= k ( y i + x j)
^ 0 Á R ˜¯
Ë
Æ È r3 r4 ˘
|E| = k y2 + x 2 = kr = 4pd0 Í - ˙
ÍÎ 3 4 R ˙˚
41. Let charge on outer shell becomes q.
1 q d Èr r2 ˘
B \E = ◊ 2 = 0Í - ˙
4pe0 r e0 ÍÎ 3 4 R ˙˚
2r A 45. Total charge inside the surface.
S2
r S1 È R3 R3 ˘ 1 3
Q = 4pd0 Í - ˙ = pd0 R
ÍÎ 3 r ˙˚ 3
1 Q d R3
E= ◊ 2 = 0 2
4pe0 r 12e0r
1 ÊQ + qˆ Èr
d0 r2 ˘
VB = ◊Á ˜ =0 46. E = Í - ˙
4pe0 ÁË 2 r ˜¯ e0
ÍÎ 3 4 R ˙˚
q = -Q For maximum intensity of electric field
57
dE d0 È1 - r ˘ = 0 1 Ê Q + q1 ˆ
= Í3 2R˙ VB = ◊Á ˜ =0
dr e0 Î ˚ 4pe0 Ë b ¯
2 q1 = - Q
fi r= R
3
Now, if A is earthed
d 2E d
= - 0 = - ve, q1
dr 2 2 Re0
q2
2
hence E is maximum at r = R.
3
È 2l Ê 2 R ˆ 2 ˘
Í Á ˜ ˙
r 3 ¯ ˙ r0 R
47. Emax = 0 Í 3 - Ë =
e0 Í 3 4 R ˙ qe0
Í ˙
ÍÎ ˙˚
1 Ê q2 q1 ˆ
48. Potential difference between two concentric qA = ◊Á + ˜ =0
4pe0 Ë a b¯
spheres do not depend on the charge on
a a
outer sphere. q2 = - q1 = Q
b b
49. When outer sphere B is earthed
q1 50. When connected by conducting wires, entire
B
charge from inner sphere flows to the outer
Q sphere, ie,
A a
q3 = q1 + q2 ÊÁ - 1ˆ˜Q
Ëb ¯
a-b
= Q
b

More than One Correct Options


1. Before earthing the surface B, 2. For the motion of particle
1 Ê qA qB ˆ y
VA = ◊Á + ˜ =2V E
4pe0 Ë R 2 R ¯
1 Ê qA + qB ˆ 3 m
VB = ◊Á ˜= V
4pe0 Ë 2 R ¯ 2
qA 1 x
fi =
qB 2 qE
ux = 0, vx = v, a x = , a y = - g,
On earthing the sphere B, m
qA ¢ = qA x0 = 0, y0 = 0
1 qA ¢ + qB ¢ 1
VB = ◊ =0 x = x 0 + ux t + ax t 2
4pe0 2R 2
qE 2
qB ¢ = - qA ¢ x= t …(i)
2m
qA ¢ 1
fi = -1 y = y0 + u y t + a yt2
qB ¢ 2
As potential difference does not depend on 1 2
= ut - gt …(ii)
charge on outer sphere, 2
V At the end of motion
V A ¢ - VB ¢ = VA - VB =
2 t = T , y = 0, x = R
1
VA ¢ = V
2
58
From Eq. (ii), 4. For all charges to be in equilibrium, force
1 experienced by either charge must be zero
0 = ÊÁu - gT ˆ˜ T
Ë 2 ¯ ie., force due to other two charges must be
2u 2 ¥ 10 equal and opposite.
T= = =2s
g 10 q1 q2 q3
From Eq. (i),
Hence all the charges must be collinear,
qE 2
R= T charges q1, and q3 must have same sign and
2m q2 must have opposite sign, q2 must have
1 ¥ 10-3 ¥ 104 maximum magnitude.
= ¥ 4 = 10 m
2¥g Such on equilibrium is always unstable.
Now, v2y - u 2y = 2a y ( y - y0 ) 5. Flux through any closed surface depends
At highest point (i.e., y = H), vy = 0 only on charge inside the surface but electric
field at any point on the surface depends on
0 - (10)2 = - 2 ¥ 10 ( H - 0)
charges inside as well as outside the surface.
H =5m
Q2
3. Let R be the radius of the sphere
1 q
V1 = ◊
4pe0 R + r1 Q1
q ¥ 109 ¥ q
fi = 100 …(i)
( R + S ) ¥ 10-2
1 q
V2 = ◊
4pe0 R + r2 6. As net charge on an electric dipole is zero,
9 net flux through the sphere is zero.
9 ¥ 10 ¥ q
fi = 75 …(ii) But electric field at any point due to a dipole
( R + 10) ¥ 10-2
cannot be zero.
On solving, 7. Gauss’s law gives total electric field and flux
R = 10 cm, due to all charges.
5 50 8. If two concentric spheres carry equal and
q= ¥ 10-9 C = ¥ 10-10 C
3 3 opposite charges, Electric field is non-zero
Electric field on surface, only in the region between two sphere and
5 potential is is zero only outside both the
9 ¥ 109 ¥ ¥ 10-9
1 q 3 spheres.
E= ◊ =
4pe0 R2 (10 ¥ 10-2 ) 9. As force on the rod due to electric field is
= 1500 V/m towards right, force on the rod due to hinge
must be left.
Potential at surface,
5 The equilibrium is clearly neutral.
9 ¥ 109 ¥ ¥ 10-9
1 q 3 10. If moved along perpendicular bisector, for all
V= ◊
4pe0 R 10 ¥ 10-2 identical charges, electrostatic potential
energy is maximum at mid point and if
= 150 V
moved along the line joining the particles,
Potential at Centre electrostatic potential energy is minimum at
3 the mid-point.
VC = VS = 225 V
2
59

Match the Columns


1. (a Æ s), (b Æ q), (c Æ r), (d Æ p). 11
= V
If charge at B is removed 8
1 q
B C Vout = ◊
Æ 4pe0 r
E Æ
E
at r = 2 R
1 q V
A Æ D V2 = ◊ =
E 4pe0 2 R 2
Æ
E 1 qr
Ein = ◊ 3
4pe0 R
F E R
at r =
Enet = ED cos 30∞ + EE cos 30∞ 2
1 q V V
= 3E E1 = ◊ 2
= = (Q R = 1 m)
4pe0 2 R 2R 2
If charge at C is removed
1 q
Enet = ED cos 60∞ + Ef cos 60∞ Eout = ◊
4pe0 r 2
=E
at r = 2 R
If charge at D is removed 1 q V V
Æ Æ Æ E2 = ◊ 2
= =
E net = 0 and E B = - E E 4pe0 (2 R) 4R 4
Æ Æ \(a Æ s), (b Æ q), (c Æ q), (d Æ p).
and EE = - EE
4. (a Æ r), (b Æ q), (c Æ q), (d Æ s)
If charge at B and C both are removed,
5. (a Æ p), (b Æ q), (c Æ r), (d Æ s)
Enet = EE + ED cos 60∞ + EF cos 60∞
For a spherical shell,
= 2E Ï 0 for r < R
2. (a Æ q), (b Æ p), (c Æ s), (d Æ r). E = ÔÌ Kq
ÆÆ Ô r 2 for r ≥ R
Ó
V = - E◊ r
Ï Kq
Æ Ô for r £ R
V = ÔÌ R
^
If r = 4 i, V = - 8 V,
Æ Ô Kq for r ≥ R
^ Ô r
If r = -4i, V = 8V Ó
Æ ^ For a solid sphere,
If r = 4 j, V = - 16 V, Ï Kqr
Æ Ô 3 for r £ R
If
^
r = - 4 j, V = 16 V E = ÔÌ R
Ô Kq for r ≥ R
3. For a solid sphere Ô r2
Ó
1 q Ï Kq
Vin = ◊ (3 R2 - r 2 ) 2 2
Ô 2 (2 R - r ) for r £ R
4pe0 2 R3 Ô 2 R
V =Ì
R Ô Kq
at r= Ô for r ≥ R
2 Ó r2
1 q Ê R2 ˆ
V1 = ◊ Á 3 R2 -
3 Á
˜
4pe0 2 R Ë 4 ˜¯
22 Capacitors
Introductory Exercise 22.1
q [ AT ]
1. C = = [C ] =
V [ML2 T -3A -1 ] 3mC 7mC –7mC 3mC
-1 -2 4 2
= [M L T A ]
2. False.
Charge will flow if there is potential
difference between the conductors. It does
not depend on amount of charge present.
3. Consider the charge distribution shown in
figure. Hence, if q1 and q2 be charge on two plates
10–q q –q (q – 4) then.
1 2 3 4 q1 q2 q3 q4

E4 E3
E2 P E1

Electric field at point P


EP = E1 + E3 - E2 - E4 q1 + q2
q1 = q4 = = 3 mC
10 - q q q q -4 2
= - + - q - q2
2e0 A 2e0 A 2e0 A 2e0 A q2 = 1 = 7 mC
2
But P lies inside conductor q - q1
q3 = 2 = - 7 mC
\ EP = 0 2
fi 10 - q - q + q - q + 4 = 0 q1 q2 q3 q4
fi q = 7 mC
Hence, the charge distribution is shown in
figure.

Sort-cut Method
Entire charge resides on outer surface of
conductor and will be divided equally on two
outer surfaces.
61
4. Charge distribution is shown in figure. 5q
q=
q + q2 q 2
q1 = q4 = 1 =- e A
2 2 and C= 0
q - q2 5q d
q2 = 1 = q 5q d
2 2 \ V= =
q2 - q1 5q C 2e0 A
q3 = =-
2 2
\ Charge on capacitor = Charge on inner
side of positive plate.

Introductory Exercise 22.2


1. All the capacitors are in parallel 2
q = CeV = ¥ 1200 = 800 mC
3
C1 C2
1mF 10V 2mF 3mF

C1 C2 C3 V1 V2

q1 = C1V = 1 ¥ 10 = 10 mC
q2 = C2V = 2 ¥ 10 = 20 mC V
q3 = C3V = 3 ¥ 10 = 30 mC
q 800
2. Potential difference across the plates of (b) V1 = = = 800 V
C1 1
capacitor
q 800
V = 10 V V2 = = = 400 V
C2 2
q = CV = 4 ¥ 10 = 40 mC
C1,V1
3. In the steady state capacitor behaves as
open circuit.
2W

4W C2,V2
A B
I
Now, if they are connected in parallel,
I
Ê C V + C2V2 ˆ
Common potential, Á V = 1 1 ˜
Á C1 + C2 ˜¯
6W 30V Ë
1 ¥ 800 ¥ 2 ¥ 400 1600
30 = = V
I= =3 A 1+2 3
6+4
1600 3200
Potential difference across the capacitor, q1 = C1V = mC, q2 = C2V = mC
3 3
VAB = 4 ¥ I = 4 ¥ 3 = 12 V 5. Common potential
\Charge on capacitor C V + C2V2
V= 1 2
q = CVAB = 2 ¥ 12 = 24 mC C1 + C2
1 1 1 1 1
4. (a) = + = + But V = 20, V2 = 0, V1 = 100 V, C1 = 100 mC
Ce C1 C2 1 2
100 ¥ 100 + C2 ¥ 0
2 \ = 20
Ce = mC 400 + C2
3
fi C2 = 400 mC
62

Introductory Exercise 22.3


1. Let q be the final charge on the capacitor, q0
fi q= (1 - e - t / t )
2
work done by battery
C
\At time t,
R q0
Charge on C1 = q = (1 - e - t / t )
2
q
Charge on C2 = q0 - q = 0 (1 + e -t / t )
2
4. Let q be the charge on capacitor at any
V instant t
C R
W = qV +q –q

Energy stored in the capacitor


1 I
U = qV
2
\Energy dissipated as heat
1 E
H = U - W = qV = U
2 By Kirchhoff’s voltage law
2. We have q
+ IR = E
I = I0 e - t / t C
dq CE - q
I0 1 =
= I0 e - t / t fi e - t / t = dt RC
2 2 q dq t dt
t = t ln 2 = 0.693 t Úq0 CE - q = Ú0 RC
t = 0.693 time constant.
fi q0 = CE (1 - e - t / t ) + q0e - t / t
3. Let capacitor C1 is initially charged and C2 is
uncharged. where, t = RC
q0 –q R 5. (a) When the switch is just closed,
Capacitors behave like short circuit.
C1–(q0–q) E S
I

q –q C1

C2 R2
C2
At any instant, let charge on C2 be q, charge
on C1 at that instant = q0 - q
R1 R3
By Kirchhoff’s voltage law,
( q0 - q) q \Initial current
- IR - = 0 E
C C Ii =
dq q0 - 2q R1
fi =
dt RC (b) After a long time, i.e., in steady state, both
q dq t dt
the capacitors behaves open circuit,
fi Ú 0 q0 - 2q = Ú 0 RC E
If =
[ln ( q0 - 2q)]q0 1 R1 + R3
fi = [ t ]t0
-2 RC 6. (a) Immediately after closing the switch,
capacitor behaves as short circuit,
63
A I2 (c) Potential difference across the capacitors in
I1 the steady state,
I V =E
E R1 R2 \Energy stored in the capacitor
1
U = CE2
2
C
(d) After the switch is open
S B
E E R2
\ I1 = and I2 = E R1
R1 R2
C
(b) In the steady state, capacitor behaves as
open circuit,
E Re = R1 + R2
\ I1 = , I2 = 0 t = R3C = ( R1 + R2 ) C
R1

AIEEE Corner
Subjective Questions (Level-1)
e0 A Cd 1 ¥ 1 ¥ 10
-3 A
1. C = fiA= = k3e0
d e0 8.85 ¥ 10-12 C3 = 2 = k3e0 A
2d / 2 2d
= 1.13 ¥ 108 m 2
e0 A1 e0 A2 Therefore, the effective capacitance,
2. C1 = and C2 = C2C3
d d C = C1 +
If connected in parallel C2 + C3
e A e A e0 A Ê k1 k1k3 ˆ
C = C1 + C2 = 0 1 + 2 2 = Á + ˜
d d Á
2d Ë 2 k2 + k3 ˜¯
e0( A1 + A2 ) e0 A
= = 4. (a) Let the spheres A and B carry charges q
d d
and - q respectively,
where, A = A1 + A2 = effective area. q –q
Hence proved.
3. The arrangement can be considered as the a b
combination of three different capacitors as
shown in figure, where d
A B
C2 1 È q q˘
\ VA = ◊ -
4pe0 ÍÎ a d ˙˚
C1
1 È q q˘
VB = ◊ - +
C3 4pe0 ÎÍ b d ˚˙
Potential difference between the spheres,
A q È1 1 2˘
V = VA - VB = + -
k1e0
2 = k1e0 A 4pe0 ÍÎ a b d ˙˚
C1 =
2d 4d q 4pe0
C= =
A V 1 + 1-2
k2e0
C2 = 2 = k2e0 A a b d
2d / 2 2d Hence proved.
64
(b) If d Æ • Let effective capacitance between A and B
4pe0 4pe0 ◊ ab
= C= C AB = x
1 1 a+b
+ As the network is infinite,
a b
C PQ = C AB = x
If two isolated spheres of radii a and b are
connected in series, Equivalent circuit is shown in figure,
then, 2C
C1C2 A P
C¢ =
C1 + C2
C x
where, C1 = 4pe0a, C2 = 4pe0b
4pe0 ◊ ab Q
\ C¢ = B
a+b
2Cx
RAB = C + =x
\ C¢ = C 2C + x
Hence proved.
fi 2C 2 + Cx + 2Cx = 2Cx + x 2
5. (a) fi x 2 - Cx - 2C 2 = 0
B
A On solving, x = 2C or - C
C
C But x cannot be negative,
C C C
Hence, x = 2C
A B
C 6. q = CV = 7.28 ¥ 25 = 182 mC
-6
C q 0.148 ¥ 10
7. (a) V = = -12
= 604 V
C 2C C 245 ¥ 10
C
A B e A Cd
(b) C = 0 fi A =
d e0
C
5C/3 2C/3 245 ¥ 10-12 ¥ 0.328 ¥ 10-3
=
A B
A B
8.85 ¥ 10-12
= 9.08 ¥ 10-3 m 2
C
= 90.8 cm 2
(b) q 0.148 ¥ 10
-6
C (c) s = = = 16.3 mC/m 2
A 9.08 ¥ 10-3
C C C C
A A B 8. (a) E0 = 3.20 ¥ 105 V/m
B
C C C E = 2.50 ¥ 105 V/m
E 3.20 ¥ 105
C k= 0 = = 1.28
E 2.50 ¥ 105
4C/3 (b) Electric field between the plates of
A B A B
C/3 capacitor is given by
s
(c) E=
e0
2C 2C 2C 2C
A
P fi s = e0E = 8.8 ¥ 10-12 ¥ 3.20 ¥ 105
= 2.832 ¥ 10-6 C/m 2
C C C C •
= 2.832 mC/m 2
B 9. (a) q1 = C1V = 4 ¥ 660 = 2640 mC
Q
q2 = C2V = 6 ¥ 660 = 3960 mC
65
As C1 and C2 are connected in parallel, For series combination,
V1 = V2 = V = 660 V US = 1.6 ¥ 10-2 J = 0.016 J
1
C1 = 4.00mF US = C S V 2
2
2US 2 ¥ 0.016
fi CS = 2 = = 0.008 F
C2 = 6.00mF V (2)2
= 8 mF
Now, C P = C1 + C2 = 5 mF
or C2 = (5 - C1 ) mF
660 V 1 1 1 1
and = + =
(b) When unlike plates of capacitors are CS C1 C2 8
connected to each other, 1 1 1
fi + =
Common potential C1 S - C1 8
C V - C1V1 6 ¥ 660 - 4 ¥ 660
V= 2 2 = On solving,
C1 + C2 6
C1 = 40 mF, C2 = 10 mF or vice-versa.
= 220 V 13. In the given circuit,
q1 = C1V = 4 ¥ 220 = 880 mC q q
VA - VB = -E + =5
q2 = C2V = 6 ¥ 220 = 1320 mC C1 C2
V 400
10. E = = = 8 ¥ 104 V/m +q
C1
–q
E
+q
C2
–q
d 5 ¥ 10-3 A B
Energy density, q q
1 1 fi - 10 + =5
u = e0E2 = ¥ 8.85 ¥ 10-12 ¥ ( 8 ¥ 10-4 )2 10-6 2 ¥ 10-6
2 2
fi q = 10 ¥ 10-6 C = 10 mC
= 2.03 ¥ 10-2 J/m 3 q q
\ V1 = = 10 V, V2 = =5V
= 20.3 mJ/m 3 C1 C2
11. Dielectric strength = maximum possible 14. (a) In order to increase voltage range n
electric field times, n-capacitors must be connected in
V V series.
E= fid=
d E Hence, to increase voltage range to 500V,
5500 5 capacitors must be connected in series.
= = 3.4 ¥ 10-4 m Now, effective capacitance of series
1.6 ¥ 107
combination,
ke A Cd 10
C= 0 fiA= CS = Cn = = 2 pF
d ke0 5
1.25 ¥ 10-9 ¥ 3.4 ¥ 10-4 Hence, no parallel grouping of such units is
=
3.6 ¥ 8.85 ¥ 10-12 required.
Hence, a series grouping of 5 such
= 1.3 ¥ 10-2 m 2 capacitors will have effective capacitance
= 0.013 m 2 2 pF and can withstand 500 V.
12. Let C P and CS be the effective capacitance of (b) If n capacitors are connected in series and
parallel and series combination respectively. m such units are connected in parallel,
Ve = nV
For parallel combination,
mC
U P = 0.19 J Ce =
n
1
U P = C PV 2 Here, V = 100 V
2
2U 2 ¥ 0.1 Ve = 300 V
fi C P = 2P = = 0.05 F V
V (2)2 \ n = e =3
V
= 50 mF
66
C = 10 pF C1 + 2 90
fi = =9
Ce = 20 pF C2 10
nCe 3 ¥ 20 fi C1 + 2 = 9C2
m= = =6
C 10 3
fi C1 + 2 = 9 ¥ C1
Hence, the required arrangement is shown 2
in figure. 25 4
fi C1 = 2 fi C1 = mF
2 25
= 0.16 mF
3
C2 = C1 = 0.24 mF
2
16. (a) q = CV = 10 ¥ 12 = 120 mC
e A
(b) C = 0
d
If separation is doubled, capacitance will
become half. i.e.,
C
C¢ =
2
15. Case I. C
C2 q¢ = E¢ V = V = 60 mC
V1 = V = 60 V 2
C1 + C2
e0 A pe0r 2
A B
(c) C = =
d d
If r is doubled, C will become four times, i.e.,
V1 V2
C ¢ = 4C
q¢ = C ¢ V = 480 mC
17. Heat produced = Energy stored in the
100 V
capacitor
1 1
C1 H = CV 2 = ¥ 450 ¥ 10-6 ¥ (295)2
V2 = V = 40 V 2 2
C1 + C2
= 19.58 J
C1 2 e A 8.85 ¥ 10-12 ¥ 2
fi = 18. (a) C = 0 =
C2 3 d 5 ¥ 10-3
3
fi C2 = C1 = 3.54 ¥ 10-6 F
2
= 3.54 mF
Case II.
C2 (b) q = CV = 3.54 ¥ 10-9 ¥ 10000
V1 = = 10 V = 35.4 ¥ 10-6 = 35.4 mC
C1 + C2 + 2
V 10000
2mF
(c) E = = = 2 ¥ 106 V/m
d 5 ¥ 10-3
19. Given,
C1 C2 C3
A B
V1 V2

100 V
C1 + 2 V
V2 = = 90 V
C1 + C2 + C C1 = 8.4 mF, C2 = 8.2 mF
C3 = 4.2 mF, V = 36 V
67
(a) Effective capacitance, (Charge is shown in mC).
1 1 1 1 Hence, charge on 6 mF capacitor = 10 mC
= + +
Ce C1 C2 C3 40
and Charge on 4 mF capacitor = mC
1 1 1 3
= + + fi Ce = 2.09 mF
8.4 8.2 4.2 21. (a)
C1=8.4mF C3=4.2mF
q = CeV = 2.09 ¥ 36 = 75.2 mC
a
As combination is series, charge on each
capacitor is same, i.e., 75.2 mC. C2=4.2mF
1 1
(b) U = qV = ¥ 75.2 ¥ 36 ¥ 10-6 b
2 2 C5=8.4mF C4=4.2mF
= 1.35 ¥ 10-3 J = 1.35 mJ 8.4mF
a
(c) Common potential,
C V + C2V2 + C3V3 4.2mF 2.1mF
V= 1 1 = 10.85 V
C1 + C2 + C3 b
1 8.4mF
(d) U ¢ = (C1 + C2 + C3 )V 2
2 8.4mF
1 a
= ¥ ( 8.4 + 8.2 + 4.2) ¥ (10.85)2 ¥ 10-6 2.52mF
2 a b 6.3mF
= 1.22 ¥ 10-3 J = 1.22 mJ
b
20. The Given circuit can be considered as the 8.4mF
sum of three circuits as shown
(b) Charge supplied by the source of emf
3mF
+12 –12 q = CV = 2.52 ¥ 10-6 ¥ 220
= 554.4 mC
+6 +2 +4
5V 6mF q1 = q5 = q = 554.4 mC
2mF 4mF 4.2
–2
q2 = q
–6 –4 4.2 + 2.1
3mF 4.2
+12 –12 = ¥ 554.4 mC = 369.6 mC
6.3
–24 +12 +24 2.1 2.1
and q3 = q4 = q= ¥ 554.4 mC
+ 6mF 2mF 4.2 + 2.1 6.3
+24 –12 –24 4mF = 184.8 mC
q1 554.4
10V V1 = = = 66 V = V5
C1 8.4
+
3mF q 369.6
–4 +4
5V V2 = 2 = = 88 V
C2 4.2
4
–4
— q 184.8
+8+8/3 3 V3 = V4 = 3 = = 44 V
+ –8/3 4mF C3 4.2
–8 +4 4
6mF 2mF —
3 22. Let C1 and C2 be the capacitances of A and B
respectively.
ke A ke A
3mF \ C1 = 1 0 1 , C2 = 1 0 2
5V d1 d2
+20mC –20mC
+50 +40 C2
–10mC — mC — mC Now, V1 = V
3 3 C1 + C2
5V 6mF 4mF
2mF –50 C2
–40 130 13
+10mC — mC — mC fi = = …(i)
3 3 C1 + C2 230 23
10V
68
C1 C2=2mF
V2 = V
C1 + C2
4mF
C1 10
fi = …(ii) 2mF
C1 + C2 23 C3=4mF
+
From Eqs. (i) and (ii), 20V
C1 10
= 3mF
C2 13
If dielectric slab of C1 is replaced by one for
which k = 5 then, C1=3mF
5e A 5
C1 ¢ = 0 1 = C1
d1 2
V2 ¢ C1 ¢ 5C1 50 2mF
\ = = =
V1 ¢ C2 2C2 26 2mF 2mF
50
V2 ¢ + V1 ¢ = 230 20V
26
Also, 3mF 3mF
V1 ¢ + V2 ¢ = 230
50
V1 ¢ = V1 ¢
26
V1 ¢ = 78.68 V
6mF
and V2 = 151.32 V
20V 20V 3mF
23. In this case 6mF
C1,V1

(b) q = CV = 3 ¥ 20 = 60 mC
(c) Potential difference across C1
6
V1 = ¥ 20 = 10 V
6+6
C2 C3
q1 = C1V1 = 3 ¥ 10 = 30 mC
Common potential, (d) Potential difference across C2
C1V1 6
V= V2 = ¥ 20 = 10 V
C2C3
C1 + 6+6
C2 + C3
1 ¥ 110 q2 = C2V2 = 2 ¥ 10 = 20 mC
V=
1 + 1.2
110 (e) Potential difference across C3
= 4
2.2 V3 = ¥ V2 = 5 V
4+4
= 50 V
Charge flown through connecting wires, q3 = C3V3 = 4 ¥ 2 = 20 mC
C2C3 25. (a) When switch S2 is open, C1 and C3 are in
= V
C2 + C3 series, C2 and C4 are in series their
effective capacitances are in parallel with
= 1.2 ¥ 50 each other.
= 60 mC Hence,
24. (a) Hence, effective capacitance across the C1C3
q1 = q3 = V
battery is 3 mF. C1 + C3
69
1 ¥3 C2C3Q
= ¥ 12 = 9 mC q=
1+3 C1C2 + C2C3 + C3C1
C2C4 C1C2C3V
q2 = q4 = =
C2 + C4 C1C2 + C2C3 + C3C1
2 ¥4 C12 (C2 + C3 ) V
= ¥ 12 = 16 mC \ q1 = Q - q =
2+4 C1C2 + C2C3 + C3C1
(b) When S2 is closed, C1 is in parallel with C1C2C3V
q2 = q3 = q =
C2 and C3 is in parallel with C4. C1C2 + C2C3 + C3C1
C1 C3 e0 A e AV
27. C = , q = CV = 0
d d
S2 e0 A
(a) C ¢ = , q¢ = q
2d
C2 C4 (As battery is disconnected)
V1 V2 q¢
V¢ = =2V

B
1 2 e0 AV 2
(b) Vi = V =
Therefore, 2C 2d
C3 + C4 1 1 e A
V1 = V2 = V Uf = C ¢ V ¢2 = ◊ 0 (2V )2
C1 + C2 + C3 + C4 2 2 2d
7 e AV 2
= ¥ 12 = 8.4 V = 0
10 d
C1 + C2 e0 AV 2
V3 = V4 = V (c) W = Uf - Ui =
C1 + C2 + C3 + C4 2d
3 28. In the steady state, capacitor behaves as
= ¥ 12 = 3.6 V
10 open circuit,
q1 = C1V1 = 1 ¥ 8.4 = 8.4 mC I1 R1 P
q2 = C2q2 = 2 ¥ 8.4 = 16.8 mC
I2 IC
q3 = C3V3 = 3 ¥ 3.6 = 10.8 mC A S B
q4 = C4V4 = 4 ¥ 3.6 = 14.4 mC C
26. Initial charge on C1 E1 E2 R2

Q = C1V0
Now, if switch S is thrown to right.
D
Let charge q flows from C1 to C2 and C3.
E1
By Kirchhoff’s voltage law, I1 = I2 = = 1 mA and IC = 0
R1 + R2
q
E1 R2
Q–q
C2 VPD = I2 R2 =
–q R1 + R2
C1
–(Q–q) q When switch is shifted to B,
C3
–q At this instant,
E1 R2
q q Q-q VPD =
+ - =0 R1 + R2
C2 C3 C1 V E1
Ê 1 I2 = PD = = 1 mA
1 1 ˆ Q R2 R1 + R2
q ÁÁ + + ˜=
˜
Ë C1 C2 C3 ¯ C1
70
E1 R2 q2 = C2V = 3 ¥ 18 = 54 mC
E2 +
E + VPD R1 + R2 After closing the switch,
I1 = 2 =
R1 R1 q1 ¢ = C1V1 = 6 ¥ 12 = 72 mC
( R1 + R2 ) E2 + E1 R2
= q2 ¢ = C2V2 = 3 ¥ 6 = 18 mC
R1
Dq1 = 18 mC, Dq2 = - 36 mC
= 2 mA V 18
30. (a) I = = =2A
IC = I1 + I2 = 1 + 2 = 3 mA R1 + R2 9
29. (a) When switch S is open, no current pass V=18.0V
through the circuit,
I C1= 6mF
V=18.0V
+
R1 = 6W q

S
R1 C1 a b

+ C2= 3mF
a b R2 = 3W q
S –

C2 R2

C1C2
q= V = 2 ¥ 18 = 36 mC
C1 + C2
Hence, Now, Va - 0 = IR2 fi Va = 6 V
Vb - 0 = 0 q 36
and Vb - 0 = = = 12 V
Vb = 0 C2 3
18 - Va = 0 fi Va = 18 V Va - Vb = - 6 V
fi Va - Vb = 18 V (b) b is at higher potential.
(b) a is at higher potential. (c) When switch S is closed, in steady state,
(c) When switch S is closed, V =18V
V=18.0V
I
R1 q1 + C1

C1
R1 a S b
I
a b
S +
R2 q2 C2
R2 –
C2
I

Va - vb = 6 V
V
I= =2A q1 = C1V1 = 6 ¥ 12 = 72 mC
R1 + R2
q2 = C2V2 = 3 ¥ 6 = 18 mC
Vb - 0 = IR2 = 2 ¥ 3 Charge flown through S
Vb = 6 V = q1 - q2 = 72 - 18 = 54 mC
(d) q1 = C1V = 6 ¥ 18 = 108 mC
71
31. E1 R2 + E2 R1 V
Ee = =
I A R F R1 + R2 2
E
I–I1 R 3R
I1 Re = R + =
2 2
V R q C
q = q0 (1 - e - t / t )
CE
q0 =
C G 2
R B
(a) Consider the circuit as combination of two 3 RC
t=
cells of emf E and OV. 2
C CE
F
R
D
\ q= (1 - e -2t / 3 RC )
2
dq E -2t / 3 RC
I1 (b) I1 = = e
I2 R dt 3 R
A B q
I
In loop EDBA + I1 R - I2 R = 0
C
I q
I2 = + I1
E F RC
E E -2t / 3 RC
R V = (1 - e -2t / 3 RC ) + e
2R 3R
E
C 3R/2
= (3 - e -2t / 3 RC )
6R

V/2

Objective Questions (Level 1)


2
Q 5. Incorrect diagram.
1. F = is independent of d.
2e0 A 6. Charge on capacitor of capacitance
q 2C
2. C = A M A
V
On connecting the plates V becomes zero. C
60V C C 60V C —
3. The system can assumed to a parallel 2
combination of two spherical conductors. B N B
C = C1 + C2 = 4pe0a + 4pe0b 2C
= 4pe0 ( a + b) C C
q = V = 30 C
4. V = 2 2
C q
VMN = = 30 V
On connecting in series C
q¢ = q = Charge on any capacitor 7. For equilibrium,
C qE = mg
C¢ = qE
n V 4
nq q = pr 3rg
\ V¢ = = nV d 3
C r3
Vµ mg
q
72
3
V2 Ê r2 ˆ q R
= ÁÁ ˜˜ ¥ 1
V1 Ë r1 ¯ q2
R R R
fi V2 = 4 V
8. Electric field between the plates is uniform
R
but in all other regions it is zero.
9. Initially the capacitor offers zero resistance.
1W
E
E 4E
12V I= =
6W 3
R + R 7R
4
3W
But potential difference across capacitor,
1W V = IR
4A 4E
fi 10 = R
7R
12V 2W
fi E = 17.5 V
12. As all the capacitors are connected in series
potential difference across each capacitor is
12
i= =4 A E 10
1+2 V= = = 2.5 V
4 4
10. q = CV = CE VA - VN = 3V = 7.5 V
11. In the steady state, capacitor behaves as VA = 7.5 V
open circuit. the equivalent diagram is given VN - VB = 2.5 V
by
VB = - 2.5 V
3
—R
4 R 13. Heat produced = Loss of energy
C1C2
= ( V1 - V2 )2
I 2 (C1 + C2 )
2 ¥ 10-6 ¥ 2 ¥ 10-6
= (100 - 0)
E 2 (2 + 2) ¥ 10-6

3R = 5 ¥ 10-3 J = 5 mJ
14. q = q0e - t / h
I = I0 e - t / h
P = I 2 R = I02e -2t / h R = P0e -2t / h
h
R R fi h¢ =
2
C V + C2V2 E
15. Common potential = 1 1 =
C1 + C2 2
E
9
16. VA - VB = 6 + 3 ¥ 2 - + 3 ¥ 3 = 12 V
1
73
17. In the steady state, current through battery C C C
R
C/2
S
C
12V A B
I 2W

C
2mF
4W
A B C C C 3C/2 C
O
A B

I 6W
3C/14
12 3
I= = A
6+2 2
22.
Potential difference across the capacitor,
3 1mF 1mF
VAB = 6 ¥ = 9 N
2
\ q = CVAB = 2 ¥ 9 = 18 mC
x 1mF y
18. C2 and C3 are in parallel
Hence, V2 = V3
Again Kirchhoff’s junction rule 2mF
- q1 + q2 + q3 = 0
fi q1 = q2 + q3 2mF 1mF
19. For the motion of electron
mu 2 sin 2q
R= =l …(i) x y
eE
mu 2 sin 2 q
and H= =d …(ii)
2eE 2mF
2
Dividing Eq. (ii) by Eq. (i), — mF
3
4d
tan q =
l
V 2Ve0 x y x y
20. V = Ed fi d = =
E 6 2mF 8
— mF
2 ¥ 5 ¥ 8.85 ¥ 10-12 3
=
10-7
k1e0 A k2e0 A
= 8.85 ¥ 10-4 = 0.88 mm 23. C1 = +
2d 2d
21.
( k1 + k2 ) e0 A
P Q R S = (Parallel grouping)
A B 2d
1 d d
= + (Series grouping)
C2 2k1e0 A 2k2e0 A
2k1k2 e0 A
C2 =
P and Q are at same potential, hence k1 + k2 d
capacitor connected between them have no
C1 ( k1 + k2 )2 (2 + 3 )2 25
effect on equivalent capacitance. = = =
C2 4k1k2 4 ¥ 2 ¥ 3 24
74
24. 1
C d
A A 2
C C A
P C d
Q P Q C 3
C CC C
C
B C C d
B
4
A A 2d
C C
C C 5
— — C C
2 2 d
C C B
B B 6
A
Capacitance of all other capacitance is same,
2C e A
B i.e., C = 0 but that of formed by plates 4
d
25. Cases (a), (b) and (c) are balanced C
and 5 is as distance between these two
Wheatstone bridge. 2
plates is 2d.
26. The given arrangement can be considered as
the combination of three capacitors as shown The equivalent circuit is shown in figure.
in figure. —
C
C C
A 2 1 5 24 4 3

5 6
C2
C1 2 C
A B
3
C3 C
C
C —
B 3

k1e0 A
Hence, C1 =
2d
C
A A B
k2e0
C2 = 2 = k2e0 A C
d/2 d 4C 4C
— —
A 7 C 3
k3e0
C3 = 2 = k3e0 A
d/2 d A B A B
C C
Effective capacitance,
C2C3 e A Èk k2k3 ˘
C = C1 + = 0 Í 1 + ˙ 11C
C1 + C2 d Î2 k2 + k3 ˚ —
7
27. Here, plate 1 is connected to plate 5 and
11 11e0 A 11
plate 3 is connected to plate 6. \Ceq = C= = ¥ 7 mF = 11 mF
7 7d 7
75

JEE Corner
Assertion and Reason
q 6. As potential difference across both the
1. Capacitance = is constant for a given
V capacitors is same, charge will not flow
capacitor. through the switch.
2. Reason correctly explains the assertion. 7. C and R2 are shorted.
1
3. U = qV , W = qV 8. Time constant for the circuit,
2
t = RC
4. For discharging of capacitor
9. In series, charge remains same
q = q0e - t / t
q2 1
dq q and U= fiU µ
= - 0 e -t / t 2C C
dt t
q 10. In series charge remains same
= - 0 e -t / t q q
RC \ V1 = , V2 =
C1 C2
Hence, more is the resistance, less will be
the slope. On inserting dielectric slab between the
5. Charge on two capacitors will be same only if plates of the capacitor, C2 increases and
both the capacitors are initially uncharged. hence, V2 decreases. So more charge flows to
C2.

Objective Questions (Level 2)


Ï 4Q ^ But, I01 = I02
Ô- i for x < d
V1 V2
Ô e0 A fi =
Ô 2Q ^ R1 R2
1. E = Ì- i for d < x < 2d
Ô e0 A 1 1
Also, >
Ô 4Q ^ R1C1 R2C2
Ô e Ai for 2d < x < 3 d
Ó 0 fi R2C2 > R1C1
2. Let E0 = external electric field As only two parameters can be different,
and E = electric field due to sheet C1 = C2
\ E1 = E0 - E = 8 R2 > R1
E2 = E0 + E = 12 and V2 > V1
s
fi E = 2 V/m fi =2 5. Charge on capacitor at the given instant.
2e0 q CE
q= 0 =
s = 4e0 2 2
3. When the switch is just closed, capacitors Heat produced = Energy stored in capacitor
behave like short circuit, no current pass q2 CE2
= =
through either 6 W or 5 W resistor. 2C 8
4. For charging of capacitor Heat liberated inside the battery,
I = I0 e - t / t r
= ¥ Total heat produced
t r + 2r
ln I = log I0 -
t CE2
V t =
ln I = ln - 24
R RC
76
6. Capacitor is not inside any loop.
E - E0 12. VA = - VB
7. I = ie, VA - VP = VP - VB [VP = 0]
R + R0
q q
q fi =
VBA = - E C123 Cn
C
E fi Cn = C123
R I 1 1 1 1
fi = + +
Cn C1 C2 C3
E +q C –q 13. When connected with reverse polarity
B A C1C2
H= ( V1 + V2 )2
2 (C1 + C2 )
C ¥ 2C 25
= ¥ ( V + 4V )2 = CV 2
E0 R0 2 (C + 2C ) 3
q H1 R2 R
- E + IR = -E 14. = =
C H2 R1 S
( E - E0 ) RC 2mF
q = IRC +
R + R0
C1C2 5W
8. C =
C1 + C2 I1
e A R
C1 = C2 = 0
d I2
e0 A
C= 1 1
2d Also, H1 + H2 = CV 2 = ¥ 2 ¥ 10-6 ¥ (5)2
2 2
2e0 A e A
C1 ¢ = , C2 ¢ = 0
d 2d H1 + H2 = 25 mJ
C1 ¢ C2 ¢ 2e0 A H2 = 25 - 10 = 15 mJ
C¢ = = <C
C1 ¢ + C2 ¢ 5d 10 R 10
= fi R= W
R 15 5 3
9. Re =
3 15. When current in the resistor is 1 A.
RC q
t = ReC = IR + =E
3 C
-t / t q
q = q0(1 - e ) 1 ¥ 5 + = 10
2
= CV (1 - e -3t / RC )
C1C2 fi q = 10 mC
10. Energy loss = ( V1 - V2 )2 When the switch is shifted to position 2. In
2 (C1 + C2 )
steady state, charge on capacitor
2 ¥4
= (100 - 50)2 ¥ 10-6 q¢ = 5 ¥ 2 = 10 mC
2 ¥ (2 - 14)
but with opposite polarity.
= 1.7 ¥ 10-3 J \Total charge flown through 5 V battery,
- t / RC
11. q = q0e = q + q¢ = 20 mC
dq q
I=- = 0 e - t / RC Work done by the battery = 20 ¥ 5
dt RC
= 100 mJ
at t = 0
q0 Heat produced = W - DU
I= = 10
RC But, DU = 0
q \ H = W = 100 mJ
V0 = 0 = 10 ¥ R = 10 ¥ 10 = 100 V
C
77
q q q
16. VA - VB = - + - =0 2.5 ¥ 0.5
6 2 3 q2 = ¥ 30
2mF 2.5 + 0.5
+ –
= 12.5 mC
q q
R1
\ Vp - Va = 1 = 18 V
C1
q q
+ – + – q
Vp - Vb = 2
6mF
A B
3mF C3
12.5
Hence, no charge will flow from A to B. = =5 V
2.5
17. As potential difference across both the Vb - Va = 13 V
capacitors is same, they are in parallel.
23. As all the capacitors are identical, potential
Hence, effective capacitance, difference across each capacitor,
2e A
C= 0 12V B
d + –
1 e A
U = CV 2 = 0 V 2
2 d C
18. Rate of charging decreases as it just charged.
19. Potential difference across capacitor = 6 V A N
C C C
A 1W B 2W
I
E
5V 6V V= =3 V
2V 4W 2mF 4
VN - VB = 3 V
D C 3W VB = - 3 V
q = CV = 2 ¥ 6 = 12 mC VA - VB = 12 V
In loop ABCD, fi VA = 9 V
I ¥1 - 2 - 5 = 0 fi I = 7 A 24. By Kirchhoff’s voltage law,
20. While charging A

Re = R fi t = RC
C1 = 60mF
While discharging
Re = 2 R fi t = 2 RC C2 = 20mF V0 C3 = 30mF
21. Common potential,
C V - C1V1 3 ¥ 100 - 1 ¥ 100 B C
V= 2 2 =
C1 + C2 1+3
q1 + q2 + q3 = 0
= 25 V C1(C A - V0 ) + C2( VB - V0 ) + C3( VC - V0 ) = 0
1 ¥ 1.5 C V + C2VB + C3VC
22. q1 = ¥ 30 = 18 mC V0 = 1 A
1 + 1.5 C1 + C2 + C3
C1 C2
60 ¥ 6 + 2 ¥ 20 + 3 ¥ 30
1.0mF
a
1.5mF V0 =
60 + 20 + 30
P q1 q1 Q 49
q2 q2 = V
11
b 25. In the steady state, there will be no current
2.5mF 0.5mF
C3 C4 in the circuit.

30V
78
3mF 1mF C1C2
3mF 1mF B 29. H = ( V1 - V2 )2 is independent of
B 2 (C1 + C2 )

3mF
resistance.
3mF
2mF 3mF 1mF 30. Immediately after switch is closed, capacitor
1mF behaves like short circuit.
2W 10W 20W 10W
10V 10V V - t / RC V
31. i1 = e , i2 = e - t / RC
A A 2R R
5t
i1 1 6 RC
= e
C1=6mF C2=3mF i2 2
B
q1 q2 Increases with time.
C1=1mF rd d
32. R = =
q3 A sA
20W 10W ke0 A
C=
A 10V d
d ke A
C2 3 10 t = RC = ¥ 0
\ V1 = V= 10 = V sA d
C1 + C2 3+6 3
e0 8.85 ¥ 10-12
E1 18 = = =6s
26. I1 = = =3 A 6 7.4 ¥ 10-12
R1 + r1 5 + 1
E2 15 33. i = i0e -t / t
I2 = = = 2.5 A i0
- ln 4
R2 + r2 4 + 2 fi = i0e RC
2
3mF ln 4
A + – B fi = ln 2
I1 I1 q RC
15V
18V 5W 4W
fi ln 4 = ln 2 RC
2W
1W q I2 I2 fi RC = 2
– + 2 2
D C fi R= = =4W
2mF C 0.5
34. Potential difference across each capacitor is
In loop ABCD, equal, hence they are in parallel,
q q
- I2 R2 + - I1 R1 = 0 charge on each capacitor
3 2
5q q = CeV = 2 ¥ 10 = 20 mC
fi = 3 ¥ 5 + 2.5 ¥ 4 fi q = 30 mC
6 As plate C contributed to two capacitors,
27. During discharging charge on plate,
q = q0 e - t / t C = 2q = + 40 mC
q0 = CE = 10 mC 35. Charge distribution on the plates of the
at t = 12 s, capacitor is shown in figure
q = 10e -12/ 6 = 10e -2 Q/2 CV +
Q Q/2
2
= (0.37)210 mC
C1C2
28. q = ( E1 - E2 ) –CV +
Q
C1 + C2 2
q C1
Vap = = ( E1 - E2 ) Q
C2 C1 + C2 CV +
Q¢ 2
\ V¢ = =
Ê E - E2 ˆ C C
=Á 1 ˜C
ÁC + C ˜ 1 Q
Ë 1 2¯ =V +
2C
79
t
36. Let q be the charge on C2 (or charge flown C2q0 Ê - ˆ
q= Á1 - e t ˜
through the switches at any instant of time) C1 + C2 Á ˜
Ë ¯
By Kirchhoff’s law C1 + C2
S1
t=
I C1C2 R
t
C Ê - ˆ
or q= q0 Á1 - e RC ˜
C1 Á ˜
Ë ¯
C1C2
+ where, C=
+ C1 + C2
q0 – q – q C2
– C1C2
37. H = ( V1 - V2 )2
q q -q 2 (C1 + C2 )
IR + - 0 =0 2
C2 C1 C1C2 Ê q0 ˆ
= Á ˜
dq C2q0 - (C1 + C2 ) q
= 2 (C1 + C2 ) ÁË C1 ˜¯
dt C1C2 R C2q02 C q02
q dq t dt = =
Ú0 C2q0 - (C1 + C2 ) q = Ú0 C1C2 R 2C1 (C1 + C2 ) 2C12

1 38. Electric field in the gap will remain same.


[ln|C2q0 - (C1 + C2 ) q]0
q
= 39. Electric field inside the dielectric slab
C1 + C2
E V
1 E¢ = = .
= t k kd
C1C2 R

More Than One Correct Options


1. Charge distribution is shown in figure I1 C 2R
EB = E1 + E2 + E3 + E4
Q Q Q Q I2 2C R
= + + -
4e0 A 4e0 A 4e0 A 4e0 A
Q
=
2e0 A
E
X Y t
+Q +Q –Q +Q Ê - ˆ

2

2

2

2 q2 = 2CE Á1 - e RC ˜
Á ˜
Ë ¯
t
dq1 E - RC
A B C = e
dt R
t
dq2 2E - 2 RC
= e
dt R
1 2 3 4 q1 1 q0 1
= fi 1 =
|EA| = |EC| = |E1 + E2 - E3 + E4| but EA and q2 2 q02 2
EC have opposite direction.
t1 = t 2 = RC

t
q 4 ¥ 10-3
Ê - ˆ 3. V1 = = = 40V
2. q1 = CE Á1 - e RC ˜ C 100 ¥ 10-6
Á ˜
Ë ¯
80
V2
fi U µd
Q Qd
V= =
200W C e0 A
V1 900W fi V µd
6. When switch S is open
100W C
A2 S

A1
C
I1 = 0 C
V1 40 2
I2 = = = A + –
900 900 45
2 80 E
V2 = I2 ¥ 200 = ¥ 200 = V
45 9 C ¥ 2C 2
80 Ce = = C
E = V1 + V2 = 40 + C + 2C 3
9 2
440 q1 = CE
= V 3
9
When switch S is closed
E
4. Initially I1 = 0,I2 = I = Ce ¢ = 2C
R
C q2 = 2CE
Charge flown through the battery
4
I2 Dq = q2 - q1 = CE = positive
I1 B 3
7. Let charge q flows to C1 at it falls to the free
I S end of the wire.
+ – A
A C1 C1 –q
E
q
As the capacitor starts charging,
I2 decreases and I1 increases, q2 q3 q2–q q3–q
+ – + – + – + –
In the steady state 2mF 3mF C2 C3
E C2 C3
I1 = I = , I2 = 0
R By Kirchhoff’s voltage law,
At any instant q2 - q q3 - q q
+ - =0
P1 = I12 R, P2 = I22 R C2 C3 C1
Steady state potential difference across the q2 q
+ 3
capacitor, C2 C3
E q=
V= 1 1 q
2 + +
C1 C2 C3
1 CE2 V2 + V3
U = CV 2 = =
2 8 1 1 1
Q2 + +
5. F = independent of d. C1 C2 C3
2e0 A 150 + 120
Q q= = 180 mC
E= independent of d. 1 1 1
+ +
e0 A 2 3 1.5
Q 2 Q 2d q2 ¢ = q2 - q = 150 ¥ 2 - 180
U= =
2C 2e0 A = 120 mC
81
e0 A - 6 ln 2
8. C = q0 12 - 1
d I= e 6 = ¥
RC 6 2
Q 2 Q 2d 1
U= = fiU µd =2 ¥
=1 A
2C 2e0 A 2
Q Qd Potential difference across 1 W resistor
V= = fi V µd
C e0 A fi 1 ¥1 = 1 V
e A 1 Potential difference across 2 W resistor
C = 0 fiC µ
d d fi 1 ¥2 = 2 V
Q
E= fi E is independent of d. \ By Kirchhoff’s voltage law, potential
e0 A
difference across capacitors = 1 + 2 = 3 V.
9. R = 1 + 2 = 3 W, C = 2 F 10. q = C1V1 = 1 ¥ 10 = 10 mF
q0 = CV0 = 2 ¥ 6 = 12C 6mF
At any instant
Ê -t ˆ 1mF 4mF 1mF 4mF 9mF
q = q0 Á e RC ˜ 3mF
Á ˜
Ë ¯
-t V1 V2 V3
dq q0 RC
I= = e
dt RC E E
at t = 0
q 12 q 10
I= 0 = =2 A V2 = = = 2.5 V
RC 3 ¥ 2 C2 4
q 10
at t = 6 ln 2 V3 = = V
C3 9

Match the Columns


1. C1¢ = kC1 = 8 mF, q¢ 1.6V
E2 ¢ = = ,
C e0 A e0 A
C2 ¢ = 2 = 2 mF
k q 2V V
E2 = = =
C ¢C ¢ ke0 A 2 ¥ e0 A e0 A
q¢ = 1 2 V = (1.6V ) mC
C1 ¢ + C2 ¢
E2 ¢ > E2
q = (2V ) mC ( a Æ q), (b Æ q), (c Æ q), (s Æ p).
\q ¢ < q 2. Before switch S is closed, charge distribution
2
q¢2 (1.6 V ) is shown in figure (1).
U2 ¢ = = = 0.64V 2,
C2 ¢ 2 ¥2 4mF
S + –
q2 (2)2 40mC
U2 = = = (1 V ) mC
2C2 2 ¥ 2 + –
3mF 20mC
+ –
U2 ¢ < U2
q¢ 1.6V 2mF
V2 ¢ = = = 0.2V ,
C2 ¢ 8
q (2V ) 30V
V2 = = = 0.5V
C2 4 Fig. 1
V2 ¢ < V
After switch S is closed, charge distribution
is shown in figure (2).
82
4mF C
S + –
+ –
120mC
+ – 60mC S
3mF + –
2mF
2C
C1V1 + C2V2 V
30V V¢ = =
C1 + C2 3
Fig. 2 1 1
U1 = C1V ¢2 =` CV 2
2 18
(a Æ s), (b Æ p), (c Æ q ), (d Æ s).
1 1
3. (a Æ q), (b Æ p, r), (c Æ q), ( d Æ p, p) U2 = C2V ¢ = CV 2
2
2 9
(p) C1 C2 C3 C1C2
a b DU = ( V1 - V2 )2
2 (C1 + C2 )
V C ¥C
V1 = V2 = V3 = = ( V - 0 )2
3 2 (C1 + C2 )
CV
q1 = q2 = q3 = 1
3 = CV 2
6
(q) C1
(a Æ r), (b Æ p), (c Æ q).
C2 ke A e A
5. C1 = 0 + 0
a b 2d 2d
C3 e0 A 3 e0 A
= ( k + 1) =
2d 2d
V1 = V2 = V3 = V 1 d d
= = +
C2 2ke0 A 2e0 A
q1 = q2 = q3 = CV
(r) C2 d Ê1 + kˆ
= Á ˜
2e0 A Ë k ¯
2ke0 A 4e A
a b fi C2 = = 0
C1 d (1 + k) 3d
C1 9
C3 =
C2 8
2V V As combination is series, q1 = q2
V1 = , V2 = V3 = q1
3 3 fi =1
2CV CV q2
q1 = , q2 = q3 +
3 3 U1 C2 8
= =
(s) C1 U2 C1 9

C2
(a Æ s), (b Æ s), (c Æ s).
a b 6. Charge distribution is shown in figure.
(a Æ p), (b Æ p, q), (c Æ s), (d Æ p, q, r).
C 4Q Q 2Q 7Q
3
2V V
V2 = , V1 = V2 =
3 3
2CV CV 7Q –3Q 3Q –2Q 2Q 0 0 7Q
q2 = , q1 = q3 =
3 3
4. Common potential
23 Magnetics
Introductory Exercise 23.1
1. [ Fe ] = [ Fm ] Æ
fi |Fm| = qvB sin q
[ qE] = [ qvB]
If Fm = 0, either B = 0 or sin q = 0,
fi È E ˘ = [v] = [L T -1 ] i.e., q = 0
Í B˙
Î ˚ Æ Æ Æ
Æ Æ Æ 4. F = q ( v ¥ B )
2. F = q ( v ¥ B )
^ ^ ^
Æ Æ Æ Æ = - 4 ¥ 10-6 ¥ 10-6 ¥ 10-2 [(2 i - 3 j + k )
\ F ^ v and F ^ B ^ ^ ^
¥ (2 i + 5 j - 3 k )]
Because cross product of any two vectors is
always perpendicular to both the vectors. ^ ^
= - 4 ¥ 10-2(4 i + 8 j + 16 k )
^
Æ Æ Æ
3. No. As Fm = q ( v ¥ B ) ^ ^ ^
= - 16 ( i + 2 j + 4 k ) ¥ 10-2 N

Introductory Exercise 23.2


1. As magnetic field can exert force on charged mv
(a) r=
particle, it can be accelerated in magnetic qB
field but its speed cannot increases as fi r µm
magnetic force is always perpendicular to
Hence, electron will describe smaller circle.
the direction of motion of charged particle. 2pr 2p m
Æ Æ Æ (b) T= =
2. Fm = - e ( v ¥ B ) v qB
Æ 1 qB
f = =
By Fleming’s left hand rule, B must be along T 2p m
positive z-axis.
1
3. As magnetic force provides necessary fi f µ
m
centripetal force to the particle to describe a
\electron have greater frequency.
circle.
mv2 4. Electrons are refocused on x-axis at a
qvB = distance equal to pitch, i.e.,
r
mv n = p = v||T
fi r=
qB 2p mv cos q
=
eB
84
mv mv 2mk
5. (a) If L ≥ r = , 6. r = =
qB eB eB
O 2m eV 1 2mV
= =
q eB B e
× × ×
For electron,
× × × × × × × × -31
× × × × × × × × q 1 2 ¥ 9.1 ¥ 10 ¥ 100
r=
× × × × × × × × 0.2 1.6 ¥ 10-19
p – q × × × × × × × ×
2 2 × × × × × × × × = 1.67 ¥ 10-4 m = 0.0167 cm
× × × × × × × × For proton
q/2 -27
× × × × × × × × 1 2 ¥ 1.67 ¥ 10 ¥ 100
L
r= -19
0.2 1.6 ¥ 10
× × × × × × × ×
= 7 ¥ 10-3 m = 0.7 cm
(b) The particle will describe a semi-circle.
mv 2m k
Hence, q = p 7. r = =
L q qB qB
(c) = cos
l 2 m
\ rµ
L q q
fi = cos
q 2
2 R sin 1 2 4
2 \ rp : rd : ra = : :
1 1 2
L 1
= sin q fi sin q = = 1 : 2 :1
R 2
p
fi q=
6

Introductory Exercise 23.3


1. Let at any instant 2p mv sin q
=
Æ qB
^ ^ ^
V = Vx i + Vy j + Vz k
Hence, coordination of the particle,
Now, Vx2 + Vy2 = V02 = constant Ê 2p mv sin q ˆ
qE = ( x, b) = ÁÁ0, ˜
˜
and V2 = V0 - f Ë qB ¯
m
Æ Æ Æ ^ ^ ^
Æ 3. F = i ( l ¥ B ) = ilB [ i ¥ ( j + k )]
V is minimum when V2 = 0
mv0 Æ
at f = ( F ) = 2 ilB
qE
^ ^ ^ ^ ^ ^ ^ ^ ^
4. No. as i ¥ ( i + j + k ) = i ¥ j + i ¥ ( j ¥ k )
and Vmin = V0
^ ^
2. After one revolution, y = 0, But i ¥ j = 0
x = p = pitch of heating ^ ^ ^ ^
\ i ¥ (i + j + k) = i ¥ ( j ¥ k)
^ ^ ^
85

Introductory Exercise 23.4


1. Consider the disc to be made up of large w qx dx
dM = di ¥ A = ¥ px 2
number of elementary concentric rings. p R2
Consider one such ring of radius x and wq
thickness dx. = 2 x 3dx
R
Charge on this ring \ Magnetic moment of entire disc,
wq R
M = Ú dM = 2 Ú x 3dx
R 0
x w q È R4 ˘ 1 2
= 2 Í ˙ = w qR
R ÍÎ 4 ˙˚ 4
Æ Æ Æ
2. M = i ¥ [(OA ¥ AB )]
q q Æ
dq = ◊ dA + ¥ 2px dx ^ ^
p R2 pR2 OA = OA cos q j + OA sin q k
2qx dx Æ ^
dq = 2 AB = AB i
R
Æ ^ ^ ^
Current in this ring, \ M = i ¥ OA ◊ AB [(cos q j + sin q k ) ¥ i ]
dq w dq w qx dx È Ê 3 ^ 1 ^ ˆ ^˘
di = + = = 4 ¥ 0.2 ¥ 0.1 Í Á j + k ˜ ¥ i˙
T 2p pR2
ÍÎ ÁË 2 2 ˜¯ ˚˙
\ Magnetic moment of this ring,
^ ^
= (0.04 j - 0.07 k ) A-m 2

Introductory Exercise 23.5


1. (a) B1 = B2 = B3 = B4 2l
2pr = 4l fi r =
m i p
= 0◊ [sin 45∞ + sin 45∞ ] m0 i pm 0 i
4p l / 2 B= r= = 24.7 mT (inward)
2 2 4l
m0 i
1 2. B = ◊
4p x
45° (As P is lying near one end of conductor 1)
i
l
2 B2 = 0 (Magnetic field on the axis of a current
3
carrying conductor is zero)
2 i x
4 l P
m 0 2 2i
◊=
4p l i
Net magnetic field at the centre of the
square,
1
m 8 2i
B = B1 + B2 + B3 + B4 = 0 ◊
4p l B = B1
2 2 m 0i m i
= 0◊
= = 28.3 mT (inward) 4p x
pl
By right hand thumb rule, direction of
(b) If the conductor is converted into a magnetic field at P is inward.
circular loop, then
86
60∞ m 0i m 0i
B2 = ◊ = (inward)
360∞ 2b 12b
3. Magnetic field due to straight conductor at O
60∞ m 0i m 0
i B3 = = = (outward)
360∞ 2a 12a
O
As B3 > B2,
Net magnetic field at P,
m0 2i B = B3 - B2
◊ B1 =
4p R m 0i È 1 1 ˘
= -
Magnetic field at O due to circular loop 12 ÍÎ a b ˙˚
m i
B2 = 0 6. AB, AP and BP from Pythagorus triplet,
2R
hence –APB = 90∞
By right hand thumb rule, both the filds are
i1
acting inward.
A X
Hence, 5.0cm
m i
B = B1 + B2 = 0 (1 + p )
2pR P Æ
B2
4p ¥ 10-7 ¥ 7 Ê 22 ˆ 13.0cm
Æ
B1
= Á1 + ˜
2p ¥ 10 ¥ 10-2 Ë 7¯
12.0 cm
= 58 ¥ 10-6 T = 58 mT (inward).
4. B1 = B2 = 0 (Magnetic field on the axis of B X
current carrying conductor is zero) i2
1
i Æ m 2i ^
\ B1 = 0 ◊ 1 PB
3
4p r1
Æ m 2i ^
B 2 = 0 ◊ 2 AP
R 4p r2
2
O B = B12 + B22
1 m 0i m 0i 2 2
B3 = ◊ = m0 Ê i1 ˆ Êi ˆ
4 2R 8R = Á ˜ + Á 2˜
2p Ár ˜ Ár ˜
4p ¥ 10-7 ¥ 5 Ë 1¯ Ë 2¯
=
8 ¥ 3 ¥ 10-2 4p ¥ 10-7 3 ˆ
2
Ê 3 ˆ
2
= ¥ ÊÁ ˜ +Á ˜
= 2.62 ¥ 10-5 T 2p Ë 0.05 ¯ Ë 0.12 ¯
= 26.2 mT (inward). = 1.3 ¥ 10-5 T
5. B1 = B2 = 0 (Magnetic field on the axis of = 13 mT
straight conductor is zero) 7. t = NIAB cos q
2 3 = 100 ¥ 1.2 ¥ 0.4 ¥ 0.3 ¥ 0.8 ¥ cos 30∞
= 9.98 N-m
1
60° Rotation will be clockwise as seen from
4 above.
87

Introductory Exercise 23.6


1. By right hand thumb rule, direction of At point B
magnetic field due to conductor A, B, C and m 0 I1 m I
B1 = ◊ ≠, B2 = 0 ◊ 2 Ø
D are as shown in figure. 4pe0 r2 4p r2
C
A X Net field at B
m0 1
B = B2 - B1 = ◊ ( I2 - I1 )
4p r2
Æ Æ 2m 10-7
B
Æ D BC Æ = ¥ (3 - 2) = 0.67 ¥ 10-4 T
BA BB
3 ¥ 10-3
Æ
B
= 67 mT
B X D
3. Consider the cylinder to be made up of large
m 0 2I number of elementary hollow cylinders.
BA = BB = BC = BD = ◊
4p r
Here, I = 5 A R

a 0.2
r= = = 0.14
2 2
\Net magnetic field at P Consider one such cylinder of radius r and
B = ( BA + BD )2 + ( BB + BC )2 thickness dr.
m0 4 2 I Current passing through this hollow
= ◊ cylinder,
4p r
di = jdA = j ( 2pr dr ) 2p br 2dr
10-7 ¥ 4 2 ¥ 5
= = 20 ¥ 10-6 T (a) Total current inside the portion of radius r1,
0.2 / 2 r1
I1 = Ú di = 2pbÚ r 2dr
= 20 mT 0
r1
Clearly resultant magnetic field is È r3 ˘
= 2 pb Í 1 ˙
downward. ÍÎ 3 ˙˚ 0
2. At point A 2
= p br13
I2 B1 3
××
I1
m
By ampere’s circuital law,
q
b
Ú B ◊ dl = m 0i1
2 pr1

2
B1 ¥ 2 pr1 = m 0 ÊÁ p br13 ˆ˜
B2

Ë3 ¯
m 0 I1
B1 = ◊ m 0 br12
4p r1 fi B1 =
3
B2 = 0 (Magnetic field inside a current (b) Total current inside the cylinder
carrying hollow cylinder is zero) R
m I i = 2 pbÚ r 2dr
\ Ba = B1 + B2 = 0 ◊ 1 0
4p r1 2
-7
p bR3
=
10 ¥1 3
= - 10-4 T
1 ¥ 10-3 m 2 i m 0 bR3
B2 = 0 =
4p r2 3 r2
= 100 mT (upward)
88

AIEEE Corner
Subjective Questions (Level-1)
1. Positive. By Flemings left hand rule. Æ ^ ^ ^
7. Let B = Bx i + By j + Bz k
2. Fm = evB sin q
Fe Æ Æ Æ
fi v= (a) F = q( v ¥ B )
eB sin q ^ ^
4.6 ¥ 10-15 7.6 ¥ 10-3 i - 5.2 ¥ 10-3 k
=
1.6 ¥ 10-19 ¥ 3.5 ¥ 10-3 ¥ sin 60∞ ^ ^
= - 7.8 ¥ 10-6 ¥ 3.8 ¥ 10 3( Bz i - Bx k )
= 9.46 ¥ 106 m / s fi Bx = - 0.175 T, Bz = - 0.256 T
3. Fm = qvB sin q (b) Cannot be determined by this information.
= (2 ¥ 1.6 ¥ 10-19 ) ¥ 105 ¥ 0.8 ¥ 1 Æ Æ Æ
(c) As F = q ( v ¥ B )
= 2.56 ¥ 10-14 N
Æ Æ Æ
Æ Æ F ^B
4. (a) Fm = e ( v ¥ B )
ÆÆ
^ ^ Hence, B ◊ F = 0
= - 1.6 ¥ 10-19 [(2.0 ¥ 106 ) i + (3.0 ¥ 106 ) j]
Æ ^
^
¥ (0.03 i + 0.15 j)
^ 8. B = B i
^ Æ ^
= - (6.24 ¥ 10-4 N) k (a) v = v j
Æ Æ Æ ^
Æ Æ Æ ^
(b) = Fm e ( v ¥ B ) = - (6.24 ¥ 10-4 N) k F = q ( v ¥ B ) = - qvB k
Æ Æ Æ Æ ^
5. Fm = e ( v ¥ B ) (b) v = v j
^ ^ ^ Æ Æ Æ ^
(6.4 ¥ 10-19 ) k = - 1.6 ¥ 10-19[(2 i + 4 j) F = q ( v ¥ B ) = qvB j
^ ^ Æ ^
¥ ( Bx i + 3 Bx j)] (c) v = - v i
^
6.4 ¥ 10-19 k = - 1.6 ¥ 10-19[2Bx k ]
^ Æ Æ Æ
F = q( v ¥ B ) = 0
6.4 ¥ 10-19 Æ
Bx = = - 2.0 T ^ ^
(d) v = v cos 45∞ i - v cos 45∞ k
- 3.2 ¥ 10-19
Æ Æ Æ qvB ^
6. (a) As magnetic force always acts F = q( v ¥ B ) = - j
perpendicular to magnetic field, magnetic 2
field must be along x-axis. Æ ^ ^
(e) v = v cos 45∞ j - v cos 45∞ k
F1 = qv1B sin q1 Æ Æ Æ qvB ^ ^
F1 5 2 ¥ 10-3 F = q( v ¥ B ) = (- j - k)
fi B= = 2
qv1B sin q1 1 ¥ 10-6 ¥ 106 ¥ 1 qvB ^ ^
2 =- ( j + k)
2
fi B = 10-3 T
mv 2m k 2 m eV
Æ ^ 9. r = = =
or B = (10-3 T ) i qB eB eB
(b) F2 = qv2 B sin q2 2mV
fi B= r
= 1 ¥ 10-6 ¥ 106 ¥ 10-3 ¥ sin 90∞ e
= 10-3 N 2 ¥ 9.1 ¥ 10-31 ¥ 2 ¥ 103
= ¥ 0.180
F2 = 1 mN 1.6 ¥ 10-19
89
= 0.36 ¥ 10-4 T 13. The component of velocity along the
B = 3.6 ¥ 10-4 T magnetic field (i.e., vx ) will remain
mv qBr unchanged and the proton will move in a
10. (a) r = fiv= helical path.
qB m
z
1.6 ¥ 10-19 ¥ 2.5 ¥ 6.96 ¥ 10-3 Æ
B
= -27
3.34 ¥ 10
vy cos wt
= 8.33 ¥ 105 ms -1
T pm
(b) t = =
2 qB vy sinwt vy

-27
3.14 ¥ 3.34 ¥ 10
=
1.6 ¥ 10-19 ¥ 2.5
= 2.62 ¥ 10-8 s At any instant,
1 Components of velocity of particle along
(c) k = eV = mv2
2 Y-axis and Z-axis
mv2 v¢y = vy cos q = vy cos wt
fi V=
2e and v¢z = - vz sin q = vz sin wt
3.34 ¥ 10-27 ¥ ( 8.33 ¥ 105 )2 qB
= where, w=
2 ¥ 1.6 ¥ 10-19 m
= 7.26 ¥ 103 V Æ ^ ^ ^
\ v = vx i + vy cos w t j - vz sin w t k
= 7.26 kV
14. For the electron to hit the target, distance
11. (a) - q. As initially particle is neutral, charge
G S must be multiple of pitch, i.e.,
on two particles must be equal and opposite.
(b) The will collide after completing half GS = np
rotation, i.e., For minimum distance, n = 1
T pm 2p mv cos q
t= = fi GS = p =
2 qB qB
× × × × ×B 2p 2 mk cos 60∞
×
×
× +q × –q ×
× × ×
×
× fi p= (mv = 2 mk)
×
×
×
×
×
×
×
×
×
×
qB
× × × × ×
×
×
×
×
×
×
×
×
×
× 2p 2 mk cos 60∞
× × × × × fi B=
×
×
×
×
×
×
×
×
×
× qp
× × × × ×
× × × q × ×
× × × × × 1
2 ¥ 3.14 ¥ 2 ¥ 9.1 ¥ 10-31 ¥ 2 ¥ 1.6 ¥ 10-16 ¥
10.0 2
12. Here, r = = 5.0 cm, =
2 1.6 ¥ 10-19 ¥ 0.1
mv mv
(a) r = fiB= fi B = 4.73 ¥ 10-4 T
qB qr
15. (a) From Question 5 (c)
9.1 ¥ 10-31 ¥ 1.41 ¥ 106
= Introductory Exercise 23.2
1.6 ¥ 10-19 ¥ 5 ¥ 10-2 L
= sin q fi L = R sin q
= 1.6 ¥ 10-4 T R
By Fleming’s left hand rule, direction of R
R sin 60∞ =
magnetic field must be inward. 2
T pm mv mv0
(b) t = = fi L= =
2 qB 2qB 2qB0
3.14 ¥ 9.1 ¥ 10-31 (b) Now, L ¢ = 2.1 L = 1.05 R
=
1.6 ¥ 10-19 ¥ 1.6 ¥ 10-4 As L ¢ > R,
= 1.1 ¥ 10-7 s
90
2
Particle will describe a semicircle and move mv
qvB0 =
out of the magnetic field moving in opposite R
direction, i.e., qB0 R
v=
v¢ = - v = - v0 i
^ m
qB0 R cos q
T pm vx = v cos q =
and t= = m
2 qB0 qB0Z
= (Q R cos q = Z)
Æ ^ Æ ^
m
16. v = (50 ms -1 ) i, B = (2.0 mT ) j 2qE0Z q2B02Z 2
Now, vz = v2 - vx2 = -
As particle move with uniform velocity, m m2
Æ Æ Æ Æ Æ ^ Æ ^
F = q(E + v ¥ B ) = 0 18. Given, E = E j , B = B k,
Æ Æ Æ Æ ^ ^

^
E = B ¥ v = - ( 0.1 N/C)k v = v cos q j + v sin q k
As protons are moving undeflected,
17. If v be the speed of particle at point (0, y, z )
Æ Æ Æ Æ
then by work-energy theorem, F = 0fi e (E + v ¥ B ) = 0
z Æ ^
B = –B j ^ ^
fi e ( E j - vB cos q j) = 0
E = EK. E
vy v or v=
B cos q
q vx Now, if electric field is switched off
Z 2p mv sin q 2p mE tan q
p= =
O
x qB qB2
(Component of velocity along magnetic field
= vz = v sin q)
19. F = I l B sin q
1 F 0.13
mv2
W = DK = I= =
2 lB sin q 0.2 ¥ 0.067 ¥ sin 90∞
But work done by magnetic force is zero, = 9.7 A
hence, network done = work done by electric Fm
20. For no tension in springs
force × ×I× ×I× ×
Fm = mg ××× ×××
= qEZ ××× ×××
××× ×××
1 fi I lB = mg ××× ×××
××× ×××
\ qE0Z = mv2 mg 13.0 ¥ 10-3 ¥ 10 mg
2 I= =
2qE0Z lB 62.0 ¥ 10-2 ¥ 0.440
fi v=
m = 0.48 A
As the magnetic field is along Y-axis, By Fleming left hand rule, for magnetic force
particle will move in XZ-plane. to act in upward direction, current in the
The path of particle will be a cycloid. In this wire must be towards right.
case, instantaneous centre of curvature of 21. (a) FBD of metal bar is shown in figure, for
the particle will move along X-axis. metal to be in equilibrium,
As magnetic force provides centripetal force Fm + N = mg Fm
to the particle, fi Fm = mg - N N
z
fi I lB = m - N mg
V
vz fi lB = mg - N
v R
q
vx R
q
fi V= ( mg - N )
R lB
X
91
For largest voltage, Æ Æ ^ ^
l 3 = cd = - (40 ¥ 10-2 ) i + (40 ¥ 10-2 m ) j
N =0
Æ Æ Æ
R mg 25 ¥ 750 ¥ 10-3 ¥ 9.8 ^
F3 = I ( l 3 ¥ B ) = - (0.04 N) k
V= =
lB 50.0 ¥ 10-2 ¥ 0.450 Æ Æ ^ ^
l 4 = da = (40 ¥ 10-2 m ) i - (40 ¥ 10-2 m ) k
= 817.5 V
Æ Æ Æ ^ ^
(b) If I lB > mg F4 = I ( l 4 ¥ B ) = (0.04 N) i + (0.04 N) k
I lB - mg = ma Æ ^
I lB - mg V lB 24. M = IA M
a= = -g ^ ^
m Rm = 0.20 ¥ p( 8.0 ¥ 10-2 )2(0.60 i - 0.80 j)
817.5 ¥ 50 ¥ 10-2 ¥ 0.45
= - 9.8 ^ ^
= (40 . 2 ¥ 10-4 ) (0.60 i - 0.80 j)A-m 2
2 ¥ 750 ¥ 10-3
Æ ^ ^
= 112.8 m/s 2 B = (0.25 T) i + (0.30 T) k
22. I = 3.50 A, l = - (1.00 cm ) i
^ Æ Æ Æ
(a) t = M ¥ B
^
fi l = - (1.00 ¥ 10-2 m ) i ^ ^
= (40.2 ¥ 10-4 )( - 0.24 i - 0.18 j + 0.2 k )
^

Æ ^ ^ ^ ^
(a) B = - (0.65 T) j = ( - 9.6 i - 7.2 j + 8.0 k ) ¥ 10-4 N-m.
Æ Æ Æ ^ ÆÆ
Fm = I ( l ¥ B ) = - (0.023 N) k (b) U = - M◊ B = - (40.2 ¥ 10-4 )(0.15) J
Æ ^ ª - 6.0 ¥ 10-4 J
(b) B = + (0.56 T) k
Æ Æ Æ 25. Consider the wire is bent in the form of a
^
Fm = I ( l ¥ B ) = (0.0196 N) j loop of N turns,
Æ L
^ Radius of loop, r=
(c) B = - (0.33 T) i 2pN
Æ Æ Æ Magnetic dipole moment associated with the
Fm = I ( l ¥ B ) = 0
loop
Æ ^ Æ i L2
(d) B = (0.33 T) i - (0.28 T ) k M = NiA = Ni ¥ pr 2 =
4pN 2
Æ Æ Æ ^ 2
Fm = I ( l ¥ B ) = - (0.0098 N) j iL B
t = MB sin 90∞ =
Æ ^ ^
4pN
(e) B = + (0.74 T) j - (0.36 T ) k
Clearly t is maximum, when N = 1
Æ Æ Æ ^ ^
Fm = I ( l ¥ B ) = - (0.0259 N) k + (0.0126 N) j and the maximum torque is given by
^ ^ i L2B
= (0.0126 N) j - (0.0259 N) K tm =
4p
Æ ^
23. B = (0.020 T) j 26. Consider the disc to be made up of large
Æ Æ number of elementary rings. Consider on
^
l1 = ab = - (40.0 cm) j such ring of radius x and thickness dx.
^ Charge on this ring,
= - (40.0 ¥ 10-2 m ) j
Æ Æ Æ
F1 = I ( l1 ¥ B ) = 0
x dx
Æ Æ ^
l 2 = bc = (40.0 cm) k
^
= - (400 ¥ 10-2 m ) k
q 2q
Æ Æ Æ ^ dq = ¥ 2px dx = 2 x dx
F2 = I ( l 2 ¥ B ) = (0.04 N) i p R2 R
92
Current associated with this ring, y
dq w dq w q g
di = = = x dx f
T 2p pR2 b c
Magnetic moment of this ring
wq
dM = px 2di = 2 x 3dx h x
R e
a d
Magnetic moment of entire disc, z
wq R 1
M = Ú dM = 2 Ú x 3dx = w qR2 …(i) Æ
R 0 4 ^
Mabcd = - i l2 k
Magnetic field at the centre of disc due to the Æ ^
elementary ring under consideration Mefgh = i l2 k
m di m 0w q2 Æ
dB = 0 = dx ^
Madeh = i l2 j
2x 2pR2
Net magnetic field at the centre of the disc, \Total magnetic moment of the closed path,
m wq R m wq Æ Æ Æ Æ
B = Ú dB = 0 2 Ú dx = 0 ^
M = Mabcd + Mefgh + Madeh = i l2 j
2pR 0 2pR
M pR3 31. Circuit is same as in Q.30
\ =
B 2m 0 Æ ^ ^
M = i l2 j = j
27. (a) By principle of conservation of energy, Æ ^
Gain in KE = Loss in PE B =2 j
KE = - PE cos q + ME Æ Æ Æ
t = M ¥B =0
K 0.80 ¥ 10-3
f cos q = 1 - =1 - m0 I
ME 0.02 ¥ 52 ¥ 10-3 32. B1 = ◊
4p r
10 m0 I
= ◊ B2 =
13 4p r
10
q = cos -1 = 76.7∞ Here, B1 and B2 are perpendicular to each
13 other, hence,
10
(b) q = cos -1 = 76.7∞ 2
13
Entire KE will again get converted into PE l

28. DU = U2 - U1 = - MB - ( + MB)
= - 2 MB
= - 2 ¥ 1.45 ¥ 0.835 = - 2.42 J 1
l
-11
2pr 2 ¥ 3.14 ¥ 5.3 ¥ 10
29. (a) T = = B = B12 + B22
v 2.2 ¥ 106
m 0 2I 10-7 ¥ 2 ¥ 5
= 1.5 ¥ 10-16 s = ◊ =
e 1.6 ¥ 10
-19 4p r 35 ¥ 10-2
(b) i = = = 1.1 ¥ 10-3 A
T 1.5 ¥ 10-16 = 2.0 ¥ 10-6 T
= 1.1 mA = 2.0 mT
(c) M = p r 2i 33. Clearly DBOC ~ DAOB
r2 AD
= 3.14 ¥ (5.3 ¥ 10-11 )2 ¥ 1.1 ¥ 10-3 \ =
r6 BC
= 9.3 ¥ 10-24 A-m 2
fi r2 = 2r
30. Suppose equal and opposite currents are
flowing in sides a d and e h, so that three = 100 mm
complete current carrying loops are formed,
93
A q
a a
p
q
B
2 q
3
q
l O 2a I
1
C r a

D r2
4
and AD = 2BC = 200 mm m0 I
r = ◊ (inwards)
q = cos -1 = 45∞ 4p a 2
BC m0 I
2 B3 = B4 = (sin 0 + sin 0)
4p 2a
m0 I 2I m I
BBC = [sin 45∞ + sin 45∞ ] = 0◊ (outwards)
4p r r 4p 2a 2
m0
= (outwards) Net magnetic field at P
4p
m I B = B1 + B2 - ( B3 + B4 )
BAD = 0 ◊ (sin 45∞ + sin 45∞ ) m I
4p r2 = 0◊ (inwards)
4p 2a
m0 2 I m
= ◊ (inwards) I m I q
4p r2 36. B = 2 ¥ 0 ◊ - 0 ¥ =0
4p R 2 R 2p
Net magnetic field at O. fi q = 2 rad.
2 m 0I È 1 1 ˘
B = BBC - BAD = Í - ˙
4p ÍÎ r1 r2 ˙˚ l
R
È 1 1 ˘ q
= 2 ¥ 10-7 ¥ 2 Í -3
- -3 ˙ l
ÍÎ 50 ¥ 10 100 ¥ 10 ˙˚
-6
= 2 ¥ 10 T = 2 mT (outwards)
37. (a) Consider a point P in between the two
34. Let us consider a point P ( x, y) where
conductors at a distance x from conductor
magnetic field is zero. Clearly the point must
carrying current I1 (= 25.0 A),
lie either in 1st quadrant or in 3rd quadrant.
l2

P(xy) I2 = 75.0 A
x P
l1 I1 = 25.0 A
r

Magnetic field at P
m 0 2I1 m 0 2I2 m I m I
B= ◊ - ◊ =0 B = 0 ◊ 1 - 0 ◊ 2 =0
4p y 4p x 4p x 4p r - x
fi I1x = I2 y I1 I
fi = 2
ÊI ˆ x r-x
fi y=Á 1˜x r - x I2
ÁI ˜
Ë 2¯ fi =
x I1
35. q = 45∞
m0 I I1 25.0
B1 = B2 = ◊ (sin q + sin q) fi x= r= ¥ 40 = 10 cm
4p a I1 + I2 100.0
94
-2
(b) Consider a point Q lying on the left of the 2 ¥ 0.0580 ¥ 2.40 ¥ 10
=
conductor carrying current I1 at a distance x 4p ¥ 10-7 ¥ 800
from it.
fi I = 2.77 A
(b) On the axis of coil,
m 2 NIA
x B= 0◊ 2
4 p ( r + x 2 )3 / 2
I2 = 75.0 A
3/ 2
BC ( r 2 + x 2 )3/ 2 Ê r2 + x 2 ˆ
I1 = 25.0 A = fiÁ ˜ =2
B r 3 Á r2 ˜
Ë ¯
m 0 I1 m 0 I2 fi x = 0.0184 m
B= ◊ - ◊ =0
4p x 4p r + x 41. Let the current I2 ( = I ) upwards
I1 I I1 I2
fi = 2
x r+x I3
I1 25.0
fi x= r= ¥ 40
I2 - I1 50.0 I4
= 20 cm
m 2 N pr 2I B = - B1 + B2 - B3 + B4
38. B = 0 ◊ 2
4 p ( r + x 2 )3 / 2 m 2
= 0 ◊ [ - I1 + I2 - I3 + I4 ] = 0
4p r
But, x = R
m NI 4 2 Br I2 = I1 + I3 - I4
B= 0 fiN= = 10 + 8 - 20
4 2r m 0I
= -2 A
4 2 ¥ 6.39 ¥ 10-4 ¥ 6 ¥ 10-2
= Negative sign indicates that current I is
4p ¥ 10-7 ¥ 2.5
directed downwards.
fi N = 69 Æ m I^
42. B KLM = - 0 i
39. For magnetic field at the centre of loop to be 4R
zero, magnetic field due to straight
conductor at centre of loop must be outward,
hence I1 must be rightwards. L M
I
At the centre of the loop
I2 I
N
I
K
R

Æ m I^
D B KNM = 0 j
4R
Æ Æ Æ m I ^ ^
I1 B = B KLM + B KNM = 0 ( - i + j)
4R
B = B1 - B2 Æ Æ Æ m Iqv ^
(a) F = q ( v ¥ B ) = - 0 k
m 0 2I1 m 0I2 4R
= ◊ - =0
4p D 2R Æ Æ ^
pD (b) l1 = l 2 = - 2 R k
I1 = I2 Æ Æ Æ
R F1 = I ( l1 ¥ B ) = 2 IRB i
^

Æ Æ Æ ^
m 0 NI 2 BR F2 = I ( l 2 ¥ B ) = 2 IRB i
40. (a) B = fiI=
2R m 0N Æ Æ Æ ^
F = F1 + F2 = 4 IRB i
95
43. (a) Length of each side ÆÆ -6
l ÚbB ◊ dl = - m 0I1 = - 5.0 ¥ 10 T-m
ÆÆ -6
Úc B ◊ dl = m 0( I2 - I1 ) = 2.5 ¥ 10 T-m
ÆÆ -6
q
a
ÚdB ◊ dl = m 0( I2 + I3 - I1 ) = 5.0 ¥ 10 T-m

46. P1 y
x
2 pr r
l= a
n

qq
p P2
q=
n a
l pr
a = cot q = cot q r
2 n
m i Current density
B = n ¥ 0 ◊ (2 sin q) I 2I
4p a J= =
2
m 2n 2 sin q a pa 2
= 0◊ pa 2 - 2p ÊÁ ˆ˜
4p p r cot q Ë2¯
p Let us consider both the cavities are
m 0 i n 2 sin 2 carrying equal and opposite currents with
= n
p current density J.
2p 2r cos
n Let B1, B2 and B3 be magnetic fields due to
p complete cylinder, upper and lower cavity
m 0i n 2 sin 2
n respectively.
(b) lim B = lim
n Æ• n Æ• p (a) At point P1
2p 2r cos
n
m 0i Æ m 2I ^ m 2J ¥ pa 2 ^
fi lim = B1 = - 0 ◊ 1 i = - 0 ◊ i
n Æ0 2r 4p r 4p r
ÆÆ m I^
44. Ú B ◊ dl = 3.83 ¥ 10-7 T-m =- 0 i
pr
2
(a) By Ampere’s circuital law 2 J ¥ p Êaˆ
Æ m 2 I2 ^ m 0 Á ˜
ÆÆ B2 = 0 ◊ i= ◊ Ë 2 ¯ ^i
Ú B ◊ dl = m 0I 4p r - a 4p r-
a
1 ÆÆ 1 2 2
¥ 3.83 ¥ 10-7
m0 Ú
fiI= B ◊ dl = m 0I ^
4p ¥ 10-7 =- i
Ê aˆ
4p Á r - ˜
= 0.3A Ë 2¯
(b) If we integrate around the curve in the Æ m 0 2 I3 ^ m0 ^
opposite direction, the value of line integral B3 = ◊ i= i
4p r + a Ê aˆ
4p Á r + ˜
will become negative, i.e., 2 Ë 2¯
- 3.83 ¥ 10-7 T-m. Æ Æ Æ Æ
ÆÆ B = B1 + B 2 + B 3
45. Ú B ◊ dl = m 0I
È ˘
As the path is taken counter-clockwise Í 4m 0I 1 1 ˙^
=Í - + + i
ÆÆ 4p a a˙
direction, Ú B ◊ dl will be positive if current is Í r r- r+ ˙
ÍÎ 2 2 ˙˚
outwards and will be negative if current is 2
Æ m 0I È 2r - a ˘ ^ 2
inwards. B = Í ˙i
ÆÆ 4pr ÍÎ 4r 2 - a 2 ˙˚
Ú B ◊ dl = 0
a
96
2 2
Æ m I È 2r - a ˘ x w
\ (B ) = 0 Í 2 2˙
, towards left.
4pr ÍÎ 4r - a ˚˙
(b) At point P2
y
Æ
B1 y z
x A
B3 sin q l
q Fig.1
q P2
q
B 2 sin Æ ÆÆ
Æ
B2
B2 cos q
B3
ÚWXYZ
B ◊ dl = m 0l l
B3 cos q ÆÆ ÆÆ ÆÆ ÆÆ
fi Ú B ◊ dl + Ú B ◊ dl + Ú B ◊ dl + Ú B ◊ dl = m 0l l
Æ m 2I ^ m I ^ WÆX X ÆY Y ÆZ Z ÆW
B1 = 0 ◊ 1 j = 0 j
4p r pr B l + 0 + B l + 0 = m 0l l
Æ m0 2 I2 1
B2 = ◊
^ ^
[ - sin q i - cos q j ] B = m0 l
4p 2 2
a
r2 + In Fig. 2.
4
- m 0I ^ ^
= [sin q i + cos q j ]
2p 4r 2 + a 2 B2 B1
P
Æ m 2 I3 ^ ^
B3 = 0 ◊ [sin q i - cos q j ]
4p a2
r2 + Q B2
B1
4
Fig.2
m 0I ^ ^
= [sin q i - cos q j ]
2p 4r 2 + a 2 At point P,
1
Æ Æ Æ Æ B1 = B2 = m0 l
B = B1 + B 2 + B 3 2
È2 ˘ B = B1 - B2 = 0,
m 0I
Í - 2 cos q ˙ ^j
= At point Q,
2p Ír 4r + a ˙˚
2 2
Î 1
B1 = B2 =
m0 l
r 2r 2
but, cos q = =
2
a 4r + a 2
2 B = B1 + B2 = m 0 l
r2 +
4 Æ Æ Æ Æ
Æ m I dl ¥ r m 0 q ( v ¥ r )
Æ m 0I È 2 4r ˘^ 48. B = 0 ◊ = ◊
\ B = Í - 2 j 4p r3 4p r3

2p ÍÎ r 4r + a ˙˚ y

m I È 2r 2 + a 2 ˘ ^ u
= 0 Í 2 ˙j
4pr ÍÎ 4r + a 2 ˙˚ o x

Æ m I È 2r 2 + a 2 ˘
(B ) = 0 Í 2 ˙ , upwards.
4pr ÍÎ 4r + a 2 ˙˚

47. Let us first find magnetic field due a current Æ ^


v = ( 8.00 ¥ 106 ms -1 ) j
carrying infinite plate.
Æ ^
Consider a rectangular amperian loop (a) r = (0.500 m ) i
(WXYZ) as shown in Fig. 1. Æ Æ
Æ m 0 q( v ¥ r )
B = ◊
4p r3
97
^ ^
10-7 ¥ 6.00 ¥ 10-6[( 8.00 ¥ 106 j) ¥ (0.500) i ] fi
= 3 Æ 10-7 ¥ 4.00 ¥ 10-6 [(2.00 ¥ 105 ^i ) ¥ ( - 0.300 ^j)]
(0.500) B1 =
Æ ^
(0.300)3
fi B = - (1.92 ¥ 10-5 T ) k
Æ ^
^
(b) r = - (0.500 m ) j = - ( 8.89 ¥ 10-7 T ) k
Æ Æ Æ Æ
Æ m 0 q( v ¥ r ) Æ m q (v ¥ r )
B = ◊ =0 B 2 = 0 ◊ 2 23 2
4p r3 4p r2
Æ ^ fi
(c) r = + (0.500 m ) k
Æ 10-7 ¥ ( - 1.5 ¥ 10-6 )[( 8.00 ¥ 105 ^i ) ¥ ( - 0.400 ^j)]
Æ Æ B2 =
Æ m 0 q( v ¥ r ) ^ (0.400)2
B = ◊ = (1.92 ¥ 10-5 T ) i
4p r3
Æ ^
^
(d) r = - (0.50 m) j + 0.500 m k
^
= - ( 7.5 ¥ 10-7 T ) k
Æ Æ Æ Æ Æ ^
Æ m 0 q( v ¥ r ) ^ B = B1 + B 2 = - (16.4 ¥ 10-6 T ) k
B = ◊ = (1.92 ¥ 10-5 T ) i
4p r3 ^
= - (1.64 ¥ 10-6 T ) k
49. q = - 4.80 mC = - 4.80 ¥ 10-6 C
Æ or B = 1.64 ¥ 10-6 T (inwards)
^
v = (6.80 ¥ 105 m / s ) i 51. Magnetic force per unit length on the
Æ ^ conductor AB,
(a) r = (0.500 m ) i m 2I I
Æ Æ f = 0◊ 1 2
Æ m 0 q( v ¥ r ) 4p r
B = ◊ =0 For equilibrium
4p r3
m
Æ ^ f = g=lg
(b) r = (0.500 m ) j l
Æ Æ m 2I I
Æ m 0 q( v ¥ r ) fi lg = 0 ◊ 1 2 …(i)
B = ◊
^
= - (1.3 ¥ 10-6 T ) k 4p r
4p r3 Suppose wire AB is depressed by x,
Æ ^ ^
(c) r = (0.500 m ) i + (0.500 m ) j f
Æ Æ A I1
B
Æ m 0 q( v ¥ r ) ^
B = ◊ = - (1.31 ¥ 10-6 T ) k r lg
4p r3
Æ ^ C D
(d) r = (0.500 m ) k I2
Æ Æ Net force on unit length of wire AB
Æ m 0 q( v ¥ r ) ^
B = ◊ = (1.31 ¥ 10-6 T ) j la = lg - f ¢
4p r3
m 2I I m 2I I
Æ Æ = 0◊ 1 2 - 0◊ 1 2
Æ m q (v ¥ r ) 4p r 4p r - x
50. B1 = 0 ◊ 1 13 2
4p r1 m 0 2 I1I2 ◊ x
=- ◊
y 4p r ( r - x )
q v If x << r
v2 m 0I1I2
0.300m
la = - x
2r 2
x m II
0.400m q2 a = - 0 1 22 x …(ii)
2l r
98
General equation of SHM D C G
I
a = - w2x …(ii) I1 FCG 2 FCD I3

Hence, motion of wire AB will be simple 3 cm 5 cm


harmonic.
r1 r2
From Eqs. (i) and (ii),
30 A 10 A 20 A
m 0I1I2
w= m 2I I
2 lr 2 f CG = 0◊ 2 3
4p r2
2p 2 lr 2 r
T= = 2p = 2p f = f CD - f CG
w m 0I1I2 g
mo ÈI I ˘
0.01 f = f CD - f CG = ◊ 2 I2 Í 1 - 3 ˙
= 2 ¥ 3.14 4p ÍÎ r1 r2 ˙˚
9.8
È 30 20 ˘
= 0.2 s = 10-7 ¥ 2 ¥ 10 Í -2
- ˙
m 0 2I1I2 ÍÎ 3 ¥ 10 5 ¥ 10-2 ˙˚
52. (a) f = ◊
4p r f = 12 ¥ 10-4 N = 1.2 ¥ 10-3 N/m
f r F = f l = 1.2 ¥ 10-3 ¥ 25 ¥ 10-2
I2 =
m0
◊ 2I1 = 3 ¥ 10-4 N
4p
4.00 ¥ 10-5 ¥ 2.50 ¥ 10-2 54. Force per unit length on wire MN
= m 2I I
10-7 ¥ 2 ¥ 0.600 f MN = 0 ◊ 1 2
4p a
= 8.33 A m II L
F = f MN ¥ L = 0 1 2
(b) As the wires repel each other, current must 2 pa
be in opposite directions. Torque acting on the loop is zero because
m 2I I magnetic field is parallel to the area vector.
53. f CD = 0 ◊ 1 2
4p r1

Objective Questions (Level 1)


1. Fact Æ Æ Æ
7. M = i A , where A = Area vector.
2 pm
2. T = is independent of speed.
qB 8. Force acting on a closed current carrying
loop is always zero.
3. Outside the wire
m 2I 9. M = NIA
B= 0◊ where, r is distance from the Æ Æ ÆÆ
4p r 10. a ^ B fi a ◊ B = 0
centre.
^ ^ ^ ^ ^ ^
4. The path will be parabola if force acting on ( x i + j - k ) ◊ (2 i + 3 j + 4 k ) = 2x + 3 - 4 = 0
the particle is constant in magnitude as well fi x = 0.5
as in direction.
12. A current carrying closed loop never
m 2I
5. B = 0 ◊ experiences a force magnetic field.
4p r mv P
4p rB 13. r = = ,
m0 = qB qB
2I
m ¥ Wb / m 2 P = mv = momentum.
Units of m 0 = 1
A \ rµ
q
= Wbm -1A -1 rp qa
6. Fact \ = fi rP : ra = 2 : 1
ra q p
99
14. W = MB (cos q1 - cos q2 ) 2 eV
fi v=
Here, q1 = p, q2 = p - q m
W = MB (cos p - cos ( p - q)) Magnetic force,
= - MB (1 - cos q) Fm = evB sin q
m0 I Fm µ v fi Fm µ v
15. BP = ◊ (2 sin q)
4p r Hence, if potential difference is doubled,
force will become 2 times.
3 cm

5 cm
19. Magnetic field at O due to P,
I
6 cm

P Q
q q

r P
3 cm

I I
5 cm
R/2 R/2
2 2 O
r = 5 - 3 = 4 cm
= 4 ¥ 10-2 m R
3
sin q = m 0 2I m I
5 B1 = ◊ = 0 (inwards)
3 4p R / 2 p R
-7
10 ¥ 50 ¥ 2 ¥
\ BP = 5 Magnetic field at O due to Q,
4 ¥ 10-2 m 2I m I
B2 = 0 ◊ = 0 (inwards)
4p R / 2 p R
= 1.5 ¥ 10-4 T
= 1.5 gauss. Net magnetic field at O,
2 m 0I
16. Magnetic field on the axis of current B = B1 + B2 =
carrying circular loop, pR
m 2M 20. As solved in Question 16,
B1 = 0 ◊ 2 …(i)
4 p ( r + x 2 )3 / 2 3/ 2
B2 Ê x 2 + R2 ˆ
=Á ˜
Magnetic field at the centre of current B1 ÁË R2 ˜¯
carrying circular loop, 3/ 2
m 2M Ê x 2 + R2 ˆ
B2 = 0 ◊ 3 …(ii) fi Á ˜ =8
4p r Á R2 ˜
Ë ¯
From Eqs. (i) and (ii), x 2 + R2
B2 ( r 2 + x 2 )3/ 2 fi =4
= R2
B1 r3 fi x= 3 R
(3 2 + 42 )3/ 2 21. Component of velocity of particle along
=
33 magnetic field, i.e.,
125 qE
= vy = t = aE t
27 m
125
fi B2 = ¥ 54 = 150 mT is not constant, hence pitch is variable.
27
mv 2 mK
Æ Æ Æ Æ Æ 22. r = =
17. F = I( l ¥ B ) = I ( ba ¥ B ) qB qB
Æ Æ Æ Æ 2 mK
= - I ( ab ¥ B ) = I ( B ¥ ab) Now, R =
eB
18. Kinetic energy of electron,
2m (2K ) 2
1 R¢ = = R
K = mv2 = e V e (3 R) 3
2
100
23. Same as question 1. Introductory exercise y
23.6.
Note. Her diagram is wrong correct diagram 1 2
should be
4
A x C
I

O x
a 3
B x D
b
mv 2 mK 2 mqV Æ - 1 Ê m 0I ˆ ^ m I ^
24. r = = = [K = qV ] B2 = Á ˜ k = - 0 k,
qB qB qB 4 Ë 2b ¯ 8b
2 mV Ê 1 ˆ Æ 1 Êm I ˆ ^ m I ^
fi r= Á ˜ B3 = Á 0 ˜ k = 0 k
q Ë B¯ 4 Ë 2a ¯ 8a
25. Magnetic field due to a conductor of finite Æ Æ Æ Æ Æ
B = B1 + B 2 + B 3 + B 4
length.
m I m 0I È 1 1 ˘ ^
B = 0 ◊ (sin a + sin b ) = - k
4p r 8 ÍÎ a b ˙˚
Here, a = - q2, b = q1 and r = a 29. Current associated with electron,
m I q
\ B = 0 (sin q1 - sin q2 ) I= = ef
2a T
m I m ef
26. In case C, magnetic field of conductor 1-2 B= 0 = 0
and 2-3 at O is inward while those of 3-4 and 2R 2R
4-1 at O is outward, hence net magnetic field 30. Same as question 1(a). Introductory Exercise
at O in this case is zero. 23.5.
2 3 31. At point 1,
Magnetic field due to inner conductor is
I
O
non-zero, but due to outer conductor is zero.
Hence, B1 π 0
1 4 At point 2,
Æ Æ Æ Magnetic field due to both the conductors is
27. dF = I ( dl ¥ B ) equal and opposite.
Æ Æ Hence, B2 = 0
But B ||dl at every point,
32. Apply Fleming’s left hand rule or right hand
Æ
hence, dF = 0. thumb rule.
33. Magnetic field due to straight conductors at
28. B1 = B3 = 0 (Magnetic field on the axis of
O is zero because O lies on axis of both the
current carrying straight conductor is zero)
conductors.
f m 0I m 0If
Hence, B = ◊ =
2p 2x 4px
34. Inside a solid cylinder having uniform
current density,
101
m 0Ir
B=
2 p R2
Here, r = R - x
m 0I ( R - x )
\ B=
2 p R2
r
35. Magnetic force is acting radially outward on
the loop.

JEE Corner
Assertion and Reason
1. For parabolic path, acceleration must be mv 2 meV
r= =
constant and should not be parallel or qB eB
antiparallel to velocity.
7. For equilibrium
2. By Fleming’s left hand rule.
Æ Æ
3. Magnetic force on upper wire must be in Fe + Fm = 0
upward direction, hence current should be in Æ Æ Æ
a direction opposite to that of wire 1. fi q E = - q( v ¥ B )
Reason is also correct but does not explain Æ Æ Æ Æ Æ
fi E =- v ¥B = B ¥ v
Assertion.
Æ Æ
4. t = MB sin a 8. Pm = Fm ◊ v
a = 90∞ Æ Æ
\t = MB π 0 As Fm is always perpendicular to v ,
5. F2 = I lBO x1 Æ
Pm = 0
y
Æ ÆÆ
B Again, Pe = Fe◊ v , may or may not be zero.
1
2 4
9. Reason correctly explains Assertion.
10. Magnetic force cannot change speed of
F2 F4
particle as it is always perpendicular to the
speed of the particle.
3 v2
11. a =
R
x1 x2
x but R also depends on v.
F qvB
\ a= m =
F4 = I lB0x2 m m
\ F4 > F2 fi a µv
Hence, net force is along X-axis.
6. Radii of both is different because mass of
both is different
102

Objective Questions (Level 2)


1. For net torque to be zero. Charge on this cylinder,

T
q
R
y
O
mg

IAB0 = mgR
mgR mgR x
I= =
AB0 pR2B0
mg q
= dq = ◊ (2p yx dx )
p RB0 4
pR3
3
2. As it is clear from diagram,
= 3 q cos 2 q sin q dq
l Current associated with this cylinder,
dq w dq 3 w q
di = = = cos 2 q sin q dq
T 2p 2p
I
Magnetic moment associated with this
cylinder,
(– 4,0) (2,0) 3 qw
dM = di A = cos 2 q sin q dq ¥ px 2
2p
3
dM = R2wqA cos 2 q sin 3 q dq
2
Effective length of wire, 3 0
M = Ú dM = R2qÚ cos 2q sin 3 q dq
Æ ^ 2 p2
l = (4 m ) i
3 0
Æ Æ Æ = R2w qÚ cos 2q (1 - cos 2 q)sin q dq
F = I( l ¥ B) 2 p/2
0
Æ 3 2 È cos 3 q cos 5 q ˘
Æ F I Æ Æ = R wq Í - ˙
a = = ( l ¥ B) 2 ÍÎ 3 5 ˙˚ p/ 2
m m 1 2
2 ^ ^ ^ = R wq
= (4 i ¥ ( - 0.02 k )) = 1.6 j m /s 2 5
0.1
5. As solved in question 5(c). Introductory
3. Impulse = Change in momentum
Exercise 23.2.
Ú I lB dt = mv - 0 L
= sin q
lBÚ dq = mv R
mv m 2 gh mV
dq = = Here, L = d, R =
lB lB qB
qB d
4. Consider the sphere to be made up of large \ = sin q
number of hollow, coaxial cylinder of mV
q V sin q
different height and radius. Consider one or =
such cylinder of radius x, height y and m Bd
thickness. 6. Force on portion AC will more compared to
Now, y = 2 R cos q, x = R sin q, dx = R cos q dq that on portion CB.
103
mv
7. Consider an elementary portion of the wire i.e., b-a£
carrying current I1 of length dx at a distance qB
x from end B. qB ( b - a )
or v≥
m
11. Consider an elementary portion of length dx
at a distance x from the pivoted end.
I1
B I2
dx dxe
a
2a
x
x

Force on this portion Charge on this portion


q
dF = I1dx B dq =
dx
l
m 2I I
= 0 ◊ 1 2 dx Current associated with this portion
4p a + x
dq qf
di = = dx
Total force on wire AB T l
m 2 a dx
F = Ú dF = 0 ◊ 2I1I2 Ú Magnetic moment of this portion
4p a a + x
pqf 2
m II dM = px 2di = x dx
= 0 1 2 ln 3 l
2p pqf l 2 1
x dx = pqfl2
l Ú0
M=
8. Magnetic field line due to current carrying 3
conductor is shown in figure. 12. At x = 0, y = ± 2 m
z
Effective length of wire
^
l = (4 m ) j
Æ Æ Æ ^ ^
y \ Fm = I( l ¥ B ) = 3(4 j ¥ 5 k )
x
^
= 60 i N
13. Effective length of wire,
× × × × × × × × × × × × × × × ×
2 IA1 P Q
m
9. B1 = 0 ◊ 2 × × × × × × × × × × × × × × × ×
4p ( x1 + r12 )3/ 2
× × × × × × × × × × × × × × × ×
m 2I ◊ pr12 I
= 0◊ 2 S 60° T
4p ( x1 + r12 )3/ 2 g
3
m 2I ◊ pr22 4d
B2 = 0 ◊ 2
4p ( x2 + r22 )3/ 2 R
B1 r12 ( x22 + r22 )3/ 2 3
= l = ST = 2 ¥ a ¥ cot 60∞
B2 r22 ( x12 + r12 )3/ 2 4
a
But, r1 = x1 tan q =
2
and r2 = x2 tan q
B1 For equilibrium, I lB = Mg
\ =2 2Mg
B2 fi I=
lB
10. b - a must be less than or equal to radius of
circular path,
104
14. For particle not collide with the solenoid, Æ Æ Æ
19. As E = - v ¥ B
radius of path of particle £ half or radius of
solenoid. Net force on the particle must be zero.
mv r
≥ 20. Consider an elementary portion of length dy
qB 2 at y - y on the wire.
But B = m 0n i Force on this portion,
rqB m 0qr n i Æ Æ
fi v> = dF = I ( dy ¥ B )
2m 2m
16. Magnetic force cannot do work on charged Æ ^
Here, dy = - dy j (Current is directed along
particle, hence its energy will remain same,
negative y-axis).
so that q remains same.
^ ^ ^
Again, magnetic force is always along the dF = - I { dy j (0.3 y i + 0.4 y j)}
string, it will never produce a torque hence, ^
T will also remain same. = - 2 ¥ 10-3( - 0.3 y dy k )
17. Let the x-coordinates of loops be as shown in Total force on the wire,
figure, 1 ^
y F = Ú dF = - 2 ¥ 10-3 Ú ( - 0.3 y dy k )
0
^
F = (3 ¥ 10-4 k ) N
Æ Æ Æ
1 21. E = - v ¥ B
Æ rqB
|E | = vB = B
m
(5 ¥ 10-2 )(20 ¥ 10-6 )(0.1)2
O x =
a b b+a (20 ¥ 10-9 )
then, F1 = Ia ( B0a ) - 0 = I a 2B0 Æ
\ E = 0.5 V/m (1 mg = 10-9 kg )
F2 = Ia ( B0 ( b + a )) - Ia ( B0b)
= I a 2B0 22. Condition is shown in figure.
Æ
B2
\ F1 = F2 π 0 1
18. Consider an amperian loop of radius
x ( b < x < c ), threaded by current the
amperian loop,
Æ
×× A B1
2

C a

B
b m 0I1
B1 =
2 R1
m I
x 2 - b2 B2 = 0 2
I¢ = I - I 2 R2
c 2 - b2
c2 - x 2 \ B = B12 + B22
= I
c 2 - b2 m0 Ê I1 ˆ
2
ÊI ˆ
2
= Á ˜ +Á 2˜
2
m 0I ¢ m 0I ( c - x ) 2
2 ÁR ˜ ÁR ˜
I= = Ë 1¯ Ë 2¯
2px 2px ( c 2 - b2 )
105
2 Æ m 2I ^
Ê ˆ 2 B1 = 0 ◊ j
4p ¥ 10 -7 Á ˜ Ê 5 2 ˆ 4p a
Á 5 ˜
= ¥ + Á ˜
2 Á 5 ¥ 10-2 ˜ Á 5 ¥ 10-2 ˜ Æ m 2I ^
Á 2 ˜ Ë ¯ B2 = - 0 ◊ i
Ë ¯ 4p a
4p ¥ 10-7 ¥ 2 Æ
= = 4p ¥ 10-5 T B3 = 0
2 ¥ 10-2
Æ Æ Æ Æ m 0i ^ ^
B = B1 + B 2 + B 3 = ( j - i)
23. Initially, net force on the particle is zero. 2pa
Hence,
26. Effective length, l = AC = 42 + 3 2
E
V=
B =5 m
Now, if electric field is switched off. C
mv E Ê q = Sˆ
r= = Á ˜
qB SB2 Ëm ¯
24. For equilibrium,
mg
f = [f = magnetic force per unit length on A B
l
the conductors]
m 0 2I1I2
fi ◊ =lg F = I lB = 2 ¥ 5 ¥ 2 = 20 N
4p r
27. At point P,
m 2I I
fi r= 0◊ 1 2 1 qx
4p l g E= ◊
4pe0 ( R2 + x 2 )3/ 2
10-7 ¥ 2 ¥ 100 ¥ 50 m 2 iA
= B= 0◊ 2
0.01 ¥ 10 4 p ( R + x 2 )3 / 2
= 0.01 m q qv
Hence, i = =
Clearly, equilibrium of conductor B is T 2 pR
unstable.
and A = p R2
Æ Æ Æ
25. If B1, B 2 and B 3 be magnetic fields at the E 1 1 c2 È 1 ˘
\ = ◊ = Íc = ˙
given point due to the wires along x, y and z B m 0e0 v v ÍÎ m 0e0 ˙˚
axis respectively, then

More than One Correct Options


m 0 N1I1 4p ¥ 10-7 ¥ 50 ¥ 2 Æ
1. B1 = = If F = 0
2 R1 2 ¥ 5 ¥ 10-2
Æ Æ Æ
= 4p ¥ 10-4 T Either, E = - v ¥ B ,
m 0 N 2 I2 4p ¥ 10-7 ¥ 100 ¥ 2 Æ Æ
B2 = = -2 E π 0, B = 0
2 R2 2 ¥ 10 ¥ 10
Æ Æ Æ
= 4p ¥ 10-4 T or E = 0, or v ¥ B = 0
If current is in same sense, Æ Æ
Again, If v ¥ B = 0
B = B1 + B2 = 8p ¥ 10-4 T
Æ
And if current is in opposite sense, Either B = 0
B = B1 - B2 = 0 Æ Æ
Æ Æ Æ Æ Æ Æ or q = 0∞, i.e., v || B .
2. F = Fe + Fm = q ( E + v ¥ B )
106
3. The particle will describe a circle in x-y plane = q ( E0 - vB0 ) k
^

with radius, E0
2 2 If v < , particle will deflect towards
mv 1 ¥ 8 + 6 B0
r= = =5m
qB 1 ¥2 positive z-axis.
2pm E
and T= = p s = 3.14 s If v > 0 , particle will deflect towards
qB B0
Æ negative z-axis.
4. t = MB sin q E
If v = 0 , particle will move undeflected and
U = - pE cos q B0
q = 80∞ its KE will remain constant.
Hence, t = 0, U = pE = maximum. 8. K = e V fi K µ V will become double
As PE (U) is maximum, equilibrium is 2mK
R= fi R µ K will become 2 times.
unstable. qB
5. Fact. qB
w= is independent of kinetic energy.
6. Upward and downward components of force 2pm
will cancel each other while leftward force is 9. Use right hand thumb rule.
more than rightward force, hence net force is 10. For cd to be in equilibrium, force on it must
leftwards. be repulsive while for ab to be in equilibrium,
Æ Æ Æ Æ force on it must be attractive.
7. F = q E + q ( v ¥ B )
Equilibrium of cd will be stable while that of
^ ^ ^
= q { E0 k + (v j) ¥ ( B0 i )} ab will be unstable.

Match the Columns


1. ( a Æ r), (b Æ q), (c Æ p), (d Æ r) is maximum if a = 90∞, i.e., in case (b) only.
Æ Æ Æ Æ Æ And U = - PE cos a
Fm = q ( v ¥ B ) = - e ( v ¥ B )
U is positive if a is obtuse, i.e., in cases (a)
Æ Æ Æ
and Fm = q E = - e E and (d).
and U is minimum if a = 0, i.e., in case (c).
2. (a Æ r), (b Æ s), (c Æ q), (d Æ p)
Æ Æ Æ
As Fm = q ( v ¥ B ) 4. (a Æ q), (b Æ r), (c Æ s), (d Æ s)
By Fleming’s left hand rule, positively Use right hand thumb rule.
charged particles deflects towards left and 5. (a Æ q), (b Æ r), (c Æ q), (d Æ r)
negatively charged particles deflects towards
1 2 3 4
right.
mv 2 mK
Again, r = =
qB qB
m
fi rµ
q Æ Æ Æ
Æ Æ
F12 F24 F34 F24 F43
3. (a Æ p, s), (b Æ p, q), (c Æ p, r), (d Æ p, s)
Æ Æ Æ Æ Æ Æ Æ
Whenever a closed current carrying loop is F14 F13 F23 F21 F31 F32 F41
placed in uniform magnetic field, net force
experienced by it is zero. Direction of different forces on different
Also t = PE sin a wires is shown in figure.
107
6. (a Æ q, s), (b Æ p, r), (c Æ p, r), (d Æ q, s) Æ I Æ Æ I Æ Æ ^
F = ( l1 ¥ B ) + ( l 2 ¥ B ) = - B0I l k
2 2
Æ
t = 0, because lines of action of force on the
i
two wires are equal and opposite.
Æ ^
If B = B0 j
When the current is increased or the loop is Æ ^
moved towards the wire, magnetic flux F = B0I l k
linked with the loop increases. As a result of Again, lines of action of force on the two
this, induced current will produce in the loop wires are equal and opposite.
to decrease the magnetic field. Because
t =0
initial magnetic flux linked with the loop is
inward, induced magnetic flux will be Æ ^ ^
If B = B0 ( i + j)
outward and induced current will be
anti-clockwise and vice-versa. Æ
F =0
7. (a Æ r, s), (b Æ r, s), (c Æ q, r), (d Æ p, r) Æ
y t =0
I/2 Æ ^
If B = B0 k
I/2 I/2 Æ ^ ^
F = B0I l ( i - j)
I x
I/2 Æ
fi |F | = 2B0I l
Effective lengths of two conductors,
^ ^
t =0
l1 = l2 = l i + l j
Æ ^
If B = B0 i
24 Electromagnetic
Induction
Introductory Exercise 24.1
1. Magnetic field inside the loop due to current 2. No.
carrying conductor is inwards. Emf is induced if the field is time varying.
As the current in the conductor dfB
increases, magnetic flux linked 3. = induced emf
dt
with the loop increases as a È dfB ˘ 2 -3 -1
result of which, induced current \ Í dt ˙ = [ V ] = [ML T I ]
will produce in the loop to Î ˚
produce an outward magnetic field, i.e.,
induced current will be anti-clockwise.

Introductory Exercise 24.2


1. If the outward magnetic flux increases,
1 2 i'
induced current will be in such a way that it i
produces inwards magnetic flux, i.e., it will
be clockwise.
2. Magnetic flux linked with the coil will not i increasing
change, hence induced current will be zero. 1 2 i'
i
3. If the current in coil 1 (clockwise) increases,
outward magnetic flux linked with the coil 2
increases and the coil 2 will produce induced i decreasing
current in clockwise direction to oppose the
change in magnetic flux linked with it. Hence, if the current in coil 1 increases,
induced current will be in same sense and
vice-versa.

Introductory Exercise 24.3


1. fB = BS = B0 S e - at Because, i = 0 as the circuit is not closed. As
df net force acting on the bar is zero, no
e = - B = a B0 S e -at
dt external force is required to move the bar
with constant velocity.
2. No.
As, Fm = i lB = 0
109
f2 - f1 m0 2i
3. |e| = B= ◊
t 4p a + x
But, f1 = NB1 A cos q, f2 = NB2 A cos q Induced emf in this portion,
NA cos q ( B2 - B1 ) m 2vi
\ |e| = de = B dxv = 0 ◊ dx
t 4p d + x
|e|t
fiA= 5. (a) EMF induced in the bar ab,
N( B2 - B1 )cos q m l dx
e = Ú de = 0 ◊ 2 viÚ
80.0 ¥ 10-3 ¥ 0.4 4p 0d + x
=
3 m
50 ¥ (600 ¥ 10-6 - 200 ¥ 10-6 ) ¥ = 0 2vi [ln ( d + x )]0l
2 4p
= 1.85 m 2 m 0vi d + l
= ln
Side of square, a = A = 1.36 m 2p d
m 0vi Ê lˆ
Total length of wire = 50 ¥ 4a = ln Á1 + ˜
2p Ë d ¯
= 50 ¥ 4 ¥ 1.36 = 272 m
4. (a) Consider an elementary portion of length (b) Magnetic field in the region ab is
dx of the bar at a distance x from end a. inwards, hence by Fleming’s left hand
rule, positive charge will move up and a
Magnetic field at this point,
will be at higher potential.
i
Or
a
d Use Fleming’s right hand rule.
x (c) No.
dx As flux linked with the square loop will
v l
b remain same.

Introductory Exercise 24.4


1. Potential difference across an inductor,
di d
V =L =L (3 t sin t )
dt dt
= 3 L [sin t + t cos t ]

Introductory Exercise 24.5


1. (a) Total number of turns on the solenoid, = 4.5 ¥ 10-5 H
l 40 ¥ 10-2 di
N= = (b) e = -L
d 0.10 ¥ 10-2 dt
0 - 10
= 400 = - 4.5 ¥ 10-5 ¥
0.10
m 0N2A
L= = 4.5 ¥ 10-3 V
l
= 4.5 mV
4p ¥ 10-7 ¥ (400)2 ¥ 0.90 ¥ 10-4
=
40 ¥ 10-2
110

Introductory Exercise 24.6


1. Consider a current i is flowing in the outer As R >> r, magnetic field inside smaller loop
loop. may assumed to be constant.
i
Hence, magnetic flux linked with the
smaller loop,
r R
m pr 2i
fm = B ¥ pr 2 = 0
2R
fm pm 0r 2
M= =
Magnetic field at the centre of the loop. i 2R
m i
B= 0
2R

Introductory Exercise 24.7


1. (a) V0 = i0 R = 36 ¥ 10-3 ¥ 175 = 6.3 V di
fi E=L + iR
(b) i = i0 (1 - e - t / t ) dt
L di 1
where, t = fi = [ E - iR]
R dt L
1
Now, at t = 58 ms = ¥ [12 - 1 ¥ 7]
3.00
i = 4.9 mA di 5
fi = = 1.67 A/s
\ 4.9 = 36 (1 - e -58 / t ) dt 3
31.1 L 3
fi e -58 / t = (c) t = =
36 R 7
fi t = 397 ms i = i0 (1 - e - t / t )
L E 12
= 397 ms = (1 - e -t / t ) = (1 - e -1.4/ 3 )
R R 7
fi L = 175 ¥ 397 ¥ 10-6 fi i = 0.639 A
= 69 mH E 12
(d) i0 = = = 1.71 A
(c) t = 397 ms R 7
[ e] [ V ][ t ] E2
2. [L] = = 4. (a) P = Ei = (1 - e - t / t )
È di ˘ [ i] R
ÍÎ dt ˙˚ (12)2
= (1 - e -7t / 3 ) = 20.6 (1 - e -2.33t ) W
[V ] 7
and [ R] =
[ i] (b) Rate of dissipation of energy,
È L ˘ [L] PR = i2 R = i02 R (1 - e -7t / R )2
\ ÍÎ R ˙˚ = [ R] = [T ]
= 20.6 (1 - e -2.33t )2 W
3. (a) Initially (c) Rate of increase of magnetic energy
di di
E=L PL = ei = L i
dt dt
di E
fi = = 20.6 ( e -2.33t - e -4.67t ) W
dt L
12.0 (d) Clearly, P = PR + PL
= = 4 A/s
3.00 5. No.
(b) E = VL + VR E = VL + VR and VR cannot be negative in
RL circuit.
111
6. Consider the system as a combination of two R1 R2
R= = 2W
batteries (E1 = 10 V and E2 = 0) as shown R1 + R2
R1 = 4W 8W E 5
A i B i0 = = = 0.5 A
i2 R + 8 10
i1
L 1
t= =
R2 = 4W 1H R + 8 10
i = i0 (1 - e - t / t )
E1 = 10V E2 = 0
i = 0.5 (1 - e -10t ) A
D C
\Current through inductor
fl i = 2.5 (1 - e -10t ) A
R 8W
In loop ABCDA
di
i1 R1 + 8i + L - E1 = 0
E 1H dt
i1 ¥ 4 + 8 ¥ 0.5 (1 - e -10t ) + 1 (5e -10t ) - 10 = 0
i1 = (1.5 - 0.25 e -10t ) A
E1 R2 + E2 R1
E= =5 V
R1 + R2

Introductory Exercise 24.8


[ q] [ i][ T ] fi |q| = 18 ¥ 10-6 ¥ 0.75 ¥ 3.40
1. [C ] = = ]
[V ] [V ] = 46.5 ¥ 10-6 C
[ e] [V ] [T ]
[L] = = = 46.5 mC
È di ˘ [ i] di Ê 1 ˆ
ÍÎ dt ˙˚ (b) e = - L = - L Á- q˜
dt Ë LC ¯
fi [ LC ] = [ L C ] = [T ] q 4.8 ¥ 10
-4
= = = 23.3 V
2. In LC oscillations, magnetic energy is C 18 ¥ 10-6
equivalent to kinetic energy in spring block
system. 4. i0 = wq0
dq dx 1
i= fiv= where, w =
dt dt LC
q0 i
Also L is equivalent to inertia (m) in fi V0 = = 0
electricity, hence C wC
1 L 20 ¥ 10-3
Magnetic energy = Li2 is equivalent to V0 = i0 = 0.1 ¥
2 C 0.5 ¥ 10-6
1
kinetic energy = mv2.
2 = 20 V
3. In LC oscillations,
di 1 di
(a) =- q fi q = - LC
dt LC dt

Introductory Exercise 24.9


dfm di
e=- = - m 0 nNA
dt dt
1. (a) B = m 0 ni
25
fm = NBA = m 0n NAi = - 4p ¥ 10-7 ¥ ¥ 10 ¥ 5.0 ¥ 10-4 ¥ ( - 0.2)
0.01
112
2
= 3.14 ¥ 10 V-6 R
=- (6.00t 2 - 8.00t )
= 3.14 mV 2r2
e 3.14 ¥ 10-6 R2
(b) E = = F = - eE = (6.00t 2 - 8.00t )
2pR 2 ¥ 3.14 ¥ 25 ¥ 10-2 ¥ 10 2r2
= 2 ¥ 10-7 V/m = 8.0 ¥ 10-21 N
As magnetic field is increasing in this region,
induced electric field will be anti-clockwise
2. B = (2.00t 3 - 4.00t 2 + 0.8) T
dB and hence, electron will experience force in
= (6.00t 2 - 8.00t ) T/s clockwise sense, i.e., downward at P2.
dt
dB (b) For point P1,
From, t = 0 to t = 1.33 s, is negative, dfm1 dB
dt Induced emf, V1 = - = - pr12
hence B is decreasing in that interval. dt dt
dB Induced electric field at P1,
For t > 1.33 s, is positive, hence B is
dt V 1 dB
E = - 1 = - r1
increasing for t > 1.33 s. 2pr1 2 dt
(a) For point P2, 1
dfm2 = - r1(6.00t 2 - 8.00t ) = 0.36 V/m
dB 2
induced emf, V2 = - = - pR2
dt dt At, t = 2.00 s
Induced electric field at P2, magnetic field is increasing, hence, induced
V R2 dB electric field will be anti-clockwise, i.e.,
E= 2 =- ◊
2pr2 2r2 dt upward at P1 and perpendicular to r1.

AIEEE Corner
Subjective Questions (Level 1)
f2 - f1 B ( A2 - A1 )
1. < e > = - =-
t t Charge flowing through the coil
A1 = pr 2 = 3.14 ¥ (0.1)2 q=<i>t
= 3.14 ¥ 10-2 = 0.0314 ( f2 - f1 ) ( - 0.04 - 0.04)
2 fiq=- =-
Ê 2pr ˆ R 50
A2 = a 2 = Á ˜
Ë 4 ¯ 0.08
= = 1.6 ¥ 10-3 C
2 50
Ê 2 ¥ 3.14 ¥ 0.1 ˆ
=Á ˜ = 0.025 = 1.6 mC = 1600 mC
Ë 4 ¯
100 (0.025 - 0.0314) 3. f1 = NBS, f2 = - NBS
\ <e>=-
0.1 Induced emf,
( f - f ) 2 NBS
= 6.4 V <e>=- 2 1 =
t t
2. f1 = NBA = 500 ¥ 0.2 ¥ 4 ¥ 10-4
Induced current
= 0.04 Wb < e > 2 NBS
f2 = - NBA = - 0.04 Wb <i>= =
R Rt
Average induced emf, Charge flowing through the coil,
(f - f ) 2 NBS
<e>=- 2 1 q=<i>t=
t R
Average induced current, qR 4.5 ¥ 10-6 ¥ 40
<e> (f - f ) fi B= =
<i>= =- 2 1 2 NS 2 ¥ 60 ¥ 3 ¥ 10-6
R Rt
113
= 0.5 T Hence, equal force in direction of motion of
coil is required to move the block with
Æ uniform speed.
^ ^
4. B = (4.0 i - 1.8 k ) ¥ 10-3 T,
(b) When the coil is entering into the
Æ ^ magnetic field, magnetic flux linked with
S = (5.0 ¥ 10-4 k ) m 2
the coil increases and the induced
ÆÆ current will produce magnetic flux in
f = B ◊ S = - 9.0 ¥ 10-7 Wb
opposite direction and will be
5. e = Blv = 1.1 ¥ 0.8 ¥ 5 = 4.4 V counter-clockwise and vice-versa.
By Fleming’s right hand rule, north end of i
BLv
the wire will be positive. i0 =
R
6. A = pr 2 = 3.14 ¥ (12 ¥ 10-2 )2 = 0.045 m 2
i0
(a) For t = 0 to t = 2.0 s
dB 0.5 - 0 x
= slope = = 0.25 T/s
dt 2.0 - 1 –i0
df dB
e=- m =-A
dt dt
= - 0.045 ¥ 0.25 = - 0.011 V 8. Consider an elementary section of length dl
|e| = 0.011 V of the frame as shown in figure. Magnetic
flux linked with this section,
(b) For, t = 2.0 s to t = 4.0 s
dB
= slope = 0 e = 0 l
dt dl
i
(c) For, t = 4.0 s to t = 6.0 s
dB 0 - 0.5
= slope = = - 0.25
dt 6.0 - 4.0 x a
df dB m 0 2i
e=- m =-A = 0.11 V dfm = BdB = ◊ adl
dt dt 4p x + l
7. (a) When magnetic flux linked with the coil Total magnetic flux linked with the frame,
changes, induced current is produced in it, in m ai a dl
such a way that, it opposes the change. fm = Ú dfm = 0 Ú
2p 0 x + l
Magnetic flux linked with the coil will m ai
change only when coil is entering in (from = 0 [ln ( x + a ) - ln x ]
3L L L 2p
x=- to x = - ) or moving (from x =
2 2 2 Induced emf
3L - dfm m ai È 1 1 ˘ dx
to x = ) of the magnetic field. e= = - 0 ◊Í - ˙
2 dt 2p Î x + a x ˚ dt
Because, of induced current, an opposing
force act on the coil, which is given by m 0a 2i m 0a 2iv
= v=
BLv B2L2v 2px ( x + a ) 2px ( x + a )
F = ilB = BL =
R R 9. As solved in Qusetion 4. Introductory
Exercise 24.3.
F
B2L2v i
F0 =
R d
v
l
F0

x
–3L –L O L 3L
2 2 2 2
114
m iv Ê lˆ Effective emf
e = 0 ln Á1 + ˜
2p Ë d¯ E r - E1r2
E= 21
Here, i = 10 A r1 + r2
v = 10 ms -1 0.008 ¥ 15.0 - 0.004 ¥ 10.0
=
l = 10.0 cm - 1.0 cm = 9.0 cm 15.0 + 10.0
d = 1.0 cm = 0.0032 V
4p ¥ 10-7 ¥ 10 ¥ 10 Ê 9.0 ˆ rr 15 ¥ 10
e= ln Á1 + ˜ r= 12 = =6W
2p Ë 1.0 ¯ r1 + r2 25
E 0.0032
e = (2 ¥ 10V ) ln (10) V i= = = 0.003 A = 0.3 mA
R+ r 5+6
10. Induced current
e Blv di
i=
= 12. (a) e = - L = - 0.54 ¥ ( - 0.030)
R R dt
Force needed to move the rod with constant = 1.62 ¥ 10-2 V
speed = Magnetic force acting on the rod (b) Current flowing from b to a is decreasing,
Blv hence, a must be at higher potential.
ie., F = i lB = lB
R 13. (a) i = 5 + 16t, |e| = 10mV = 10 ¥ 10-3 V
2 -2 2
B2l2v (0.15) ¥ (50 ¥ 10 ) ¥ 2 di d
= = |e| = L fi 10 ¥ 10-3 = L (5 + 16t )
R 3 dt dt
-3
fi F = 0.00375 10 ¥ 10
L= = 0.625 mH
11. Suppose the magnetic field is acting into the 16
plane of paper. (b) at t = 1 s
Rods 1 and 2 can be treated as cells of emf i = 5 + 16 (1) = 21 A
E1 ( = Blv1 ) and E2 ( = Blv2 ) respectively. Energy stored in the inductor,
2 1 1 1
U = Li2 = ¥ 0.625 ¥ 10-3 ¥ (21)2
2 2
B
ƒ = 0.138 J
dU di
P= = Li = 0.625 ¥ 10-3 ¥ 21 ¥ 16
v2 R v1 dt dt
= 0.21 W
14. From t = 0 to t = 2.0 ms
V -0 5.0 - 0
= =0
r2 r1 t - 0 2.0 ¥ 10-3
fl fi V = 2500 t
di
L = 2500 t
dt

2500
E2
R
E1 E R
Ú di = L Ú tdt
r1 i i 2500 t
r2 r fi Ú 0 di = L Ú 0 tdt
1250 2
i= t
L
Now, E1 = Blv1 = 0.010 ¥ 10.0 ¥ 10-2 ¥ 4.00 at t = 2.0 ms
= 0.004 V 1250
i= ¥ (2.0 ¥ 10-3 )2
E2 = Blv2 = 0.010 ¥ 10 ¥ 0 ¥ 10-2 ¥ 8.00 150 ¥ 10-3
= 0.008 V = 3.33 ¥ 10-2 A
115
From t = 2.0 ms to t = 4.0 ms di
18. (a)|e| = M = 3.25 ¥ 10-4 ¥ 830
V - 5.0 0 - 0.50 dt
=
t - 2.0 ¥ 10-3 (4.0 - 2.0) ¥ 10-3 = 0.27 V
V = - 2500 ( t - 2.0 ¥ 10-3 ) + 5.0 di
As, is constant, induced emf is
= - 2500 t + 10.0 dt
di constant.
L = - 2500 t + 10.0
dt (b) Coefficient of mutual induction remains
1 same whether current flows in first coil
di = ( - 2500 t + 10.0) dt
L or second.
1 di
i = [ - 1250 t 2 + 10.0 t ] Hence, | e| = M1 = 0.27 V
L dt
at t = 4 s 19. (a) Magnetic flux linked with the secondary
1
i= [ - 1250 ¥ (4.0 ¥ 10-3 )2 coil,
150 ¥ 10-3
f2 = Mi1
+ 10.0 (4.0 ¥ 10-3 )] f2 0.0320 ¥ 400
= 3.33 ¥ 10 A -2 M= =
i1 6.52
di |e| 0.0160
15. (a)|e| = L fi L = = = 1.96 H
dt di / dt 0.0640
(b) f1 = Mi2 = 1.96 ¥ 2.54 = 4.9784 Wb
= 0.250 H
Flux per turn through primary coil
(b) Flux per turn f 4.9784
Li 0.250 ¥ 0.720 = 1 =
f= = N1 700
N 400
= 7.112 ¥ 10-3 Wb/turn.
= 4.5 ¥ 10-4 Wb
di i -i 20. Same as Question 2. Introductory Exercise
16. |e| = M =M 2 1 24.4
dt t
-3 12 - 4 21. i = i0(1 - e - t / t )
fi 50 ¥ 10 = M ◊ Rt
0.5 E -
= (1 - e L )
50 ¥ 10-3 ¥ 0 . 5 R
M= = 3.125 ¥ 10-3 H Rt
8 di E - L
= e
= 3.125 mH dt L
If current changes from 3 A to 9 A in 0.02 s. Power supplied by battery,
di i -i E2 -
Rt
|e| = M =M 2 1 P = Ei = (1 - e L )
dt t R
9 -3
= 3.125 ¥ 10-3 ¥ Rate of storage of magnetic energy
0.02 Rt Rt
di E2 - -
= 0.9375 V P1 = Li = (1 - e L ) e L
dt R
17. (a) Magnetic flux linked with secondary coil, Rt 10 ¥ 0.1
P1 - -
fm2 = M i1 =e L =e 1 = e -1 = 0.37
P
f 6.0 ¥ 10-3 ¥ 1000 L 2
M= 2 = =2H 22. (a) t = = = 0.2 s
i1 3 R 10
dfm2 di L R
(b) e=- =-M 1
dt dt
0 -3
= -2 ¥ = 30 V K
0.2
fm 600 ¥ 5 ¥ 10-3
(c) L = 1 = =1H E
i1 3
116
E 100 i
(b) i0 = = = 10 A But i = 0
R 10 2
t Rt
- i0 Ê - ˆ
(c) i = i0(1 - e t ) = i0 Á1 - e L ˜
2 Á ˜
-
1 Ë ¯
i = 10 (1 - e 0.2 )
-
Rt
1
-5 e L =
= 10 (1 - e ) = 9.93 A 2
23. (a) Power delivered by the battery, L 1.25 ¥ 10-3
Rt
t = ln 2 = ¥ 0.693
E2 - R 50.0
P = Ei = (1 - e L )
R = 17.3 ¥ 10-6 = 17.3 ms
(3.24)2
12.8 ¥ 0.278 1 1 Ê1 ˆ
= (1 - e 3.56 ) (b) U = Li2 = Á L i02 ˜
12.8 2 2 Ë2 ¯
1
= 0.82 (1 - e -1 ) = 0.518 W i= i0
2
= 518 mW Rt ˆ
Ê - i
(b) Rate of dissipation of energy as heat i0 Á1 - e L ˜ = 0
Rt Á ˜ 2
E2 - Ë ¯
P2 = i2 R = (1 - e L )2 Rt
R - 2 -1
fi e L =
= 0.82 (1 - e -1 )2 = 0.328 W 2
= 328 mW L 2
fi t = ln
(c) Rate of storage of magnetic energy R 2 -1
P1 = P - P2 = 190 mW = 30.7 ms .
di
24. E = VL + VR = L + iR 26. Steady state current through the inductor
dt L, r
L R
i0
VL VR
K R

(a) Initially, i = 0 S
E
di E 6.00
\ = = = 2.40 A/s E
dt L 2.50 i0 =
r
(b) When, i = 0.500 A
di E - iR 6.00 - 0.500 ¥ 8.00 When the switch S is open
= = L
dt L 2.50 t=
R+ r
= 0.80 A/s
E ÊÁ -
Rt ˆ (a) i = i0 e -t / t
(c) i = 1-e L ˜ Ê (R + r) ˆ
R ÁË ˜
¯ E - ÁÁË L ˜˜¯
fi e i=
8.00 ¥ 0.250 ˆ r
6.00 ÊÁ -
˜
= 1-e 2.5
(b) Amount of heat generated in the solenoid
8.00 Á ˜ • •
Ë ¯ H = Ú i2r dt = i02r Ú e -2t / t dt
-0.8 0 0
= 0.750 (1 - e ) = 0.413 A
E 6.00 E 2 Ï t -2 t / t • ¸
(d) i0 = = = 0.750 A = Ì- [ e ]0 ˝
R 8.00 r Ó 2 ˛
Ê Rt ˆ ( R + r ) E2
- =
25. (a) i = i0 Á1 - e L ˜ 2rL
Á ˜
Ë ¯
117
When the switch S is open, current i2 flows
in the circuit in clockwise direction and is
given by
27. At any instant of time,
5 mH i2 = i0e -t / t
E
i1 Here, i2 =
R2
10 mH L
t=
R1 + R2
i Ê R1 + R 2 ˆ
20 V E - ÁÁË L ˜˜ t
i2 = e ¯
R2
5W
12 -10t
di1 di = e = (6 e -10t ) A
L1 = L2 2 2
dt dt
29. For current through galvanometer to be
fi L1i1 = L2i2 zero,
i1 = 2i2 …(i) L1,R1 R3
P i1
In steady state,
G
inductors offer zero resistance, hence i1
20 L2,R2 R4
i= =4A
5 i2 Q i2
But i1 + i2 = i
4 8
i2 = A, i1 = A
3 3 K
E
28. When the switch is closed,
i i2
VP = VQ
di1 di
L1 + i1 R1 = L2 2 + i2 R2 …(i)
dt dt
i1 L
Also, i1 R3 = i2 R4 …(ii)
E
R1 From Eqs.(i) and (ii),
di di
L1 1 + i1 R1 L2 2 + i2 R2
R2 dt = dt …(iii)
S
i1 R3 i2 R4
In the steady state,
E di1 di2
(1 - e - R2 t / L )
i2 = = =0
R2 dt dt
di2 E - R2 t / L R1 R2 R R
= e \ = fi 1 = 3
dt L R3 R4 R2 R4
Potential difference across L Again as current through galvanometer is
di always zero.
V + L 2 = E e - R2 t / L = (12e -5t ) V
dt i1
= constant
i2
i2
di1 / dt
or = constant
di2 / dt
i1 L
di1
E
R1 or dt = i1 …(iv)
di2 i2
R2 dt
S i2 From Eqs. (iii) and (iv),
118
L1 R3 R1
= =
L2 R4 R2 q0 5.00 ¥ 10-6
32. (a) V0 = =
30. (a) In LC circuit C 4 ¥ 10-4
Maximum electrical energy = Maximum = 1.25 ¥ 10-2 V= 12.5 mV
magnetic energy
1 1 (b) Maximum magnetic energy = Maximum
fi CV02 = Li02 electric energy
2 2
2 2 1 2 q02
ÊV ˆ Ê 1.50 ˆ Li0 =
L = C ÁÁ 0 ˜˜ = 4 ¥ 10-6 Á ˜ 2 2C
i Á 50 ¥ 10-3 ˜
Ë 0¯ Ë ¯ q
fi i0 = 0
= 3.6 ¥ 10-3 H LC
fi L = 3.6 mH 5.00 ¥ 10-6
fi i0 = = 8.33 ¥ 10-4 A
1 0.090 ¥ 4 ¥ 10-4
(b) f =
2p LC
1 (c) Maximum energy stored in inductor,
= 1
2 ¥ 3.14 3.6 ¥ 10 -3
¥ 4 ¥ 10 -6 = L i02
2
= 1.33 ¥ 103 Hz 1
= ¥ 0.0900 ¥ ( 8.33 ¥ 10-4 )2
= 1.33 kHz 2
(c) Time taken to rise from zero to maximum = 3.125 ¥ 10-8 J
value, (d) By conservation of energy,
t=
T
=
1
=
1 q2 1 2 1 2
+ Li = Li0
4 4f 4 ¥ 1.33 ¥ 103 2C 2 2
i0
= 3 ¥ 10-3 s = 3 ms. But i =
2
31. (a) w = 2pf = 2 ¥ 3.14 ¥ 103
q2 3 2
= 6.28 rad/s = Li0
2C 8
1 1 -3
T = = 3 = 10 s = 1 ms i 3
f 10 q = 0 3 LC = q0
2 2
(b) As initially charge is maximum, (i.e.., it is 1.732
extreme position for charge). = ¥ 5.00 ¥ 10-6
2
q = q0 cos w t = 4.33 ¥ 10-6 C
q0 = CV0 = 1 ¥ 10-6 ¥ 100 1 1 Ê1 ˆ
Um = Li2 = Á Li02 ˜
= 10-4 2 4 Ë2 ¯
\ q = [10 cos (6.28 ¥ 103 ) t ] C.
-4
= 7.8 ¥ 10-9 J
1 1 1
(c) w = 33. (a) w = =
LC LC 2.0 ¥ 10 ¥ 5.0 ¥ 10-6
-3
1 1
fiL= 2 =
w C (6.28 ¥ 103 )2 ¥ 10-6 = 104 rad/s
Ω di Ω
Ω Ω = w2Q
= 2.53 ¥ 10-3
Ω dt Ω
fi L = 2.53 mH
= (104 )2 ¥ 100 ¥ 10-6 = 104 A/s
(d) In one quarter cycle, entire charge of the
(b) i = w Q02 - Q2
capacitor flows out.
q 4CV = 104 (200 ¥ 10-6 )2 - (200 ¥ 10-6 )2 = 0
<i>= =
t T
-6 (c) i0 = wQ0 = 104 ¥ 200 ¥ 10-6 = 2 A
4 ¥ 10 ¥ 100
= = 0.4 A (d) i = w Q02 - Q 2
10-3
119
At a distance x from centre of the region,
i0 Magnetic flux linked with the imaginary
fi = w Q02 - Q 2
2 loop of radius x
w Q0
fi = w Q02 - Q 2 fm = p x 2B
2 - dfm dB
3 1.73 ¥ 200 ¥ 10-6 e= = - p x2
Q= Q0 = dt dt
2 2 Induced electric field,
= 173 mC e 1 dB
E= = x
34. As initially charge is maximum 2p x 2 dt
q = q0 cos w t At a,
and |i| = i0 sin w t 1 dB
E= r , towards left.
1 1 4 dt
where, w = =
LC 3.3 ¥ 840 ¥ 10-6 At b ,
1 dB
ª 19 rad./s E= r , upwards.
2 dt
i0 = wq0 = 19 ¥ 105 ¥ 10-6
At c,
ª 2.0 ¥ 10-3 A = 2.0 mA
E =0
At t = 2.00 ms
36. Inside the solenoid,
q2 q2
(a) Ue = = 0 (cos 2 wt ) B = m 0ni
2C 2C
dB di
(105 ¥ 10-6 )2 = m 0n
= [cos 2 (38 rad)] dt dt
2 ¥ 840 ¥ 10-6
Inside the region of varying magnetic field
Ue = 6.55 ¥ 10-6 J = 6.55 mJ 1 dB 1 di
E= r = m 0nr
1 1 2 dt 2 dt
(b) Um = Li2 = Li02 (sin w t )
2 2 (a) r = 0.5 cm = 5.0 ¥ 10-3 m
1 1 di
= ¥ 3.3 ¥ (2 ¥ 10-3 )2 sin 2(38 rad ) E = m 0rn
2 2 dt
= 0.009 ¥ 10-6 J = 0.009 mj 1
= ¥ 4p ¥ 10-7 ¥ 5.0 ¥ 10-3 ¥ 900 ¥ 60
q2 1 2
(c) U = 0 = Li02
2C 2 = 1.7 ¥ 10-4 V/m
= 6.56 ¥ 10-6 J = 6.56 mJ (b) r = 1.0 cm = 1.0 ¥ 10-2 m
35. As the inward magnetic field is increasing, 1 di
E= rn
induced electric field will be anticlockwise. m0 dt
× × × × × × × × × × × ×
× × × × × × × × × × × × 1
× × × × × × × × × × × × = ¥ 4p ¥ 10-3 ¥ 5.0 ¥ 10-3 ¥ 900 ¥ 60
× × × × × × ×E × × × × × 2
× × × × × × × × × × × × = 3.4 ¥ 10-4 V/m
× × × × × × × × × × × ×
× × × × × × × × × × × ×
× × × × × × × × × × × ×
× × × × × × ×x× × × × ×
× × × × × × × × × × × ×
× × × × × × × × × × × ×
× × × × × × × × × × × ×
× × × × × × × × × × × ×
× × × × × × × × × × × ×
× × × × × × × × × × × ×
120

AIEEE Corner
Objective Questions (Level 1)
di 1 2 1 L 2
1. V = L 8. Li0 = CV02 fi i0 =2 ¥
dt 2 2 C 4 ¥ 10-6
[ V ][ T ] [ ML2 T -3 A -1 ] [T ]
[L] = = = 2 ¥ 103 V
[ i] [A ]
1
9. e = B l2w, is independent of t.
= [ ML2 T -2 A -2 ] 2
2. M µ n1n2 df Df
10. |e| = =
3. Both will tend to oppose the magnetic flux dt t
changing with them by increasing current in Df =|e|t = iRt
opposite direction. = 10 ¥ 10-3 ¥ 0.5 ¥ 5
4. Moving charged particle will produced = 25 ¥ 10-3 Wb
magnetic field parallel to ring, Hence
= 25 mWb.
fm = 0
11. As inward magnetic field is increasing,
Velocity of particle increases continuously induced electric field must be anti-clockwise.
due to gravity.
Hence, direction of induced electric field at P
5. Induced electric field can exist at a point will be towards and electron will experience
where magnetic field is not present, i.e., force towards right (opposite to electric
outside the region occupying the magnetic field).
field. 12. f = at ( t - t ) = att - at 2
6. At, t = 1 s df
|e| = = at - 2at
2W 4V 2H 2F dt
a b |e| at - 2at
i – + i= =
a R R
2
t ( at - 2 at )
q = 4t 2 = 4 C t 2
H = Ú i R dt = Ú dt
dq 0 0 R
i= = 8t = 8 A
dt 3 t
1 È ( at - 2 at ) ˘
di = Í ˙
= 8 A/s R ÍÎ 3 ¥ ( - 2a ) ˙˚
dt 0
1
di d 2q = [ - a t - a 3t 3 ]
3 3
As, = = Positive - 6 Ra
dt dt
Charge in capacitor is increasing, current i a 2t 3
=
must be towards left. 3R
di q
Vab = - 2I + 4 - L -
dt C di
4 13. E = - L
= - 2 ¥ 8 + 4 - 2 ¥ 8 _ = - 30 V dt
2 di
di d 14. VBA = - L + 15 - iR
7. |e| = M =M ( i0 sin w t ) dt
dt dt = - 5 ¥ 10-3( - 103 ) + 15 - 5 ¥ 1
= w Mi0 cos w t = 15 V
Maximum induced emf = w Mi0 di
15. = 10 A/s, at t = 0, i = 5A
= 100p ¥ 0.005 ¥ 10 dt
= 5p di
= 10 A/s
dt
121
di E 12
VA - VB = iR + L - E =0 25. i0 = = = 40 A
dt R 0.3
= 5 ¥ 3 + 1 ¥ 10 - 10 = 15 V 1 1
U0 = Li02 = ¥ 50 ¥ 10-3 ¥ (40)2
Ê di ˆ Ê d 2q ˆ q 2 2
16. Á ˜ = ÁÁ ˜
˜ = w2q0 = 0
Ë dt ¯ max dt LC = 40 J
Ë ¯ max t Rt ˆ
di
Ê - ˆ E ÊÁ -
17. V = L 26. i = i0 Á1 - e t ˜ = 1-e L ˜
Á ˜ RÁ ˜
dt Ë ¯ Ë ¯
Rt
18. fm = BA cos q di E - L
df dq = e
fi e = - m = BA sin q dt L
dt dt di -
Rt
dq VL = L = Ee L
fi iR = BA sin q dt
dt
dq dq at t = 0
fi R = BA sin q
dt dt VL = E = 20 V
BA at t = 20 ms
fi dq = sin q dq
R R ¥ 20 ¥ 10 -3 R
- -
BA 3p/ 2 VL = Ee L fi 5 = 20e 50L
R Úp/ 2
fi q= sin q dq = 0
R
= ln 4 fi R = (100 ln 4) W
Æ ^ Æ ^ ^ ^ 50L
19. A = ab k, B = 20t i + 10t 2 j + 50 k
|e| 1 df 1 dB
ÆÆ 27. |i| = = ◊ = NA
fm = B ◊ A = 50 ab R R dt R dt
df 10 ¥ 10 ¥ 10-4
e = - m =0 = ¥ 108 ¥ 10-4
dt 20
20. E = Vb + iR =5A
Vb = E - iR = 200 - 20 ¥ 1 . 5 = 170 V 28. In the steady state, inductor behaves as
V N 1 short circuit, hence entire current flows
21. s = s fi Vs = ¥ 290 = 10 V through it.
Vp N p 2
ip 29. fm = AB cos q
N
= s But, q = 90∞
is Np
\ fm = 0
Np
fi is = ip = 2 ¥ 4 = 8 A |e| 1 dfm
Ns 30. i = =-
R R dt
22. Vr = 0, hence magnetic flux linked with the dq - nBA d
fi = (cos q)
coil remain same. dt R dt
- df nBA dq
\e= =0 = sin q
dt R dt
1 nBA
23. s = at 2 fi dq = sin q dq
2 R
Due to change in magnetic flux linked with nBA p 2nBA
R Ú0
Q1 = sin q dq =
the ring, magnet experiences an upward R
force, hence, nBA 2p
R Ú0
Q2 = sin q dq = 0
a<g
1 2 Q2
s < gt fi s < 5 m \ =0
2 Q1
di 31. According to Lenz’s law, induced current
24. VA - VB = L =-at
dt always opposes the cause producing it.
122
Ê -
t
ˆ EÊ -
Rt ˆ df
32. i = i0 Á1 - ˜ = Á1 -
e t e L ˜ 37. E = -
Á ˜ RÁ ˜ dt
Ë ¯ Ë ¯
5 ¥2 ˆ 38. Magnetic flux linked with the coil does not
15 ÊÁ -
change, hence
= 1 - e 10 ˜ = 3 (1 - e -1 )
5 Á ˜ e - 1 df
Ë ¯ i= = ◊ =0
Ê 1ˆ R R dt
= 3 Á1 - ˜ A 1 Ê l ˆ
Ë e¯ 39. e = Blv cos q = Bl2w cos q ÁQ v = w˜
2 Ë 2 ¯
33. Velocity of AB is parallel to its length.
As|cos q|varies from 0 to 1
34. Velocity of rod is parallel to its length. 1
e varies from 0 to B l2w.
35. Vc - Va = Vc - Vb = BRV 2
and Va - Vb = 0
36. Induced current always opposes the cause
producing it.

JEE Corner
Assertion and Reason
1. Magnetic flux linked with the coil is not 8. L = m rm 0n 2lA, for ferromagnetic substance,
changing with time, hence induced current mr > g
is zero.
and L does not depends on i.
2. Both Assertion and Reason are correct but
9. As soon as key is opened
Reason does not explain Assertion.
a
3. Induced electric field is non-conservative but
can exert force on charged particles. R1

4. i = 2t - 8 R2
di E
=2 L
dt
di
Va - Vb = L = 2 ¥2 = 4 V i
dt b
Ê di ˆ E
5. Á ˜ = ( imax ) w = 1 ¥ 2 = 2 A/s i = i0 =
Ë dt ¯ max R1
6. Va - Vb = Vc - Va 10. Inductors oppose change in current while
Vc > Va > Vb resistor does not.
7. Fact.

Objective Questions (Level 2)


1. By conservation of energy
1 1
L i02 = mv02 iR iC
2 2
m
i0 = v0
k
+
R E q C

2. Wire AB behaves as a cell of emf, E = Blv


123
E Blv \ fm = B tan q x 2
iR = =
R R df dx
e = - m = - 2B tan q x
ic = 0 dt dt
1 1 = 2B tan q vx
Uc = CE2 = CB2l2v2
2 2 R = r l = r (2x tan q)
3. Apply Fleming’s left hand rule. where, r = resistance per unit length of the
4. For SHM, conductor.
v = w A cos wt e Bv
\ i= = = constant.
e = Blv = Blw A cos wt R r
Ï T 10. fm = BA cos q = BA cos wt
Ô e0 cos wt for nT < t < (2n - 1)
2 df
e =Ì e = - m = w BA sin wt
(2n - 1)T dt
Ô- e0 cos wt for > t > nT 2
Ó 2 But A=b
5. fm = BA \ e = b2Bw sin wt
At any instant when wires have moved 11. Induced emf
through a distance x, dB
e = a2 = (1)2 ¥ 2 ¥ 10-3
A = ( a + 2 x )2 dt
fm = B( a + 2x )2 = 2 ¥ 10-3 V
df dx W = qe = 1 ¥ 10-6 ¥ 2 ¥ 10-3
|e| = m = 4B( a + 2x )
dt dt = 2 ¥ 10-9 J
= 4B( a + 2x ) v0 12. In the steady state, current through
|e| 4B ( a + 2x ) v0 Bv0 capacitor = 0.
|i| = = =
R l ¥ 4 ( a + 2x ) l 20
iL = =4A
5
6. A = l2
dA dl Ê dl ˆ f1 = 0, f2 = iL L = 4 ¥ 500 ¥ 10-2
= 2l = - 2 la Áa = - ˜
dt dt Ë dt ¯ = 2 Wb
fm = BA Df = f2 - f1 = 2 Wb.
df dA 1 1 Ê1 ˆ
e=- m =-B = 2Bla 13. Li2 = Á Li02 ˜
dt dt 2 2 Ë2 ¯
i0
at l=a i=
2
e = 2a aB t
Ê - ˆ i
7. At this instant, direction of motion of wire i0 Á1 - e t ˜ = 0
PQ is perpendicular to its length. Á ˜ 2
Ë ¯
\ e = Blv 2 -1
e -t / t =
8. q = CV = CBlv 2
= 20 ¥ 10-6 ¥ 0.5 ¥ 0.1 ¥ 0.2 Ê 2 ˆ˜
t = t ln Á
= 0.2 mC Á 2 - 1˜
Ë ¯
Plate A is positive while plate B is negative. L Ê 2 ˆ˜
Ê1 ˆ = ln Á
9. fm = BA = B Á lx ˜ R Ë 2 - 1 ˜¯
Á
Ë2 ¯
m i
l
14. B = 0
2pa
v m iqv
q x F = qvB = 0
2pa
15. Consider an elementary section of loop of
But l = 2x tan q width dx at a distance x from wire AB
124
B 18. Induced current opposes change in magnetic
flux.
P Q 19. VL = E - iR
i 20. The rod can be assumed as a cell of emf
C
E = Blv
a
S dx R The equivalent circuit is shown in figure,
b 2W i E
A x 2W
m i 4W E 12W fi
dfm = BdA = 0 C dx
2px
m iC b dx m 0iC b 3W
fm = 0 Ú = ln
2p a x 2p a E Blv 0.50 ¥ 0.25 ¥ 4
f m C b i= = =
M = m = 0 ln 2+3 5 5
i 2p a
= 0.1 A
16. From previous question
P 21. Outside the region of magnetic field, induced
A R electric field,
C r 2 dB Br 2
E= =
x dx 2 R dt 2R
i E F
F = qE
v 1
a t = qER = qBr 2
B 2
D
b 22. VA - V0 = B (2 R)V
Q
fi VA - V0 = 2 BRV
m 0iy b hL L
fm = ln 23. L1 = , L2 =
2p a h+1 h+1
b
m 0i ln L1 R1
dfm a dy
e= =
dt 2p dt
m 0iv b
e= ln L2
2p a R2
e m 0iv b
i= = ln E
R 2pR a
Consider an elementary portion of length dx hR hR
R1 = , R2 =
of the rod at a distance, x from the wire PQ. h+1 h+1
Force on this portion, 1 1 1
= +
dF = i dxB Le L1 L2
m 2i h+1 h+1
= i 0 ◊ dx = +
4p x hL L
m0 b dx
(h + 1) È 1 1 ˘
4p Úa x
F=i ◊ 2i
= +
L ÍÎ h 1 ˙˚
È m iv b ˘ m i b
= Í 0 ln ˙ 0 ln hL
Î 2 pR a ˚ 2 p a Le =
2 ( h + 1 )2
1 È m 0iv b ˘
= ln ˙ hR L L
vR ÍÎ 2p a˚ Similarly, Re = 2
\ t= e =
( h + 1) R e R
1 dB
17. E = r fi E µr
2 dt
125
24. i = i0e -t / t When switch is thrown to position 2.
Bi0 = i0e -T / t E E
i1 = , i2 = -
T R R
t= 1 2 1 Ê1
Ê1ˆ 2ˆ
ln Á ˜ 29. Li = Á L i0 ˜
Ë B¯ 2 4 Ë2 ¯
L i
25. Given, i02 R = P, =t i= 0
R 2
t
Ê - ˆ i
when, choke coil is short circuited, i0 Á1 - e t ˜ = 0
Á ˜ 2
Total heat produced = Magnetic energy Ë ¯
stored in the choke coil t = t ln 2
1 2 1 ÊPˆ 1 L
= Li0 = ( Rt )Á ˜ = Pt fi t = ln 2
2 2 Ë R¯ 2 2
Rt
-
26. i = i0 e L 30. At the moment when switch is thrown to
position 2,
For current to be constant current in capacitor = current in inductor
i = i0 just before throwing the switch to position 2,
Rt
- E
e L =1 fi ic =
R
Rt
= 0 = not possible. 31. Initially, inductor offers infinite resistance,
L hence,
27. To final time constant, short the battery and di
i = 0 and = maximum
find effective resistance in series with dt
inductor
R \ E = VL + VC + VR
R/2
But VC = VR = 0
L
fi VL = E

R 32. Same as Q.12 objective Questions (Level 2).
L 33. Let V0 = Potential of metallic rod,
R VB - V0 = B (2 R)V = 2 BR2w …(i)
Re =
2 V0 - VC = B (2 R)V = 2 BR2w …(ii)
L 2L
t= = Adding Eqs. (i) and (ii), we get
Re R
VB - VC = 4 BwR2
28. When switch is at position 1. 34. e = Blvc
In steady state, v1 + v2
1 i2 vc = dx2
2 2 v2
1
\ e= Bl (v1 + v2 )
R 2
or v1
dx1
dA
E L e=B
dt
1
L dA = l ( dx1 + dx2 )
2
i1 1 Ê dx1 dx2 ˆ
R e = Bl Á + ˜
2 Ë dt dt ¯
1
E = Bl (v1 + v2 )
i1 = 2
R
i2 = 0
126
35. Initially, capacitor offer zero resistance and |e| A dB
37. |i| = =
inductor offers infinite resistance. R R dt
Effective circuit is given by B A B [(2b)2 - pa 2 ]
= 0 = 0
R R R
B0(4b2 - pa 2 )
R =
R
R
As inward magnetic field is increasing, net
current must be anticlockwise. Hence
E = 5V current in inner circle will be clockwise.
R
38. From Q. 48 Subjective Questions (Level 1).
m ai a
fm = 0 ln ÊÁ1 + ˆ˜
R fl 2p Ë x¯
R/3 5R/6=5 W
Case 1
i x = b, a = a
m ai a

fm1 = 0 ln ÊÁ1 + ˆ˜
E =5V E =5V 2p Ë b¯
m 0 ai Ê b + a ˆ
R/2 = ln Á ˜
2p Ë b ¯
E
i=
=1A Case 2
R
t x=b-a
E ÊÁ - R2 C ˆ˜
R t
E ÊÁ - 1 ˆ
36 . i1 = 1 - e L ˜ , i2 = e a=a
R1 ÁË ˜
¯ R2 Á ˜
m 0 ai Ê a ˆ˜
Ë ¯
fm2 = ln Á1 +
2p Á b - a ˜¯
Ë
4W 5W
m 0ai Ê b ˆ
= ln Á ˜
E = 20V 2p Áb - a˜
5W Ë ¯
L = 10 mH fm2 - fm1
K <e>=-
t
6W C = 0.1mF <e> fm - fm1

<e>= =- 2
i1 i2
R Rt
fm2 - fm1
R1= 10W q=<i>t=-
R2= 10W R
E m 0 ai È Ê b + a ˆ Ê b ˆ˘
L = 0.1mH =- Í ln Á ˜ - Á ln ˜˙
C = 0.1mF 2pR Î Ë b ¯ Ë b - a ¯ ˚
- m 0ai Ê b ˆ˜
= ln Á 2
2pR Á b - a2 ˜
i = i1 + i2 Ë ¯
R t t
m ai Ê b ˜ ˆ
E ÊÁ - 1 ˆ E - R2 C |q| = 0 ln Á 2
= 1-e L ˜ + e 2pR Á b - a2 ˜
R1 Á ˜ R2 Ë ¯
Ë ¯
at t = 10-3 ln 2 39. Magnetic flux linked with the coil.
m n iA
Ê 10 ¥ 10 -3 ln 2 ˆ 10 3 ln 2 fm = nBA = 0
20 Á -
10 ¥ 10 -3 ˜ 20 -
-3 2r
i= Á1 - e ˜+ e 10 ¥ 0.1 ¥ 10 dfm
10 Á ˜ 10 |e| =
Ë ¯ dt
Ê 1ˆ Ê1ˆ dfm
= 2 Á1 - ˜ + 2 Á ˜ = 2 A fi iR =
Ë 2¯ Ë2¯ dt
127
dq df 1 dB
R = m fi dq = dfm = 2 T/s, A = 0.2 ¥ 0.4 = 0.08 m 2
dt dt R dt
m 0nA i m 0n iA 0.08
2rR Ú0
q= di = \ i= ¥ 2 = 16 A [Q R = r ¥ ( b + 2l)]
2rR 1 ¥ 1.0
40. Induced electric field inside the region of As outward magnetic field is increasing,
varying magnetic fields, induced current must be clockwise.
1 dB 1 dA dB dB
E= r = r(6t 2 + 2x ) = 3 r ( t 2 + x ) V/m 48. e = B + A = Blv + A
2 dt 2 dt dt dt
R
At, t = 2.0 s and r = = 1.25 cm At t = 2 s,
2
B = 4 T, A = 0.2 ¥ (0.4 - vt ) = 0.06 m 2
= 1.25 ¥ 10-2 m
v = 5 cm/s = 0.05 m / s
E = 3 ¥ 1.25 ¥ 10-2 ¥ (4 + x )
\ e = - 4 ¥ 0.2 ¥ 0.05 + 0.06 ¥ 2
= 0.3 V/m
= - 0.04 + 0.12 = 0.08 V
F = eE = 1.6 ¥ 10-19 ¥ 0.3 e
49. F = ilB = lB
= 48 ¥ 10-21 N R
1 dB 0.08
41. E = r fi E µr = ¥ 0.2 ¥ 4
2 dt 1 ¥ 0.8
42. As inward magnetic field is increasing, = 0.008 N
induced electric field must be anticlockwise.
df dB 50. When terminal velocity is attained,
43. e = m = pa 2 = pa 2B0 power delivered by gravity = power dissipated
dt dt
e 1 in two resistors
44. E = = aB0
2pa 2 mgv = 0.76 + 1.2
45. t = qEa = ia 1.96
v= = 1 m/s
1 0.2 ¥ 9.8
q¥ aB0a
qEa 2
a= = 51. e = Blv = 0.6 ¥ 1 ¥ 1 = 0.6 V
ma 2 ma 2 e2
qB0 P1 =
= R1
2m
e 2 (0.6)2
46. P = tw = t( at ) = ia 2 ◊ t fi R1 = = = 0.47 W
P1 0.76
q2B02
= ma 2 ¥ t e2
m m2 52. P2 =
R2
At t = 1 s
q2B02a 2 e 2 (0.6)2
P= fi R2 = = = 0.3 W
4m P2 1.2
e A dB
47. i = = ◊
R R dt
128

More than One Correct Options


Ê1ˆ 1 e 80
1. e = B Á ˜v = BLv i= = = 20 A
Ë2¯ 2 R 4
By Fleming’s left hand rule, P must be q = it = 20 ¥ 0.1 = 2 C
positive w.r.t. Q. Current is not given as a function of time,
2. dfm = BdA = Ba dx hence heat produced in the coil cannot be
determined.
I
7. In LC oscillations,
dx x 1 w 1
w= ,f = =
LC 2p 2p LC
1
T = = 2p LC
f
q
i0 = w q0 = 0
a x LC
Ê di ˆ q0
Á ˜ = wq0 =
Ë dt ¯ max LC
m 0a i
= dx Ê di ˆ q
2px ( VL )max = L Á ˜ = 0
m ai Ë ¯ max C
dt
fm = 0 ln 2
2pi 8. If magnetic field increases, induced electric
fm m 0a field will be anticlockwise and vice-versa.
M= = ln 2
i 2p 9. q = 2t 2
If the loop is brought close to the wire, upward dq
i= = 4t
magnetic flux linked with the loop increases, dt
hence induced current will be clockwise. di
= 4 A/s
3. f = Li = Henry-Ampere. dt
V V dt Volt -second dq
L= = = As = Positive
di / dt di Ampere dt
L Charge on the capacitor is increasing, hence
4. t = =1s current flows from a to b.
R
E 1H 2F 4W
i = i0(1 - e - t / t ) = (1 - e - t / t ) a b c d
R i + –
= 4 (1 - e -t )
t = 1 s, q = 2 C, i = 4 A
At t = ln 2, di
= 4 A/s
i = 2A dt
di
Power supplied by battery, P = EI = 16 J/s. Va - Vb = L = 1 ¥4 = 4 V
dt
Rate of dissipation of heat in across resistor q 2
Vb - Vc = = = 1 V
= i2 R = 8 J/s c 2
VR = iR = 4 V Vc - Vd = iR = 4 ¥ 4 = 16 V
Va - Vd = 4 + 1 + 16 = 21 V
Va - Vb = E - VR = 4 V 1
10. Va - Vb = Bl2w
5. In both the cases, magnetic flux linked with 2
increases, so current i2 decreases in order to 1
Vc - Vb = Bl2w
oppose the change. 2
6. f1 = BA = 4 ¥ 2 = 8 Wb, f2 = 0 Va - Vc = 0
f -f 8 [Direction of velocity of rod a-c is parallel to
e= 2 1 = = 80 V length a-c]
t 0.1
129

Match the Columns


[F] [ ML T -2 ] E 9
1. [ B] = = 4. i1 = = = 1.6 A
[ i][ l] [ A ][L ] R1 6
i1 i2
= [ ML0 T -2 A -1 ]
[ V ][ dt ] [ ML2 T -3 ][T ] i1
[L] = = L
[ di] [A ]
E R1
= [ ML2 T -2 A -2 ]
[ LC ] = [T 2 ] R2

[ fm ] = [ B][ S ] R2 t
= [ ML T - 2 A -1 ] [L2 ] = [ ML 2 T - 2 A -1 ]
0 E -
i2 = (1 - e L ) = 3 (1 - e -t / 3 )
-t / t R2
2. i = i0 (1 - e )
L At t = (ln 2) s
t==1s q
R VL = E - i2 R2 = qe - t / 3 =
E 21/ 3
i0 = =5A
R VR 2 = i2 R2 = q (1 - e - t / 3 )
VR = iR = E (1 - e -t ) Ê 1 ˆ
= q Á1 - 1/ 3 ˜
VL = E - VR = Et - t Ë 2 ¯
At t = 0, VR1 = i1 R1 = 9 V
VL = E = 10 V, VR = 0 Vbc = VL + VR2 = 9 V
at t =1s (a Æ s), (b Æ s), (c Æ p), (d Æ p).
Ê 1ˆ 5. Induced emf
VL = E (1 - e -1 ) = Á1 - ˜ 10 V
Ë e¯ f(Wb)
10
VR = V
e
3. In LC oscillations,
1 1
w= = = 2 rad/s 4
LC 1

4
q0 = 4 C
2 t
i0 = wq0 = 8 A
Ê di ˆ |e| = slope of f - t graph
Á ˜ = w2q0 = 16 A/s.
Ë dt ¯ max 4 -0
= =2V
When, q = 2 C 2 -0
q |e| 2
VL = VC = = 8V |i| = = =1A
C R 2
Ê di ˆ 1 Ê di ˆ
When, Á ˜= Á ˜ = 8 A/s. |q| = |i|t = 1 ¥ 2 = 2 C
Ë dt ¯ 2 Ë dt ¯ max
As current i is constant
di
VC - VL = L =1 ¥8 = 8V H = i2 Rt = (1)2 ¥ 2 ¥ 2 = 4J
dt
25 Alternating Current
Introductory Exercise 25.1
VDC 100 XL = XC
1. R= = = 10 W
I 10 1
fi wL =
V 150 wC
Z = AC = = 15 W
I 10 1 1
fi L= 2 =
X L = Z 2 - R2 = (15)2 - (10)2 w C ( 2 pf )2 C
=5 5 W 1
=
XL XL 5 5 (360)2 ¥ 10-6
L= = =
w 2 pf 2 ¥ 3.14 ¥ 50 ª 7.7 H
ª 0.036 H As XL = XC
\ VL = IX L = 50 5 V \ Z =R
V 120
= 111.8 V I= = =6A
Z 20
2. For phase angle to be zero,

Introductory Exercise 25.2


1. Resonating frequency, 2. Resistance of arc lamp,
1 1 VDC
wr = = R=
LC -6 I
0.03 ¥ 2 ¥ 10
40
104 = =4W
= 10
6
Impedance of series combination,
wr 104
fr = = V 200
2 p 2 ¥ 3.14 ¥ 6 Z = AC = = 20 W
I 10
ª 1105 Hz R 4 1
Power factor = cos f = = =
Phase angle at resonance is always 0∞. Z 20 5
131

AIEEE Corner
Subjective Questions (Level-1)
1. (a) X L = w L = 2 pfL 3. (a) Power factor at resonance is always 1,
= 2 ¥ 3.14 ¥ 50 ¥ 2 R
as Z = R, Power factor = cos f = = 1.
= 628 W Z
X I 0 E0 cos f E20
(b) X L = w L fi L = L (b) P = =
w 2 2R
XL 2 (150)2
= = = = 75 W
2 pf 2 ¥ 3.14 ¥ 50 3 ¥ 150
= 6.37 mH (c) Because resonance is still maintained,
1 1 average power consumed will remain
(c) X C = = same, i.e., 75 W.
wC 2 pfC
1 4. (a) As voltage is lag behind current,
= inductor should be added to the circuit
2 ¥ 3.14 ¥ 50 ¥ 2 ¥ 10-6
to raise the power factor.
= 1592 W = 1.59 kW R
1 1 (b) Power factor = cos f =
(d) X C = fiC= Z
wC w XC R 60 250
fi Z= = = W
1 cos f 0.720 3
= = 1.59 mF
2 ¥ 3.14 ¥ 50 ¥ 2
X C = Z 2 - R2
2 2
2. (a) Z = R + ( X L - X C ) 2
250 ˆ
2 = ÊÁ ˜ - (60)
2
Ê 1 ˆ Ë 3 ¯
= R2 + ÁÁ w L - ˜
Ë wC ˜¯ = 58 W
2 1
Ê 1 ˆ C=
= (300)2 + ÁÁ 400 ¥ 0.25 - ˜ w XC
400 ¥ 8 ¥ 10-6 ˜
Ë ¯ 1
=
= 367.6 W 2pf X C
V0 120 1
I0 = = = 0.326 A =
Z 367.6 2 ¥ 3.14 ¥ 50 ¥ 58
X - XC
(b) f = tan -1 L = 54 mF
R
For resonance,
-1 212.5
= tan ª - 35.3 ∞ 1
300 wr =
LC
As X C > X L voltage will lag behind 1
current by 35.3∞. fi L= 2
wr C
(c) VR = I 0 R = 0.326 ¥ 300 = 97.8 V,
1
VL = I 0 X L = 32.6 V =
(2pf )2 C
VC = I 0 X C = 0.326 ¥ 312.5 1
fi L=
= 101.875 V ª 120 V (2 ¥ 3.14 ¥ 50)2 ¥ 54 ¥ 10-6
= 0.185 H
132
5. V ( t) = 170 sin (6280 t + p / 3) volt (b) w = 1000 rad/s
60
i ( t) = 8.5 sin (6280t + p / 2) amp. \ I= = 1.2 ¥ 10-2 A
1000 ¥ 5
V
170V (c) w = 10000 rad/s
60
\ I= = 1.2 ¥ 10-3 A
O
0.25 0.50 0.75 1.00 1.25 t (ms)
10000 ¥ 5

–170V 7. VR = (2.5 V ) cos [( 950 rad/s ) t ]


VR
(a) I =
i
R
(2.5 V ) cos [(950 rad/s) t ]
=
300
O
1 1 7 5 13 4 = ( 8 . 33 mA) cos [(950rad / s) t ]
— — — — — — t (ms)
12 3 12 6 12 3 (b) X L = w L = 950 ¥ 0.800
= 760 W
w 6280 (c) VL = I 0 X L cos ( w t + p / 2)
(b) f = = = 1000 Hz
2p 2 ¥ 3.14 fi VL = - I 0 X L sin w t
= - 6.33 sin [(950 rad / s) t ] V
= 1 kHz
p p p 8. Given, L = 0.120 H, R = 240 W, C = 7.30 mF,
(c) f = - = I rms = 0.450 A , f = 400 Hz
2 3 6
p 3 X L = w L = 2pfL
fi cos f = cos
=
6 2 = 2 ¥ 3.14 ¥ 400 ¥ 0.120
As phase of i is greater than V, current is = 301.44 W
leading voltage. 1 1
XC = =
wC 2pf C
(d) Clearly the circuit is capacitive in
nature, we have 1
=
R 2 ¥ 3.14 ¥ 400 ¥ 7.3 ¥ 10-6
cos f =
Z = 54.43 W
3 R 2 R R
fi = fiZ= R (a) cos f = =
2 Z 3 Z R + ( X L - X C )2
2
V 170
Also, Z= 0 = = 20 W 240
i0 8.5 =
(240) + (301.44 - 54.43)2
2
3
R= Z = 10 3 W = 0.697
2
Again, Z = R2 + X 2C fi X C = Z 2 - R2 f = cos -1(0697
. ) ª 458. ∞
(b) Z = R2 + ( X L - X C )2
= 400 - 300 = 10 W
1 1 1 = (240)2 + (301.44 - 54.43)2 = 344 W
XC = fi =
wC w X C 6280 ¥ 10 (c) Vrms = I rms Z = 0.450 ¥ 344
= 1592
. mF = 154.8 V ª 155 V
V V (d) Pav = Vrms I rms cos f
6. I = =
XL wL = 155 ¥ 0.450 ¥ 0.697 = 48.6 W
(a) w = 100 rad/s (e) PR = I 2rms R = (0.450)2 ¥ 240 = 48.6 W
60 (f) and (g) Average power associated with
\ I= = 0.12 A
100 ¥ 5 inductor and capacitor is always zero.
133

Objective Questions (Level-1)


1. In an AC circuit, cos f is called power 13. V = 10 cos 100p t
factor. 1
at t = s,
2. DC ammeter measures charge flowing in 600
the circuit per unit time, hence it 1
V = 10 cos 100p
measures average value of current, but 600
average value of AC over a long time is p 3
= 10 cos = 10 ¥ =5 3 V
zero. 6 2
3. Z = R2 + ( X L - X C )2 14. For purely resistive circuit f = 0.
2 1 1 1
Ê 1 ˆ 15. X C = fi X C µ or X C µ
2
= R + ÁÁ w L - ˜ wC w f
Ë w C ˜¯ X 1
16. sin f = =
Hence, for X L < X C , Z decreases with Z 3
increase in frequency and for X L > X C , Z -1 È 1 ˘
fi f = sin
increases with increase in frequency. ÍÎ 3 ˙˚
p 3p I E
4. As voltage leads current and f < , hence
2 17. f = , P = 0 0 cos f = 0
2 2
either circuit contains inductance and VDC
resistance or contains inductance, 18. R = = 100 W
I DC
capacitance and resistance with X L > X C .
V 100
5. RMS value of sine wave AC is 0.707 I 0 , Z = AC = = 200 W
but can be different for different types of I AC 0.5
AC’s. X L = Z 2 - R2 = 100 3 W
6. P = I v Ev cos f = 0 X L X L 100 3
2 2
L= = =
7. Z = R + ( X L - X C ) w 2pf 2p ¥ 50
V0 I 0 Ê 3ˆ
8. P = [V0 and I 0 are peak voltage and = ÁÁ ˜˜ H
2 Ë p ¯
current through resistor only] Vrms
V 19. I rms = = wCVrms
9. Vrms = 0 = 170 V XC
2
200 2
f =
w
=
120
ª 19 Hz = 100 ¥ 1 ¥ 10-6 ¥
2p 2 ¥ 3.14 2
I rms = 20 mA
10. Current is maximum at
1 1 20. V = VR2 + VL2 = (20)2 + (15)2
w = wr = =
LC 0.5 ¥ 8 ¥ 10-6 = 25 V, V0 = 25 2 V
I V cos f
= 500 rad/s. 21. P = 0 0 =0
2
I 0 E0 cos f
11. P = fi cos f = 0
2
100 ¥ 100 p fi f = 90∞
= cos ¥ 10-3 = 2.5 W
2 3 22. R is independent of frequency.
1 23. L is very high so that circuit consumes
12. X C = = • if w = 0, i.e., for DC
wC less power.
134
XL X 1 1 1
24. tan f = fi tan 45∞ = L = = = = 5 ms
R 100 4 f 4 ¥ 50 200
fi X L = 100 W 26. f = 60∞
w L = 100 W 1
4 ¥ 220 ¥
100 I 0 V0 cos f 2
L= ª 16 mH P= =
2 ¥ 3.14 ¥ 103 2 2
= 220 W
25. The minimum time taken by it in reaching
T
from zero to peak value =
4

JEE Corner
Assertion and Reasons
1. X C and X L can be greater than Z because 5. On inserting ferromagnetic rod inside the
2
Z = R + ( X L - X C) 2 inductor, X L and hence VL increase. Due
to this current will increase if it is lagging
Hence, VC = IX C and VL = IX L can be and vice-versa.
greater than V = IZ . 6. VR = VL = VC fi R = X L = X C
2. At resonance X L = X C , with further Hence, f = 0 and I is maximum.
increase in frequency, X L increases but
as Z = R2 + ( X L - X C )2 is minimum.
X C decreases hence voltage will lead
current. 7. I = I L - I C = 0
1
3. fr = , if dielectric slab is inserted 8. P = I 2rms R = ( 2)2 ¥ 10 = 20 W
2p LC
9. Inductor coil resists varying current.
between the plates of the capacitor, its
E0 wL
capacitance will increase, hence, fr will 10. I 0 = , f = tan -1
decrease.
2
R +wL 2 2 R
4. q = Area under graph 11. At resonance, current and voltage are in
1 1 V0
= ¥ 4 ¥ (2 + 3) + ¥ 4 ¥ (2 + 4) same phase and I 0 = . Hence, I 0
2 2 R
= 22 C depends on R.
q 22
Average current = = = 3.6 A
t 6

Objective Questions (Level-2)


Single Correct Options
1. For parallel circuit I = I 0 sin ( w t - f)
-1 È1 /
XL ˘ -1 4 Ê X ˆ
f = tan Í ˙ = tan = I 0 sin Á w t - tan -1 L ˜
Î 1 / R ˚ 3 Ë R ¯

= 53 ∞ 3. R = R1 + RL = 10 W
2. Current will remain same in series circuit X L = w L = 10 W,
given by 1
XC = = 10 W
wC
135
Reading of ammeter 8. For parallel RLC circuit,
V 10 2 I = I 2R + ( I C - I L )2
I rms = rms =
R 10 2 2
ÊV ˆ ÊV V ˆ
= 2 A = 1.4 A fi I = Á 0 ˜ + ÁÁ 0 - 0 ˜˜
Reading of voltmeter, Ë R ¯ Ë XC XL ¯
2
V = I rms RL = 5.6 V 1 Ê 1 ˆ
1 1 = V0 + ÁÁ wC - ˜
4. X C = = R2
Ë w L ˜¯
wC 2p ¥ 5 ¥ 103 ¥ 1 ¥ 10-6
p 9. V = VL2 + VR2 = 72.8 V
= 100 W VL 2
f = tan -1 = tan -1ÊÁ ˆ˜
V 200 VR Ë7 ¯
IR = = = 2 A,
R 100 10. Clearly P is capacitor and Q is resistor,
V 200
IC = = =2 A as, VP = V Q , X C = R .
X C 100
\When connected in series,
[Question is wrong. It should be choose Z = X 2C + R2 = 2 R
the correct statement].
p
5. Let i = i1 + i2 and f = , leading.
4
where, i1 = 5 A, i2 = 5 sin 100 w t A 1 p
\I = A, leading in phase by .
Average value of i1 = 5 A 4 2 4
Average value of i2 = 0 11. I = I 2R + ( I C - I L )2
\Average value of i = 5 A
Here, I C < I or I L > I
Another method
12. I = I L - I C = 0.2 A
È p ˘
i = 5 Í1 + cos ÊÁ + 100 w t ˆ˜˙ 13. For a pure inductor voltage leads with
Î Ë 2 ¯˚ p
current by .
È 2Ê p ˆ ˘ 2
= 5 Í2 cos Á + 50 w t ˜˙
Î Ë4 ¯˚ 14. VR = IR = 220V
p
= 10 cos2 È + 50 wt ˘ Hence it is condition of resonance, i.e.,
ÍÎ 4 ˙˚
VL = VC = 200 V
p 1
Average value of cos2 ÊÁ + 50 w t ˆ˜ = 15.
H1 I 2DC R
= 2 =
I2
=2
Ë4 ¯ 2 H2 I rms R ( I / 2)2
10
\ average value of i = = 5 A. I 20 R V02 R
2 16. H = I 2rms R = =
2 2( R + w2 L2 )
2
6. As voltage is leading with current, circuit
p 17. VL = IX L = Iw L
is inductive, and as f = ,X L = R
4 I
VC = IX C =
R R wC
or L= =
w 100
If w is very small,
7. As X C > X L voltage will lag with current.
VL ª = 0,VC ª V0 .
Again V = VR2 + ( VL - VC )2 = 10 V V
18. Resistance of coil, R = =4W
\ V < VC I
R V 4 When connected to battery
and cos f = = R = V 12
Z V 5 I= = = 1.5 A
R+r 4+4
Hence, a, b and c are wrong.
136
2 1
19. VR = V - VC2 =6V XC =
wC
VC 4
f = tan -1 = tan -1 1
VR 3 = =8 W
50 ¥ 2500 ¥ 10-6
20. VC = V 2 - VR2 = 16 V
Z = R2 + ( X L - X C )2 = 5 W
p
21. I = I 0 sin ÊÁ t + p ˆ˜ 2
Vrms R
Ë2 ¯ Average power = I 2rms R =
p 3p Z2
I = I 0 at + p = (12)2 ¥ 3
2 2 = = 17.28 W
(5)2
t =1s
V 25. Already X C > X L , with increase in w, X C
22. I0 = 0
2R further decrease in w, X C increases and
X L decreases, hence, I will decrease.
3
X C¢ = = 3R 26. For maximum current
wC
1 1
V I w = wr = =
fi I0 = 0 = 0 LC -6
1 ¥ 10 ¥ 4.9 ¥ 10-3
2R 2
VDC 12 105
23. R = = =3 W = rad/s.
I DC 4 7
27. In resonance,
24. X L = Z12 - R2 = (5)2 - (3)2
Z = R2P + X 2C ª 77 W
=4W
28. In resonance, cos f = 1.

More than One Correct Answers


1. VR2 + VL2 = 10000 …(i) P V
3. I = = 1 A, R = = 60 W
VL - VC = 120 …(ii) V I
VR2 + ( VL - VC )2 = (130)2 = 16900 …(iii) For AC,
100
On solving Z= = 100 W
1
Vr = 50 V, VL = 86.6 V, VC = 206.6 V
X C or X L = Z 2 - R2 = 80 W
V 50 5
and cos f = R = = XL 80 4
V 130 15 L= = = H
w 2p ¥ 50 5p
As VC > VL , circuit is capacitive in nature.
1 1 125
2. i = 3 sin w t + 4 cos w t or C= = = mF
w X C 2p ¥ 50 ¥ 80 p
= R sin ( w t + f)
4 V
R = 5 and f = tan -1 or R¢ + R =
3 I
2i 2R 10 fi R¢ = 100 - 60 = 40 W
im = 0 = =
p p p R Ï 1 if R = Z
4. cos f = =Ì
If V = Vm sin w t Z Ó 0 if R = 0
current will lead with the voltage. 5. As X L > X C , voltage will lead with the
If V = Vm cos w t current.
current will lag with voltage. Z = R2 + ( X L - X C )2 = 10 2 W
137
XL - XC p Vrms = VR2 + ( VL - VC )2 = 100 V ,
f = tan -1 = = 45∞
R 4
R 1 V0 = 100 2 V
cos f = = V
Z 2 8. I =
2
6. As X L > X C , w > wr Ê 1 ˆ
R2 + ÁÁ wL - ˜
with increase in w, X L and hence, Z will Ë w C ˜¯
increase while with decrease in w, Z will
first decrease and then increase. with change in L or C I may decrease or
1
V increase depending on effect on wL - .
7. X c = C = 50 W wC
I
VR = IR = 80 V
VL = IX L = 40 V
Match the Columns
1. (a) Æ (p, r), (b) Æ (q, r), (c Æ s), (d) Æ (p) 4. (a Æ q),
Concept based insertion. VR 40
R= = = 20 W
2. (a) Æ (p, s) current and voltage are in I 2
same phase so either X C = 0, X L = 0 (b Æ p)
or X C = X L π 0. VC = IX C = 2 ¥ 30 = 60 V
(b) Æ (q) (c Æ r)
I = - I 0 cos w t VL = IX C = 2 ¥ 15 = 30 V
p
= I 0 sin ÊÁ w t - ˆ˜ (d Æ s )
Ë 2¯ V = VR2 + ( VL - VC )2
f = 90∞ fi R = 0
= 50 V
(c) Æ (r, s) current is leading with voltage
p 5. (a Æ s) R is independent of f .
by , either X L = 0 or X C > X L 1
6 (b Æ p) X C µ
f
but X C and R are non-zero.
p 1
(d) Æ (s) current lags with voltage by , R (c Æ r ) X L µ
6 f
and X L are both non-zero. (d Æ q)
3. (a) Æ (q, s), (b) Æ (r, s), (c) Æ (r, s), Ê
2
1 ˆ
(d) Æ (r, s). Z = R2 + ÁÁ w L - ˜
Ë w C ˜¯
V V2r
I=and P = 2
Z Z i.e., first decreases then increases.
with increase in L, C or f , Z may increase
or decrease, hence power and current.

Vous aimerez peut-être aussi